Sie sind auf Seite 1von 166

MAT2125: Introduction to real analysis

Aaron Tikuisis

Winter 2018
Chapter 1

The real numbers

The field of real numbers, R, is a fundamental starting point for analysis. It


is rich with structure: we will begin with field operations (+, −, ×, ÷) which
are applied to finitely many elements at a time1 , and then order relations (<
, >, ≤, ≥); then we will see the operations of least upper bound and greatest
lower bound, which are applied to (typically infinite) sets of real numbers.
We will describe the real numbers by giving axioms; the real numbers R
is the unique structure satisfying these axioms. Proving that the object R
exists is a non-trivial feat, which will not be covered in this course. Rather,
we will take R and its axioms as given, and prove many other great things
using them.

1.1 Field axioms


Definition 1.1.1. A field is a set F equipped with two operations, addition
+ : F × F → F , and multiplication, · : F × F → F , which satisfy the
following properties:

(F1) For all a, b ∈ F , we have a + b = b + a; (+ is commutative)

(F2) For all a, b, c ∈ F , we have (a + b) + c = a + (b + c); (+ is associative)

(F3) There is an element 0 ∈ F such that a + 0 = 0 + a = a for all a ∈ F ;


(+ has an identity)
1
Later in the course, we discuss infinite sums, but we need to develop limits before this
is possible.

1
(F4) For any a ∈ F , there is an element −a ∈ F such that a + (−a) =
(−a) + a = 0; (+ has inverses)

(F5) For all a, b ∈ F , we have a · b = b · a; (· is commutative)

(F6) For all a, b, c ∈ F , we have (a · b) · c = a · (b · c); (· is associative)

(F7) There is an element 1 ∈ F such that 1 6= 0 and a · 1 = 1 · a = a for all


a ∈ F; (· has an identity)

(F8) For any a ∈ F \ {0}, there is an element a−1 ∈ F such that a · a−1 =
a−1 · a = 1; (· has inverses)

(F9) For any a, b, c ∈ F , (a + b) · c = a · c + b · c. ( distributivity)

Example 1.1.2. The real numbers R is a field. Q and C are also fields; N and
Z are not.
All of the valid operations in arithmetic are encoded by the field axioms.
In this course, you are allowed to use intuition about the consequences of
these field axioms (e.g., “if ab = ac and a 6= 0 then b = c” – this is true,
and can be proven directly from the axioms, but a proof from the axioms
isn’t expected). We will also use familiar notation from arithmetic, such as
a + b + c for (a + b) + c, a − b for a + (−b), ab for a · b, and ab for ab−1 .
Exercise 1.1.1. [TBB08, Exercise 1.3.5] Define Z/5Z to be the set {0, 1, 2, 3, 4}
with + and · defined by the following tables.

+ 0 1 2 3 4 · 0 1 2 3 4
0 0 1 2 3 4 0 0 0 0 0 0
1 1 2 3 4 0 1 0 1 2 3 4
2 2 3 4 0 1 2 0 2 4 1 3
3 3 4 0 1 2 3 0 3 1 4 2
4 4 0 1 2 3 4 0 4 3 2 1

Prove that Z/5Z is a field.


Exercise 1.1.2. [TBB08, Exercise 1.3.6] Define Z/6Z to be the set {0, 1, 2, 3, 4, 5}

2
with + and · defined by the following tables.
+ 0 1 2 3 4 5 · 0 1 2 3 4 5
0 0 1 2 3 4 5 0 0 0 0 0 0 0
1 1 2 3 4 5 0 1 0 1 2 3 4 5
2 2 3 4 5 0 1 2 0 2 4 0 2 4
3 3 4 5 0 1 2 3 0 3 0 3 0 3
4 4 5 0 1 2 3 4 0 4 2 0 4 2

Prove that Z/6Z isn’t a field. Determine precisely which axioms F1–F9 don’t
hold in Z/6Z.

1.2 Order structure


Definition 1.2.1. An ordered field is a field F equipped with a binary rela-
tion < satisfying the following properties
(O1) If a < b and b < c then a < c; (< is transitive)
(O2) For all a, b ∈ F , exactly one of the following hold: a = b or a < b or
b < a; (< is a total order)
(O3) For all a, b, c ∈ F , if a < b then a + c < b + c; (< is compatible with
+)
(O4) For all a, b, c ∈ F , if a < b and 0 < c then a · c < b · c; (< is
compatible with ·)
Example 1.2.2. The real numbers R is an ordered field. So is Q, but C is not
(if we try to use the operation a + bi < c + di if a = c and b < d, then (O2)
fails; see Exercise 1.2.3 to see that no other relation works).
We can define ≤, >, ≥ in terms of <:

a ≤ b means a < b or a = b,
a > b means b < a, and
a ≥ b means a > b or a = b.

Exercise 1.2.1. Prove that if F is an ordered field then 1 > 0 in F .


[ Hint. If not, then 0 > 1; in this case, prove that −1 > 0 and use this to
prove 1 > 0 after all.]

3
Exercise 1.2.2. Prove that if F is an ordered field then for all a, b, c ∈ F , if
a < b and c < 0 then a · c > b · c.
Exercise 1.2.3. Prove that for any binary relation < on C, we do not get an
ordered field.
[Hint. By contradiction. Consider two cases, depending on whether i > 0 or
i < 0.]
Exercise 1.2.4. Prove that for any binary relation on Z/5 (from Exercise
1.1.1), we do not get an ordered field. More generally, if F is a field in which
1 + · · · + 1 = 0 (some finite, nonempty sum), then for any binary relation on
F , we do not get an ordered field.
Exercise 1.2.5. Let F be an ordered field and let a, b ∈ F . Show that if a < b
then there exists c ∈ F such that

a < c < b.

[Hint. Use the previous exercise to justify that one can average a and b.]

1.3 Bounded sets, infima, and suprema


Definition 1.3.1. Let F be an ordered field, let S ⊆ F be a subset, and let
a ∈ F . a ∈ F is an upper bound for S if for any x ∈ S,

x ≤ a.

a ∈ F is a lower bound for S if for any x ∈ S,

a ≤ x.

S is bounded above if there exists an upper bound for S. S is bounded below


if there exists a lower bound for S. S is bounded set if it is bounded above
and bounded below.
Example 1.3.2. Consider S := [0, 1] = {x ∈ R : 0 ≤ x ≤ 1}. An upper
bound for S is 1. Another upper bound for S is 2. A lower bound for S is 0.
Another lower bound for S is −100. S is bounded.
Example 1.3.3. Consider N≥1 = {1, 2, 3, . . . }. This set is not bounded above.
(However, we don’t have enough information about R to prove this yet! It
will be Theorem 1.3.13.) It is bounded below, for example by 0.

4
Example 1.3.4. Consider T := {1/n : n ∈ N≥1 }. Then T is bounded below
by 0 and bounded above by 1.
Sometimes a set S is bounded below by an element a in that set: for
example [0, 1] is bounded below by 0, which is in [0, 1]. In such a case, we
say that a is the minimum of S, and write a = min S. Likewise, if a set S
contains an upper bound b, then b is the maximum of S, and write a = max S.
However, some bounded sets don’t contain their bounds. For
example, T := {1/n : n ∈ N≥1 } doesn’t contain any element which is a lower
bound for T (Exercise 1.3.1).
Remember: when you write a = min S or a = max S, this implies
that a ∈ S.

Definition 1.3.5. Let F be an ordered field, let S ⊆ F be a nonempty subset,


and let a ∈ F . a is a least upper bound(or supremum) for S if:

(i) it is an upper bound for S, and

(ii) for any upper bound b of S, we have b ≥ a.

When a is a least upper bound for S, we write

a = sup S.

Similarly, a is a greatest lower bound (or infimum) for S, written a =


inf S, if

(i) it is a lower bound for S, and

(ii) for any lower bound b of S, we have b ≤ a.

By writing a = sup S or a = inf S, it suggests that the supremum and


infimum of a set (if they exist) are unique; indeed, this is the case (Exercise
1.3.3).
We also have the following conventions:

• If S is not bounded above, we write sup S = ∞.

• If S is not bounded below, we write inf S = −∞.

• We write sup ∅ = −∞ and inf ∅ = ∞.

5
We emphasize however that ±∞ are not real numbers, and we are not
allowed to manipulate them as if they were.
(Note that every real number is both an upper bound and a lower bound
for ∅ – so although ∅ has upper and lower bounds, it does not have least,
greatest ones respectively.)
Example 1.3.6. Consider the set S := (0, 1) = {x ∈ R : 0 < x < 1}. Then 1
is the least upper bound for S. Certainly, by definition, we see that 1 is an
upper bound. For (ii), suppose for a contradiction that b is another upper
bound and b ≤ 1 doesn’t hold. Then since < is a total order, it follows that
b < 1. By Exercise 1.2.5, we can find x ∈ R such that b < x < 1. But then
x ∈ S and since b < x, this contradicts that b is an upper bound.
Example 1.3.7. If a = max S then a = sup S. To see this, first a must be an
upper bound. For (ii), suppose that b is another upper bound for S. Then
since a ∈ S and b ≥ x for all x ∈ S, we have b ≥ a.

Definition 1.3.8. An ordered field F is complete if for any nonempty set


S ⊆ F which is bounded above, there is an element a ∈ F which is a least
upper bound for S. In short, if S ⊆ F is nonempty and bounded above, then
sup S exists.

Example 1.3.9. The real numbers R is complete. (We will not prove this –
but rather take it as a fundamental fact.)
Interesting fact 1.3.10. In fact (although we also won’t prove it), R is the
unique ordered field which is unique: in other words, if F is some other
ordered field which is complete then there is a bijection φ : F → R which
preserves +, ·, ≤, i.e., for all a, b ∈ F ,

φ(a + b) = φ(a) + φ(b),


φ(a · b) = φ(a) · φ(b),

and
a ≤ b if and only if φ(a) ≤ φ(b).

Proposition 1.3.11. For any set S ⊆ R which is nonempty and bounded


below, inf S exists.

Proof. Exercise 1.3.4


Here is an example of how one might use completeness.

6
Proposition 1.3.12. There exists a real number a ∈ R such that a2 = 2.

Proof. Set
S := {x ∈ R : x ≥ 0 and x2 ≤ 2}.
Claim. For b > 0, b is an upper bound for S if and only if b2 ≥ 2.
To see this, first suppose b2 ≥ 2. If b isn’t an upper bound then there is
x ∈ S such that x > b. But then x2 > x · b > b2 ≥ 2, which contradicts that
x ∈ S.
Conversely, suppose that b is an upper bound for S and, for a contradic-
tion, that b2 < 2. Clearly b ≥ 1. Then set  := 2 − b2 > 0 and x := b + 3b ,
and observe that
2b 2
x2 = b 2 + + 2
3b 9b
2 2
≤ b2 + + (since  ≤ 2)
3 9
2
<b +
= 2.

This means that x ∈ S, which is a contradiction since x > b. This proves


the claim.
We see from the claim that S is bounded above, for example by 2. By
the completeness of R, we may set a := sup S.
Since a is an upper bound for S, we have a2 ≥ 2. If a2 6= 2 then a2 > 2,

and we may set  := a2 − 2 > 0 and b := a − 2a . Then

2 2
b2 = a2 − 2a + = 2 + ≥ 2.
4 4
Thus, by the claim, b is an upper bound for S, but since b < a, this contradicts
that a is the least upper bound.
The following may seem obvious, but it is an important consequence of
completeness.

Theorem 1.3.13 (The Archimedean Property). The set N≥1 is not bounded
above.

Proof. Suppose for a contradiction that N≥1 were bounded above. Then by
completeness, it would have a least upper bound, a = sup N≥1 . Since a is the

7
least upper bound, a − 1 is not an upper bound, so that there exists some
m ∈ N≥1 such that
m > a − 1.
But then m + 1 ∈ N≥1 and m + 1 > a, contradicting that a is an upper
bound.
Exercise 1.3.1. Let T := {1/n : n ∈ N≥1 }.

(a) Show that the set T doesn’t contain any element which is a lower bound
for T . In other words, we are not allowed to write min T .

(b) Determine inf T (and prove your answer is correct).

Exercise 1.3.2. Show that every finite set is bounded, and contains its bounds.
Exercise 1.3.3. Here we show that suprema are unique (when they exist).
Let F be an ordered field, let S be a subset of F and suppose that a and a0
are both least upper bounds for S. Prove that a = a0 .
Exercise 1.3.4. Prove Proposition 1.3.11.
Exercise 1.3.5. Find the inf, sup, min, and max of the following sets, or show
that they don’t exist.

(a) Z;

(b) N≥1 ;

Exercise 1.3.6. (a) Define S := {x ∈ Q : x2 ≤ 2}. Prove carefully (along the


lines of the proof of Proposition 1.3.12); that a := inf S satisfies a2 = 2. (b)
Prove (without using the fact mentioned in Interesting Fact 1.3.10) that Q
is not complete.
Exercise 1.3.7. Define p(x) := x2 − x and S := {x > 0 : p(x) ≤ 1}. Prove,
along the lines of the proof of Proposition 1.3.12 (and definitely without using
quadratic formula) that a := sup S exists and satisfies p(a) = 1.
Exercise 1.3.8. Let S ⊆ Z be a nonempty set which is bounded above, and
set p := sup S. Prove that p ∈ Z (i.e., that p = max S).

8
1.4 The absolute value and distances between
numbers
The absolute value of a real number a ∈ R is defined by
(
a, if a ≥ 0;
|a| :=
−a, if a < 0.

In other words, |a| = max{a, −a}.


Here are the basic properties of | · | : R → [0, ∞).
Proposition 1.4.1. Let x, y ∈ R. Then
(i) | − x| = |x|,

(ii) −|x| ≤ x ≤ |x|,

(iii) |xy| = |x| · |y|,

(iv) |x + y| ≤ |x| + |y| ( triangle inequality), and



(v) |x| − |y| ≤ |x − y|.
Proof. Exercise 1.4.1.
Definition 1.4.2. Let x, y ∈ R. The distance between x and y is

d(x, y) := |x − y|.

Here are the basic properties of the distance d : R × R → [0, ∞).


Proposition 1.4.3. Let x, y, z ∈ R. Then
(i) d(x, y) = d(y, x), ( d is symmetric)

(ii) d(x, y) = 0 iff x = y, and ( distinct points are separated)

(iii) d(x, z) ≤ d(x, y) + d(y, z). (triangle inequality)


Proof. Exercise 1.4.2.
Exercise 1.4.1. Prove Proposition 1.4.1.
Exercise 1.4.2. Prove Proposition 1.4.3.

9
Exercise 1.4.3. [TBB08, Exercise 1.10.3] Let x, L ∈ R and let  > 0. Show
that the following are equivalent:

(i) |x − L| < ,

(ii) L −  < x < L + .

These conditions will appear prominently in the definitions of limits.


Exercise 1.4.4. [Sav17, Exercise 1.6.7] Let S ⊆ R be a bounded set. Prove
that
sup{|a − b| : a, b ∈ S} = sup A − inf A.

10
Chapter 2

Sequences

2.1 Sequences and boundedness


Definition 2.1.1. A sequence of real numbers is a function f : N≥1 → R.

The above is the formal definition of a sequence. However, in practice,


we think of a sequence f : N≥1 → R as an infinite list of numbers

f (1), f (2), f (3), . . . .

As a convention, we rarely write a sequence as a function, and instead we


use the following two ways of writing sequences:

(an )∞
n=1 or (a1 , a2 , a3 , . . . ).

In both cases, the sequence corresponds to a function f : N≥1 → R by


an = f (n).

Definition 2.1.2. A sequence (an )∞


n=1 is bounded if the set {an : n ∈ N≥1 } ⊆
R is bounded. Likewise, we may define (an )∞ n=1 to be bounded above or
bounded below, by the corresponding property for {an : n ∈ N≥1 }.

Note that in many textbooks (including the ones recommended in the


syllabus), sequences are denoted by {an }∞
n=1 or even {an }. We emphasize
that sequences are not sets, for two reasons:

(i) The order in a sequence matters. Sets, by contrast, are not ordered:
{1, 2} = {2, 1}.

11
(ii) Repeats in a sequence matter. Sets, by contrast, do not see repeats:
{1, 1} = {1}.
Exercise 2.1.1. Determine which of the following sequences are bounded.
(a) (1, 2, 3, 4, . . . )
(b) (1, 21 , 13 , 41 , . . . )
(c) (an )∞
n=1 defined by
(
k, if n = 2k for some k ∈ N≥1 ;
an :=
0, otherwise.

(d) (an )∞
n=1 defined by

a1 := 1,
a2 := 2,
an−1 + nan−2
an := , for n ≥ 3.
n+1
Exercise 2.1.2. Prove parts (ii)-(iv) of Proposition 2.3.1

2.2 Convergence
One of the primary things that we are interested in with sequences is their
“long-term behaviour”: that is, what an looks like when n is large. To this
end, the first thing we might ask is: does an get close to some particular
number, as n gets large? This is what convergence of sequences means. Here
is the formal definition:
Definition 2.2.1. Let (an )∞
n=1 be a sequence of real numbers and let L ∈ R.

We say that (an )n=1 converges to L if for every  > 0 there exists n0 ∈ N≥1
such that for all n ≥ n0 ,
|an − L| < .
In this case, we may write
L = lim an or an → L as n → ∞.
n→∞

If a sequence does not converge to any real number, then we say it di-
verges.

12
1 ∞
Example 2.2.2. The sequence (an )∞

n=1 = n n=1
converges to 0. To prove
this, we work from the definition.
Let  > 0 be given. We must prove that there exists n0 ∈ N≥1 such that
for all n ≥ n0 ,
|an − 0| < .
By the Archimedean Property (Theorem 1.3.13) there exists n0 such that
n0 > 1 . Then if n ≥ n0 ,
1 1
≤ < .
n n0
1
Since n
> 0, we have |an − 0| = | n1 | < .
Example 2.2.3. Let us prove that

2n2 + 1
lim = 2.
n→∞ n2 + n

To prove this, we first rearrange


2 2
2n + 1 − 2(n2 + n)

2n + 1
n2 + n − 2 =

n2 + n

1 − 2n
= 2
n + n
1 2n
≤ 2 + 2
n +n n +n
1 2
= 2 +
n +n n+1
1 2
≤ + .
n n
Now, given  > 0, pick n0 such that n0 > 3 . Then if n ≥ n0 ,
2
2n + 1 3
− 2 ≤ < 3 < .
n2 + n n n0

13
Proposition 2.2.4 (Uniqueness of limits). Let (an )∞
n=1 be a sequence and let
L1 , L2 ∈ R. If
lim an = L1 and lim an = L2
n→∞ n→∞

then L1 = L2 .
Proof. Suppose for a contradiction that L1 6= L2 , so without loss of generality,
L1 < L2 . Then define  := L2 −L
2
1
.

Since lim an = L1 , there exists n0 such that for all n ≥ n0 ,


n→∞

L1 −  < an < L1 + .1

Using the second inequality and the definition of , we have for n ≥ n0 , that
L1 + L2
an < L1 +  = .
2
Likewise, since lim an = L2 , there exists m0 such that for all n ≥ m0 ,
n→∞

L2 −  < an < L2 + ,

and from the first inequality we get for n ≥ m0 ,


L1 + L2
an > L2 −  = .
2
Take n := max{n0 , m0 }, so that n ≥ n0 and n ≥ m0 . Then we have
L1 + L2
an < < an ,
2
a contradiction.
Complementing the definition of limits is the following notion of diver-
gence to ±∞.
1
We are using Exercise 1.4.3 here.

14
Definition 2.2.5. Let (an )∞ n=1 be a sequence of real numbers. We say that

(an )n=1 diverges to ∞ if for every R > 0 there exists n0 ∈ N≥1 such that for
all n ≥ n0 ,
an > R.
Likewise, we say that (an )∞
n=1 diverges to −∞ if for every R > 0 there exists
n0 ∈ N≥1 such that for all n ≥ n0 ,

an < −R.

If (an )∞
n=1 diverges to ∞, we may write

∞ = lim an or an → ∞ as n → ∞.
n→∞

If it diverges to −∞, we may write

−∞ = lim an or an → −∞ as n → ∞.
n→∞

Again, we caution that ∞ is not a real number, so we must be careful


not to treat it as one when we write something like lim an = ∞.
n→∞
Example 2.2.6. lim n = ∞. To prove this, given R, we just choose n0 ≥ R.
n→∞

Example 2.2.7. The sequence ((−1)n )∞


n=1 diverges, but it does not diverge to
∞ or −∞ (Exercise 2.2.1). Note that this sequence is in fact bounded.
Proposition 2.2.8. Let (an )∞n=1 be a sequence which converges to some num-
ber L ∈ R. Then (an )∞
n=1 is bounded.
Proof. Since lim an = L, using  := 1, there exist n0 such that for all n ≥ n0 ,
n→∞

|an − L| < 1.

Take
M := max{a1 , a2 , . . . , an0 −1 , L + 1}.
(This maximum exists since the set is finite.)
Claim. M is an upper bound for the set {an : n ∈ N≥1 }.
To see this, let n ∈ N≥1 , and we must show an ≤ M . If n < n0 then an
is among the list a1 , . . . , an0 −1 , so that an ≤ M by the definition of M . On
the other hand, if n ≥ n0 then by the choice of n0 , we have an < L + 1 ≤ M .
This proves the claim.

15
Setting
M 0 := min{a1 , . . . , an0 −1 , L − 1},
then the same reasoning shows that M 0 is a lower bound for the set {an : n ∈
N≥1 }. Thus the sequence is bounded both above and below, as required.
A couple notes.

• The previous proposition can be reformulated as: if a sequence is not


bounded then it diverges. This can be used to show that a sequence
which diverges to ±∞ does not converge (a fact which is suggested by
the terminology “diverges to ±∞”).

• The converse of the previous proposition does not hold: a sequence


which is bounded might not converge (Example 2.2.7).

Exercise 2.2.1. Prove that the sequence ((−1)n )∞


n=1 from Example 2.2.7 does
not converge.
[Begin by supposing it does converge to some L ∈ R, and reach a contradic-
tion.]
 ∞
Exercise 2.2.2. Prove that √2n converges to 0. Prove this directly from
n=1
the definition, rather than using any theorems or rules (for example from
previous calculus courses).
Exercise 2.2.3. Suppose that (an )∞
n=1 is a sequence that converges to some

L ∈ R, and define a sequence (bn )n=1 by

bn := a2n , n ∈ N≥1 .

Prove that (bn )∞


n=1 also converges to L.

Exercise 2.2.4. Suppose that (an )∞ ∞


n=1 and (bn )n=1 are sequences that both
converge to some L ∈ R, and define a sequence

(cn )∞
n=1 = (a1 , b1 , a2 , b2 , a3 , b3 , . . . ).

Prove that (cn )∞


n=1 converges to L.

Exercise 2.2.5. [TBB08, Exercise 2.4.15 and 2.4.16] If (an )∞


n=1 is a sequence
of nonnegative real numbers
√ converging to L ≥ 0, prove that the sequence
√ ∞
( an )n=1 converges to L. [Hint. You might find it easiest to treat the cases
L = 0 and L > 0 separately.]

16
Exercise 2.2.6. Let (an )∞
n=1 be a sequence of nonzero numbers. Prove that if
lim an = ∞ then lim a1n = 0. Is the converse true?
n→∞ n→∞
Exercise 2.2.7. Let (an )∞ ∞
n=1 be a bounded sequence and let (bn )n=1 be a se-

quence that converges to 0. Prove that (an bn )n=1 converges to 0.

2.3 Properties of limits


In this section, we will establish many of the tools which one likes to apply
in everyday limit computations.
Proposition 2.3.1 (Algebra of limits). Let (an )∞ ∞
n=1 and (bn )n=1 be sequences
converging to La and Lb respectively, and let c ∈ R. Then
(i) (an + bn )∞
n=1 converges to La + Lb .

(ii) (can )∞
n=1 converges to cLa .

(iii) (an bn )∞
n=1 converges to La Lb .
 ∞
1 1
(iv) If an 6= 0 for all n and La 6= 0 then an
converges to La
.
n=1
Remark 2.3.2. (i) and (ii) of the above proposition say that the set V of
converging sequences forms a vector space, and that the map (an )∞ n=1 7→
lim an is a linear map from V to R.
n→∞

Proof. (i): Let  > 0 be given. Since lim an = La , there exists na such that
n→∞
for all n ≥ na , |an − La | < 2 . Since lim bn = Lb , there exists nb such that
n→∞
for all n ≥ nb , |bn − Lb | < 2 . Set n0 := max{na , nb }. Then for n ≥ n0 ,
|(an + bn ) − (La + Lb )| ≤ |an − La | + |bn − Lb |
 
< + = .
2 2
(since n ≥ na and n ≥ nb ).
(ii)-(iv) are Exercise 2.1.2
Proposition 2.3.3. Let (an )∞ ∞
n=1 and (bn )n=1 be converging sequences. If

an ≤ b n for all n
then
lim an ≤ lim bn .
n→∞ n→∞

17
Proof. This is very similar to the proof of Proposition 2.2.4. Suppose for a
contradiction that La > Lb . Define  := La −L
2
b
. Since lim an = La , there
n→∞
exists na such that for all n ≥ na ,
La + Lb
La +  > an > La −  = .
2
Likewise, since lim bn = Lb , there exists nb such that for all n ≥ nb ,
n→∞

La + Lb
Lb −  < bn < Lb +  = .
2
La +Lb
Using n := max{na , nb }, we get an > 2
> bn , a contradiction.
Corollary 2.3.4. Let (an )∞
n=1 be a converging sequence such that

m ≤ an ≤ M for all n.

Then
m ≤ lim an ≤ M.
n→∞

Proof. Since lim M = M , we apply the previous proposition with (bn )∞


n=1 :=
n→∞
(M, M, M, . . . ) to get the second inequality. Similarly for the first.
Theorem 2.3.5 (Squeeze Theorem). Let (an )∞ ∞ ∞
n=1 , (bn )n=1 , (cn )n=1 be sequences
such that:
(i) (an )∞ ∞
n=1 and (cn )n=1 converge to the same number L, and

(ii) an ≤ bn ≤ cn for all n.


Then (bn )∞
n=1 also converges to L.

Proof. Let  > 0 be given. Using the definition of convergence for both
(an )∞ ∞
n=1 and (bn )n=1 and taking the maximum n0 from both, there exists n0
such that for all n ≥ n0 ,

L −  < an < L +  and L −  < cn < L + .

Thus,
L −  < a n ≤ bn ≤ c n < L + 
for all n ≥ n0 , as required.

18
sin(n)
Example 2.3.6. Let us show that lim n
= 0. We use an := − n1 , bn :=
n→∞
sin(n)
n
, and cn := n1 . Since −1 ≤ sin(x) ≤ 1 for all x, it follows that
an ≤ b n ≤ c n for all n.
Moreover, by Example 2.2.2, cn → 0, and then using Algebra of Limits,
an → −0 = 0. Hence the Squeeze Theorem applied and shows that bn → 0.
Here is a variant on the Squeeze Theorem for divergence to ±∞.
Proposition 2.3.7. Let (an )∞ ∞
n=1 and (bn )n=1 be sequences such that

an ≤ b n for all n.
Then
(i) If lim an = ∞ then lim bn = ∞.
n→∞ n→∞

(ii) If lim bn = −∞ then lim an = −∞.


n→∞ n→∞

Proof. Exercise 2.3.1


Example 2.3.8. Fix r ∈ R. The sequence (rn )∞ n=1 is an important sequence
called the geometric sequence. Let us analyze its convergence behaviour.
The cases r = 0 and r = 1 are clear, as in these cases, the sequence is
constant and so the limit exists (and equals r).
If r > 1, then set x := r − 1 > 0. By the Binomial Theorem, we have
 
n n n 2
r = (1 + x) = 1 + nx + x + · · · + xn ≥ nx.
2
Since nx → ∞, it follows by Proposition 2.3.7 that rn → ∞.
If r ∈ (0, 1), then set s := r−1 , so that rn = s1n . Since sn → ∞, it follows
by Exercise 2.2.6 that rn → 0.
If r ∈ (−1, 0) then set s := |r| ∈ (0, 1). We have
−sn ≤ rn ≤ sn
for all n, and by the previous case, sn → 0. Hence −sn → 0 also and then
by the Squeeze Theorem, rn → 0.
If r = −1 then we already saw in Example 2.2.7 that the sequence (rn )∞
n=1
diverges.
Finally, if r < −1 then since |r|n → ∞, we see that the sequence (rn )∞
n=1
is bounded neither above (due to the even terms) nor below (due to the odd
terms). It diverges, but doesn’t diverge to ±∞.

19
Here is an important fact.

Proposition 2.3.9. For any real number α ∈ R, there exists a sequence


(an )∞
n=1 of rational numbers (i.e., an ∈ Q for all n) such that

lim an = α.
n→∞

Proof. We will construct an such that


1 1
α− ≤ an ≤ α + . (2.1)
n n
It will follow from the Squeeze Theorem (Theorem 2.3.5) that

lim an = α.
n→∞

We may find an integer p ∈ Z such that

nα − 1 ≤ p ≤ nα + 1

(For example, let p be the supremum of {k ∈ Z : k ≤ nα}, a set which is


bounded above by nα, and is nonempty by the Archimedean Property. By
Exercise 1.3.8, p ∈ Z. Also, we have p ≤ nα + 1, or else p − 1 would be a
smaller upper bound for the set, and on the other hand, p ≥ nα − 1, or else
p + 1 would be in the set, contradicting that p is an upper bound.)
Setting an := np , the inequalities (2.1) clearly hold.
Exercise 2.3.1. Prove Proposition 2.3.7.
Exercise 2.3.2. Let (an )∞ ∞
n=1 and (bn )n=1 be two convergent sequences, such
that an < bn for all n. Must it hold that

lim an < lim bn ?


n→∞ n→∞

Give a proof or counterexample.


Exercise 2.3.3. [TBB08, Exercise 2.7.5] Let (an )∞ ∞
n=1 and (bn )n=1 be sequences.
Which of the following is true?

(a) If (an )∞ ∞ ∞
n=1 and (bn )n=1 are both divergent, then so is (an + bn )n=1 .

(b) If (an )∞ ∞ ∞
n=1 and (bn )n=1 are both divergent, then so is (an bn )n=1 .

20
(c) If (an )∞ ∞ ∞
n=1 and (an + bn )n=1 are both convergent, then so is (bn )n=1 .

(d) If (an )∞ ∞ ∞
n=1 and (an bn )n=1 are both convergent, then so is (bn )n=1 .
 ∞
(e) If (an )∞
n=1 is convergent, then so is 1
an
.
n=1

(f) If (an )∞ 2
n=1 is convergent, then so is ((an ) )n=1 .

(g) If ((an )2 )n=1 is convergent, then so is (an )∞
n=1 .

(h) If ((an )2 )n=1 is convergent and an ≥ 0 for all n, then so is (an )∞
n=1 .

Exercise 2.3.4. [TBB08, Exercise 2.8.9] Let (an )∞


n=1 be a sequence of positive
an an
numbers. Prove that if lim an+1 exists and is strictly less than 1 ( lim an+1 <
n→∞ n→∞
1) then
lim an = 0.
n→∞

2.4 Monotone convergence criterion


Here we study sequences which are monotone, meaning that they always tend
in the same direction (increasing or decreasing).

Definition 2.4.1. Let (an )∞


n=1 be a sequence.

(i) We say (an )∞


n=1 is (weakly) increasing if

a1 ≤ a2 ≤ a3 ≤ · · · .

(ii) We say (an )∞


n=1 is strictly increasing if

a1 < a2 < a3 < · · · .

(iii) We say (an )∞


n=1 is (weakly) decreasing if

a1 ≥ a2 ≥ a3 ≥ · · · .

(iv) We say (an )∞


n=1 is strictly decreasing if

a1 > a2 > a3 > · · · .

21
(v) Finally, we say (an )∞
n=1 is monotone if it is either increasing or de-
creasing.

Note that in some textbooks (including [TBB08]), “increasing” is used


where we use “strictly increasing”, and “nondecreasing” is used where we
use “increasing”.2 In our context, the concept of (weakly) increasing and
decreasing sequences is most important, and strictness is secondary.
The following is a powerful result about monotone sequences: the con-
verge exactly when they are bounded.

Theorem 2.4.2 (Monotone Convergence Criterion). Let (an )∞ n=1 be a mono-


tone sequence. Then it converges if and only if it is bounded.
Specifically, if (an )∞
n=1 is bounded and increasing, then it converges to
sup{an : n ∈ N≥1 }, whereas if it is bounded and decreasing, then it converges
to inf{an : n ∈ N≥1 }.

Proof. If the sequence converges then it is bounded by Proposition 2.2.8. For


the converse, we will do the case that (an )∞
n=1 is bounded and increasing. The
decreasing case is exactly the same (or alternatively, it can be recovered from
the increasing case by considering the sequence (−an )∞ n=1 ).
Set L := sup{an : n ∈ N≥1 }. To show convergence, let  > 0 be given.
Since L is the least upper bound for {an : n ∈ N≥1 }, L −  is not an upper
bound, so there exists n0 such that

L −  < an0 .

This is our n0 ; now suppose n ≥ n0 . Then since the sequence is increasing,

an ≥ an0 > L − .

On the other hand, since L is an upper bound, an ≤ L. Altogether,

L −  < an ≤ L < L + ,

as required.
2
This use of “nondecreasing” is confusing. It is natural to think that “nondecreas-
ing” means “not decreasing”, but there are sequences which are neither increasing nor

decreasing, such as ((−1)n )n=1 .

22
 ∞
Example 2.4.3. Consider the sequence √1 . This sequence is decreasing,
n
n=1
since for all n, we have
√ √ 1 1
n≤n+1⇒ n≤ n+1⇒ √ ≥ √ .
n n+1
It is also bounded below by 0. By the Monotone Convergence Criterion, it
follows that this sequence converges to its infimum, which is 0.
Exercise 2.4.1. [Sav17, Exercise 2.3.1] Suppose that (an )∞ ∞
n=1 and (bn )n=1 are
sequences such that (an )∞
n=1 is increasing, an ≤ bn for all n, and (bn )n=1


converges. Prove that (an )n=1 converges.
Exercise 2.4.2. [TBB08, Exercise 2.9.2] Define a sequence (an )∞
n=1 by

a1 := 1,

an+1 := 1 + an , n ≥ 1.

Prove that this sequence is increasing, and determine whether it converges.


Exercise 2.4.3. Let (an )∞ ∞ ∞
n=1 , (bn )n=1 , and (cn )n=1 be sequences such that

an ≤ b n ≤ c n for all n.

(a) If (an )∞ ∞ ∞
n=1 and (cn )n=1 are both increasing, must (bn )n=1 be?

(b) If (an )∞ ∞ ∞
n=1 and (cn )n=1 are both bounded, must (bn )n=1 be?

(c) If (an )∞ ∞ ∞
n=1 and (cn )n=1 are both convergenc, must (bn )n=1 be?

2.5 Subsequences
Given a sequence
(a1 , a2 , a3 , . . . )
we may want to forget parts of the sequence and only take certain terms.
What we get when we do this is a subsequence. Here is the formal definition.

Definition 2.5.1. Let (an )∞n=1 be a sequence. A subsequence is a sequence


of the form (ank )∞
k=1 , where (nk )∞
k=1 is a strictly increasing sequence with
nk ∈ N≥1 for all k.

23
One needs to be careful not to get confused with the notation of subse-
quences. Note that we typically index a subsequence with a different variable
(k instead of n in the above definition). Since the subsequence arises from the
sequence, we typically don’t write something like (bn )∞ n=1 for a subsequence.
If we think of a sequence as a list, a subsequence arises by erasing some
of the entries of the list (specifically, erasing all of the entries that aren’t in
{nk : k ∈ N≥1 }).
Since we require the indices of the subsequence, (nk )∞ k=1 , to be strictly
increasing, the following are not examples of subsequences:

(a1 , a1 , a1 , . . . ),
(a2 , a1 , a4 , a3 , a6 , a5 , . . . ).

Example 2.5.2. Let (an )∞


n=1 be a sequence. Then the sequence of even terms
(a2n )n=1 is a subsequence. (We could write this sequence as (ank )∞

k=1 by
setting nk := 2k.)
Example 2.5.3. Define (an )∞
n=1 by

1
an := n + .
n
Here are a couple subsequences of this sequence:

(i) Taking nk := 2k , we have

(ank )∞ −1 2 −2 3 −3
k=1 = (1 + 1, 2 + 2 , 2 + 2 , 2 + 2 , . . . ).

(ii) Taking nk := k + 1, we have


 ∞
1
(ank )∞
k=1 = k+1+ .
k+1 k=1

Proposition 2.5.4. If (an )∞


n=1 converges to L, then any subsequence also
converges to L.

Proof. Exercise 2.5.1.

Proposition 2.5.5. Every sequence contains a monotone subsequence.

24
Proof. Let (an )∞n=1 be a sequence. We begin by trying to construct an de-
creasing sequence in a naı̈ve way: we look for points m of the sequence where
am is bigger than all later terms (i.e., am ≥ an for all n ≥ m). We’ll call such
indices m “turn-back points”. The idea is that, if there are infinitely many
of these, then we will get an decreasing sequence by taking them all; if there
are only finitely many, then there must be a increasing subsequence. Let us
explain: we have two cases.
Case 1. There are infinitely many turn-back points. In this case, we may
take a sequence n1 < n2 < · · · such that each nk is a turn-back point.
In particular, we have an1 ≥ an2 ≥ · · · , so the subsequence (ank )∞ k=1 is
decreasing.
Case 2. There are only finitely many turn-back points. In this case, let
M be the last turn-back point, so that every n > M is not a turn-back
point. Take some n1 > M ; since it is not a turn-back point, there is some
n2 > n1 such that an1 < an2 . Likewise, n2 is not a turn-back point, so there
is some n3 > n2 such that an2 < an3 . Continuing in this way, we obtain a
subsequence (ank )∞k=1 that is strictly increasing.

Here is an important corollary of the previous proposition.


Corollary 2.5.6 (Bolzano–Weierstrass Theorem). Every bounded sequence
has a convergent subsequence.
Proof. Let (an )∞
n=1 be a bounded sequence. By the previous proposition,
it has a subsequence (ank )∞
k=1 which is monotone. This subsequence is,
of course, also bounded. Hence by the Monotone Convergence Criterion,
(ank )∞
k=1 converges.

Exercise 2.5.1. Prove Proposition 2.5.4.


Exercise 2.5.2. [TBB08, Exercise 2.11.3] Suppose that (an )∞ ∞
n=1 and (bn )n=1
are sequences and (ank )∞ ∞
k=1 , (bmk )k=1 (respectively) are subsequences. Must
(ank + bmk )k=1 be a subsequence of (an + bn )∞

n=1 ?

Exercise 2.5.3. Let (an )∞ ∞


n=1 be a sequence. Prove that (an )n=1 is not bounded
above if and only if it has a subsequence (ank )∞
k=1 which diverges to ∞.
Exercise 2.5.4. [TBB08, Exercise 2.11.11] Give an example of a sequence
(an )∞
n=1 such that:

(i) For every natural number k ∈ N≥1 , there is a subsequence of (an )∞


n=1
converging to k, and

25
(ii) For every α ∈ R, if α is a limit of a subsequence of (an )∞
n=1 , then
α ∈ N≥1 .
Exercise 2.5.5. Let (an )∞
n=1 be a sequence. Suppose that for every M , there
exists a subsequence (ank )∞ 1
k=1 such that lim ank = M . Prove that there is a
k→∞
subsequence (amk )∞
k=1 such that lim amk = 0. Is it possible to even arrange
k→∞
that (amk )∞
k=1 is monotone?

2.6 Cauchy sequences


It is useful (especially in developing further theory) to characterize when a
sequence converges, in a way that doesn’t require knowing what the sequence
converges to. This is what we will do in this section. Note that the Monotone
Convergence Theorem does this, in the special case of monotone sequences:
it says that a monotone sequence converges if and only if it is bounded; the
condition “it is bounded” makes no reference to any (potential) value for the
limit. But that theorem is limited to monotone sequences, and we already
know that the condition “it is bounded” will not characterize convergence
for non-monotone sequences.
Definition 2.6.1. A sequence (an )∞
n=1 is Cauchy if for all  > 0 there exists
n0 such that for all m, n ≥ n0 ,

|am − an | < .

Theorem 2.6.2 (Cauchy Convergence Criterion). Let (an )∞ n=1 be a sequence


of real numbers. Then it converges if and only if it is Cauchy.
Proof. ⇒: Assume that (an )∞ n=1 converges to some value, L ∈ R. We will
show that the definition of a Cauchy sequence holds; so let  > 0 be given.
By the definition of convergence with /2 in place of , there exists n0 such
that for all n ≥ n0 ,

|an − L| < .
2
Now, if m, n ≥ n0 then

|am − an | ≤ |am − L| + |L − an | (∆-inequality)


 
< + = .
2 2

26
as required.
⇐: Assume that (an )∞ ∞
n=1 is Cauchy. First, we must establish that (an )n=1
is bounded; this is Exercise 2.6.1. Now, we may apply the Bolzano–Weierstrass
Theorem (Corollary 2.5.6) to get a subsequence (ank )∞ k=1 which converges to
some value, L ∈ R. Let us show that (an )∞ n=1 converges to L.
Given  > 0, by the definition of a Cauchy sequence, pick n0 such that
for all m, n ≥ n0 ,

|am − an | < .
2
Next, since lim ank = L, we may also find k0 such that for all k ≥ k0 ,
k→∞


|ank − L| < .
2
Let us now fix k1 ≥ k0 such that nk ≥ n0 ; this is possible since the sequence
(nk )∞
k=1 is strictly increasing.
For n ≥ n0 , we now have

|an − L| ≤ |an − ank | + |ank − L| (∆-inequality)


 
< + = .
2 2
as required.
Exercise 2.6.1. Finish the proof of Theorem 2.6.2, by proving (without using
that theorem) that every Cauchy sequence is bounded.
Exercise 2.6.2. [TBB08, Exercise 2.12.1] Show directly that the sequence
1 ∞

n n=1
is Cauchy.
Exercise 2.6.3. (cf. [TBB08, Example 2.42]) Let (γn )∞ n=1 be a sequence of

numbers in (0, 1). Define a sequence (an )n=1 by a1 := 1, a2 := 2, and

an := γn an−2 + (1 − γn )an−1 , for n ≥ 3.

(a) Prove that


|an − an−1 | = γn γn−1 · · · γ3
for all n ≥ 3.

(b) Prove that for all m ≥ n,

|am − an | ≤ |an+1 − an |.

27
(c) Give an example of a sequence (γn )∞
n=1 for which the associated sequence

(an )n=1 does not converge.

(d) Prove that if


lim inf γn < 1
n→∞

then (an )∞
n=1 does converge.
Exercise 2.6.4. [TBB08, Exercise 2.12.5] Give an example of a sequence
(an )∞
n=1 such that for every  > 0, there exists n0 such that for all n ≥ n0 ,

|an+1 − an | < .

[Hint. Suppose that (bn )∞ n=1 is another sequence and an is defined by an :=


b1 + · · · + bn . What does this condition tell us about the bn ? ]

2.7 lim sup and lim inf


Definition 2.7.1. Let (an )∞
n=1 be a sequence of real numbers. The limit

superior of (an )n=1 is

lim sup an := inf{β ∈ R : ∃n0 such that an ≤ β ∀n ≥ n0 }.


n→∞

The limit inferior of (an )∞


n=1 is

lim inf an := sup{β ∈ R : ∃n0 such that an ≥ β ∀n ≥ n0 }.


n→∞

In other words, the limit superior is the infimum of the set of eventual
upper bounds for (an )∞
n=1 , and similarly for the limit inferior. Using the nota-
tional conventions we introduced around inf and sup, we write the following:
• If (an )∞
n=1 is not bounded above then (since we have no eventual upper
bounds), lim sup an = ∞.
n→∞

• If (an )∞
n=1 is not bounded below then (since we have no eventual lower
bounds), lim inf an = +∞.
n→∞

• If (an )∞
n=1 converges to −∞ then (since every number, no matter how
negative, will be an eventual upper bound), lim sup an = −∞.
n→∞

28
• If (an )∞
n=1 converges to ∞ then (since every number, no matter how
large, will be an eventual lower bound), lim inf an = ∞.
n→∞
1
Example 2.7.2. Define an := n
+ (−1)n . Let’s first compute lim sup an .
n→∞
First, we see that any number β > 1 will be an eventual upper bound: as
1
long as n ≥ n0 := β−1 , we have that an ≤ β. On the other hand, 1 is not an
eventual lower bound (there is no n0 such that an ≤ 1 for all n ≥ n0 ), since
there are arbitrarily large n such that an > 1. Thus,
{β ∈ R : ∃n0 such that an ≤ β ∀n ≥ n0 } = (1, ∞)
and so
lim sup an = inf(1, ∞) = 1.
n→∞
Now we compute lim inf an . Here, −1 is clearly a lower bound, and so
n→∞
any number ≤ −1 is also a lower bound (whence an eventual lower bound).
However, for any β > −1, β is not an eventual lower bound: for any n0 , we
1
may find n ≥ n0 such that n is odd and n ≥ β+1 , and so an < β. Thus,

{β ∈ R : ∃n0 such that an ≥ β ∀n ≥ n0 } = (−∞, −1]


and so
lim inf an = sup(−∞, −1] = −1.
n→∞

Proposition 2.7.3. For any sequence (an )∞


n=1 ,

lim inf an ≤ lim sup an .


n→∞ n→∞

Proof. If the sequence isn’t bounded then either lim sup an = ∞ or lim inf an =
n→∞ n→∞
−∞, and in either case, the result is trivial. So let us assume that (an )∞
n=1 is
bounded.
Consider the sets used to define lim sup and lim inf:
S := {β ∈ R : ∃n0 such that an ≤ β ∀n ≥ n0 },
T := {α ∈ R : ∃m0 such that an ≥ α ∀n ≥ m0 }.
If β ∈ S and α ∈ T then there is some n0 as in the definition of S and an
m0 as in the definition of T . Then with n := max{m0 , n0 }, we get
α ≤ an ≤ β.

29
Thus we have shown that every α ∈ T is below every β ∈ S (abusing notation,
we could write T ≤ S).
Rewording this, we have that every α ∈ T is a lower bound for S, and
thus for all α ∈ T ,
α ≤ inf S = lim sup an .
n→∞

But then this shows that lim sup an is an upper bound for T , so that
n→∞

lim sup an ≥ sup T = lim inf an ,


n→∞ n→∞

as required.
We now give another formula for lim sup and lim inf, which justifies the
notation.

Proposition 2.7.4. Let (an )∞


n=1 be a bounded sequence of real numbers. Then

lim sup an = lim sup{an , an+1 , an+2 , . . . } and (2.2)


n→∞ n→∞

lim inf an = lim inf{an , an+1 , an+2 , . . . }.


n→∞ n→∞

Proof. We will prove the formula for lim sup; the other one can be proven by
essentially the same argument.
For notational convenience, let us set

bn := sup{an , an+1 , an+2 , . . . }.

Note that since {an , an+1 , an+2 , . . . } ⊆ {an+1 , an+2 , . . . }, it follows that

bn ≥ bn+1 for each n.

Thus the sequence (bn )∞ ∞


n=1 is decreasing. It is also bounded, since (an )n=1 is,
so by the Monotone Convergence Criterion, it converges.
Set L := lim bn . We will prove lim sup an = L by two-way inequality.
n→∞ n→∞
≤: For each k, bk is an eventual upper bound: setting n0 := k we have
that bk ≤ an for all n ≥ n0 . Thus, by the definition of lim sup, we have
lim sup an ≤ bk . Consequently by Corollary 2.3.4 (with m := lim sup an ),
n→∞ n→∞

lim sup an ≤ lim bk = L.


n→∞ k→∞

30
≥: Let β be an eventual upper bound for (an )n→∞ , so that there exists
n0 such that an ≤ β for all n ≥ n0 . Then for all n ≥ n0 , β is an upper bound
for {an , an+1 , an+2 , . . . }. Hence,

β ≥ sup{an , an+1 , . . . } = bn .

Since this holds for all n ≥ n0 , it follows from Corollary 2.3.4 (with M = β)
that β is greater than or equal to the limit of (bn0 , bn0 +1 , . . . ), which is L (by
Proposition 2.5.4):
β ≥ L.
Thus, L is a lower bound for the set {β ∈ R : ∃n0 such that an ≤ β ∀n ≥ n0 }
used in the definition of lim sup, so

L ≤ inf{β ∈ R : ∃n0 such that an ≤ β ∀n ≥ n0 } = lim sup an .


n→∞

Remark 2.7.5. In the above proof, if we assume only that (an )∞


n=1 is bounded
above, then the bn are still real numbers, and the same arguments still work
(although lim bn = −∞ = lim sup an is possible), and so (2.2) still holds.
n→∞ n→∞
In the case that (an )∞
n=1 is not bounded above, this means in the notation of
the above proof, bn = ∞ for all n. However, if we agree that lim ∞ = ∞,
n→∞
then under this convention, again holds.
In conclusion, adopting the right conventions, the hypothesis that (an )∞
n=1
is bounded is not needed in the above proposition.

Theorem 2.7.6. Let (an )∞ n=1 be a sequence of real numbers. Then (an )n=1

converges if and only if lim sup an = lim inf an and this value is finite. In this
n→∞ n→∞
case,
lim an = lim sup an = lim inf an .
n→∞ n→∞ n→∞

Proof. First, suppose that the sequence converges, and set L := lim an .
n→∞
Given  > 0 there exists n0 such that for all n ≥ n0 ,

L −  < an < L + .

Hence,
L −  ≤ inf{an0 , an0 +1 , . . . }

31
and so from the formula in Proposition 2.7.4,

L −  ≤ lim inf an .
n→∞

Since  is arbitrary, we find L ≤ lim inf an . Likewise we may infer that


n→∞
L ≥ lim sup an . Hence,
n→∞

L ≤ lim inf an
n→∞
≤ lim sup an (Proposition 2.7.3)
n→∞
≤ L.

We conclude that

lim an = lim sup an = lim inf an .


n→∞ n→∞ n→∞

This shows one direction of the “if and only if”, as well as the final statement
of the theorem.
Conversely, suppose that

lim sup an = lim inf an ,


n→∞ n→∞

and that this value is finite. Set L := lim sup an = lim inf an .
n→∞ n→∞
Let  > 0 be given. We may find n0 such that

sup{an0 , an0 +1 , . . . } ∈ (L − , L + )

and
inf{an0 , an0 +1 , . . . } ∈ (L − , L + ).
It follows from the first of these that for all n ≥ n0 ,

L +  > sup{an0 , an0 +1 , . . . } ≥ an ,

and likewise, L −  < an . Thus, for n ≥ n0 , we have |an − L| < , as


required.
Exercise 2.7.1. [TBB08, Exercise 2.13.2] Compute lim sup and lim inf for
each of the following sequences.

32
(a) ((−1)n n)∞
n=1 ,

(b) (sin(nπ/8))∞
n=1 ,

(c) (n sin(nπ/8))∞ n=1 ,


 ∞
(d) (n+1) sin(nπ/8)
n
,
n=1

(e) (1 + (−1)n )∞
n=1 ,

(f) (rn )∞
n=1 , consisting of all the rational numbers in the interval (0, 1), ar-
ranged in some order. (Does the order matter?)

Exercise 2.7.2. [TBB08, Exercise 2.13.3] Find a sequence (an )∞


n=1 of rational
numbers such that:
√ √
(a) lim sup an = 2 and lim inf an = − 2.
n→∞ n→∞

(b) lim sup an = ∞ and lim inf an = − 2.
n→∞ n→∞

(c) lim sup an = π and lim inf an = e.


n→∞ n→∞

Exercise 2.7.3. [TBB08, Exercise 2.13.7] Let (an )∞ n=1 be a bounded sequence,
set L := lim sup an , and let  > 0. Prove the following.
n→∞

(a) an > L +  for only finitely many n ∈ N≥1 , and

(b) an > L −  for infinitely many n ∈ N≥1 .

Exercise 2.7.4. [TBB08, Exercise 2.13.9] Let (an )∞ ∞


n=1 and (bn )n=1 be two
bounded sequences.

(a) Prove that lim sup(an + bn ) ≤ lim sup an + lim sup bn .


n→∞ n→∞ n→∞

(b) Give an example where

lim sup(an + bn ) 6= lim sup an + lim sup bn .


n→∞ n→∞ n→∞

33
Exercise 2.7.5. Suppose that (an )∞
n=1 is a Cauchy sequence. Prove directly
(without using the Cauchy Convergence Criterion, though you are allowed
to use that (an )∞
n=1 is bounded) that lim sup an = lim inf an . If we then use
n→∞ n→∞
Theorem 2.7.6, we thus get an alternative proof of the Cauchy Convergence
Criterion (Theorem 2.6.2).
Exercise 2.7.6. Let (an )∞
n=1 be a bounded sequence. Prove that there is a
subsequence (ank )∞
k=1 which converges to lim sup an .
n→∞

34
Chapter 3

Series

This chapter is about infinite sums, also called series. Finite sums make
sense in any field; however, to sum infinitely many elements requires taking
a limit, so depends on the theory of sequences developed in the last chapter.

3.1 Definition and basic properties


A series (of real numbers) is an infinite sum

X
an = a1 + a2 + a3 + · · ·
n=1

where (an )∞
n=1 is a sequence of real numbers.
Since addition in R is (a priori) only defined for finitely many numbers
at a time1 we need to define what an infinite sum means for us.

Definition 3.1.1. Let (an )∞


n=1 be a sequence of real numbers. For N ∈ N≥1 ,
define
XN
sN := an = a1 + · · · + aN ,
n=1

1
In fact, recall that we only defined addition for two numbers at a time. But by iterating
N
P
we can of course do finitely many numbers: an = a1 + a2 + · · · + an formally means
n=1
(· · · ((a1 + a2 ) + a3 ) + · · · + an−1 ) + an .

35
∞ ∞
called the N th partial sum of the series
P P
an . We say that the series an
n=1 n=1
converges (to L) if the sequence (sN )∞
N =1 converges (to L), and in this case,
we may also write
X∞
an = L.
n=1

If a series does not converge, we say it diverges. If (sN )∞


n=1 diverges to ±∞,
we write ∞
X
an = ±∞.
n=1

In other words,

X N
X
an := lim an ,
N →∞
n=1 n=1

provided that the limit on the right exists.


We use theory from the previous topic (sequences) to establish results
about series.
Proposition 3.1.2. Let (an )∞ ∞
n=1 and (bn )n=1 be sequences and let c ∈ R.
Suppose that the series

X ∞
X
an and bn
n=1 n=1

both converge.

P
(i) (an + bn ) converges, and
n=1


X ∞
X ∞
X
(an + bn ) = an + bn
n=1 n=1 n=1


P
(ii) can converges, and
n=1


X ∞
X
can = c an .
n=1 n=1

Proof. Exercise 3.1.1.

36
The above says that the set

X
V := {(an )∞
n=1 : an converges}
n=1


is a vector space and the function V → R defined by (an )∞
P
n=1 7→ an is a
n=1
linear map.
Proposition 3.1.3. Let (an )∞ ∞
n=1 and (bn )n=1 be sequences such that an ≤ bn
for all n. If the series
X ∞ X∞
an and bn
n=1 n=1

both converge, then



X ∞
X
an ≤ bn .
n=1 n=1

Proof. Exercise 3.1.2.


Often, as in the following proposition, we start an infinite sum at a dif-
ferent index than 1. This is no problem: we can rewrite such a series as one
starting at 1, for example as

X ∞
X
an = bn
n=m n=1

where bn = am+n−1 . (This is needed to make sense of such an infinite sum


from a theoretical perspective, but in practice, when it is clear what is going
on, we don’t need to do this.)
Proposition 3.1.4. Let (an )∞
n=1 be a sequence of real numbers and let m ∈
N≥1 . Then
X∞
an
n=1

converges if and only if



X
an
n=m
converges.

37
Proof. Exercise 3.1.3.
Example 3.1.5. Consider the series

X 1
.
n=1
n(n + 1)

Note that
1 1 1
= − .
n(n + 1) n n+1
Therefore, we may simplify the partial sum
     
1 1 1 1 1 1
sN = − + − + ··· + −
1 2 2 3 N N +1
1
=1− ,
N +1
by cancelling terms in the middle. Therefore,
∞  
X 1 1
= lim 1 − = 1.
n=1
n(n + 1) N →∞ N +1

A series in which the partial sums simplify in this way is called a telescoping
series.
Typically, series are not telescoping, so we want more systematic tech-
niques for determining whether a series converges.
Example 3.1.6. The harmonic series is

X 1
.
n=1
n

Consider the partial sum


1 1
s2k = 1 + + ··· + k
2 2
each term ≥1/4 each term ≥1/8 each term ≥1/2k
z }| { z }| { z }| {
1 1 1 1 1 1 1
=1+ + + + + · · · + + · · · + k−1 + ··· + k
2 3 4 5 8 2 +1 2

38
We have 2 terms which are ≥ 14 , so their sum is at least 12 . Following this, we
have 4 terms which are ≥ 18 , so their sum is at least 21 . Continuing, we can
see that each grouping sums to at least 12 . There are k − 1 such groupings,
so we have
1 1 k
s2k ≥ 1 + + (k − 1) = 1 + .
2 2 2

It follows that (sN )N =1 is not bounded above, so it diverges. In fact, the
sequence (sN )∞
N =1 is increasing, so we see that sN → ∞ and thus


X 1
= ∞.
n=1
n

The previous example is very important, since n1 → 0. Intuitively, one



P
might have expected that a series an converges if and only if its terms
n=1
converge to 0; while the harmonic series shows that one direction of this is
not true, the intuition is correct in the other direction, as the next result
shows.

Proposition 3.1.7. Let (an )∞


n=1 be a sequence of real numbers. If the series


X
an
n=1

converges, then
lim an = 0.
n→∞


P
Proof. Set L := an . Consider the partial sum
n=1

N
X
sN = an ,
n=1

so that lim sN = L. But then we also have lim sN −1 = L, so


N →∞ N →∞

0 = L − L = lim (sN − sN −1 ) = lim aN .


N →∞ N →∞

39
It was mentioned before the above proposition, but bears repeating: the
converse is not true. If a sequence (an )∞
n=1 converges to 0, it does not follow

P
that the series an converges.
n=1
By taking the contrapositive of Proposition 3.1.7, we have a useful test
for divergence of a series:
The Divergence Test. If a sequence (an )∞
n=1 does not converge to 0, then the
series ∞
X
an
n=1
does not converge.
Example 3.1.8. A geometric series is one of the form
1 + r + r2 + r3 + · · ·
where r ∈ R. When |r| ≥ 1, the sequence (rn )∞ n=0 does not converge to 0,
so by the Divergence Test, it follows that the corresponding geometric series
diverges.
On the other hand, for |r| < 1, the sequence (rn )∞
n=0 does converge to 0,
so the corresponding geometric series at least has a chance to converge. We
rewrite the partial sum
1 − rN +1
sN = 1 + r + r 2 + · · · + r N = .
1−r
From this we see that the series does converge,

X 1 − rN 1
rn = lim = .
N →∞ 1 − r 1−r
n=0

Exercise 3.1.1. Prove Proposition 3.1.2.


Exercise 3.1.2. Prove Proposition 3.1.3.
Exercise 3.1.3. Prove Proposition 3.1.4. When they converge, what is the
P∞ P∞
relationship between an and an ?
n=1 n=m

P
Exercise 3.1.4. [TBB08, Exercise 3.4.3] If (an + bn ) converges, what can
n=1
you say about the series

X ∞
X
an and bn ?
n=1 n=1

40

P
Exercise 3.1.5. [TBB08, Exercise 3.4.4] If (an + bn ) diverges, what can
n=1
you say about the series

X ∞
X
an and bn ?
n=1 n=1

Exercise 3.1.6. [Leb16, Exercise 2.5.4] Let (an )∞


n=1 be a sequence of real num-
bers.
P∞ P∞
(a) Prove that if an converges, then so does (a2n−1 + a2n ).
n=1 n=1

(b) Give an example where the converse of (a) doesn’t hold, that is, such

P ∞
P
that (a2n−1 + a2n ) converges but an does not.
n=1 n=1


P
This exercise shows that we must be cautious when we write a series an
n=1
as
a1 + a2 + · · · .

3.2 Convergence tests


Here we will establish a number of results which are useful for proving con-
vergence of series in various different settings.

Proposition 3.2.1 (Boundedness Test). Let (an )∞


n=1 be a sequence of real
numbers. Suppose that:

(i) an ≥ 0 for all n, and

(ii) There is a bound M ∈ R on the partial sums, so that


N
X
an ≤ M
n=1

for all N ∈ N≥1 .



P
Then an converges.
n=1

41
Proof. Since an ≥ 0, the partial sums (sN )∞
N =1 satisfy

sN ≤ sN +1 for all N.
In other words, (sN )∞
N =1 is an increasing sequence. The second condition
ensures that this sequence is bounded above, Therefore, by the Monotone

P
Convergence Criterion (Theorem 2.4.2), it converges. Hence, an con-
n=1
verges.
Proposition 3.2.2 (Comparison Test). Let (an )∞ ∞
n=1 and (bn )n=1 be sequences
such that:
0 ≤ an ≤ bn for all n.
Then:

P ∞
P
(i) if bn converges, then so does an .
n=1 n=1

P ∞
P
(ii) if an diverges, then so does bn .
n=1 n=1

Warning: it is easy to get confused about the hypothesis of the Compar-


P∞
ison Test. If 0 ≤ an ≤ bn , and an converges, then we cannot conclude
n=1

P ∞
P
anything about bn . Likewise, if bn diverges, we cannot conclude any-
n=1 n=1

P
thing about an .
n=1

Proof. (ii) is the contrapositive of (i). We will prove (i) now, using the
Boundedness Test. Hypothesis (i) of the Boundedness Test is true since it is
a hypothesis here.

P
Set M := bn . Since the sequence
n=1

N
!∞
X
bn
n=1 n=1

is increasing and converges to M , we have that M is the supremum of this


sequence, and in particular,
XN
bn ≤ M
n=1

42
for all N . Therefore,
N
X N
X
an ≤ bn ≤ M.
n=1 n=1

This verifies hypothesis (ii) of the Boundedness Test, so by that test we



P
conclude that an converges.
n=1

Proposition 3.2.3 (Absolute Convergence Test). Let (an )∞


n=1 be a sequence
of real numbers. If the series

X
|an |
n=1

converges, then so does the series



X
an .
n=1

Proof. Write

(an )+ := max{an , 0} and


(an )− := max{−an , 0}.

Then 0 ≤ (an )+ ≤ |an |, so by the Comparison Test (i) (with (an )+ in place

P
of an and |an | in place of bn ), (an )+ converges. By the same argument,
n=1

P
the series (an )− converges. Finally, we observe that an = (an )+ − (an )− ,
n=1
so by linearity,

X ∞
X ∞
X
an = (an )+ − (an )−
n=1 n=1 n=1

converges.

P
Interesting fact 3.2.4. One calls a series an absolutely convergent when the
n=1

P
series |an | converges. It is nontrivial result (by Dirichlet and Riemann)
n=1

43

P
that a series an is absolutely convergent if and only if any rearrangement
n=1
of it converges to the same value, i.e.,

X ∞
X
as(n) = an
n=1 n=1

for any bijection s : N≥1 → N≥1 .


As an illustration of this, consider the alternating harmonic series
1 1
1− + − ··· .
2 3
This series converges to some value L ≥ 12 (by the Alternating Series Test,
Proposition 3.2.9 below; the estimate 21 comes by looking at the second partial
sum). If rearrangements were allowed, then we could do the following
1 1 1 1 1
L=1− + − + − + ···
2 3 4 5 6
1 1 1 1 1 1 1 1
= 1 − − + − − + ··· + − − + ···
 2 4 3 6 7  2k + 1 4k + 2 4k + 3
1 1 1 1 1
= 1− − + − − + ···
2 4 3 6 8
1 1 1 1
= − + − + ···
2 4 6 8 
1 1 1
= 1 − + − ···
2 2 3
L
= .
2
Since L 6= 0, this is a contradiction. This shows that rearranging the al-
ternating harmonic series can change its value, confirming a special case of
the Dirichlet–Riemann result, since we already know that the alternating
harmonic series does not converge absolutely, by Example 3.1.6.
Proposition 3.2.5 (Ratio Test). Let (an )∞
n=1 be a sequence of nonzero real
numbers.
(i) If
an+1
lim sup
<1
n→∞ a n

44

P
then an converges (absolutely).
n=1

(ii) If
an+1
lim inf
>1
n→∞ an

P
then an diverges.
n=1

Proof. (i): Suppose


an+1
q := lim sup < 1.
n→∞ an

P
We wish to show that |an | converges, so we will replace an with |an | and
n=1

P
show that an converges. This allows us to assume that an ≥ 0 for all n.
n=1
Pick r ∈ (q, 1). Then by the definition of lim sup, r is an eventual upper
bound for an+1
an
. That is, there exists n0 such that
an+1
≤r
an
for all n ≥ n0 , which we rearrange as

an+1 ≤ an r.

Thus we have,

an0 +1 ≤ an0 r,
an0 +2 ≤ an0 +1 r ≤ an0 r2 ,

and continuing in this way, an0 +k ≤ an0 rk for all k ∈ N≥0 . Since r ∈ (0, 1), the

rk converges (Example 3.1.8). Hence by the comparison
P
geometric series
k=0

P
test, an0 +k converges, which is the same as saying
k=0


X
an converges.
n=n0

45

P
By Proposition 3.1.4, it follows that an converges.
n=1
(ii): Suppose
an+1
q := lim inf > 1.
n→∞ an
Then by the definition of lim inf, since 1 < q, 1 is an eventual lower bound
for |a|an+1
n|
|
, so there exists n0 such that

|an+1 |
≥1
|an |
for all n ≥ n0 , which we rearrange as

|an+1 | ≥ |an |.

Similarly to what we did in part (i), from this we get |an+k | ≥ |an0 | for
all k ∈ N≥0 . We conclude that (an )∞n=1 doesn’t converge to 0, so by the

P
Divergence Test, an diverges.
n=1

Remark 3.2.6. In many examples of series you will see, the limit lim an+1

n→∞ an

will exist (so will be equal to the lim inf and the lim sup). However, it often
happens that
an+1
lim = 1,
n→∞ an

and in this case, we cannot conclude anything from the Ratio Test. To see
why, recall the two series from Examples 3.1.5 and 3.1.6:

X 1
= 1,
n=1
n(n + 1)

X 1
= ∞.
n=1
n

However, in both cases, the ratio an+1


an
converges to 1.
For an example where lim sup an > 1 > lim inf an+1
an+1
an
, define
n→∞ n→∞
(
1, n even;
an :=
2, n odd.

46
Then (
an+1 2, n even;
= 1
an 2
, n odd.

P
Of course in this case, an diverges (by the Divergence Test). See also
n=1
Exercise 3.2.8.
Proposition 3.2.7 (Root Test). Let (an )∞
n=1 be a sequence of real numbers.

(i) If p
n
lim sup |an | < 1
n→∞

P
then an converges (absolutely).
n=1

(ii) If p
n
lim sup |an | > 1
n→∞

P
then an diverges.
n=1

Proof. This proof is similar to the proof of the Ratio Test.


(i): Suppose p
q := lim sup n |an | < 1.
n→∞

P
We wish to show that |an | converges, so we replace an with |an | and we’ll
n=1

P
show that an converges.
n=1
Pick r ∈ (q, 1), and using the definition of lim sup, there exists n0 such
that
√n
an ≤ r for all n ≥ n0 .
In other words,
0 ≤ an ≤ r n

rn converges, we conclude by
P
for all n ≥ n0 . Since the geometric series
n=n0

P ∞
P
the Comparison Test that so does an . Thus by Proposition 3.1.4, an
n=n0 n=1
converges.

47
(ii) Suppose p
n
q := lim sup |an | > 1.
n→∞

Using the definition of lim sup, there are infinitely many n such that
p
n
|an | ≥ 1.

For such n, we have


|an | ≥ 1.
We conclude that (an )∞
n=1 doesn’t converge to 0, so by the Divergence Test,

P
an diverges.
n=1
p
Remark 3.2.8. Although n |an | will often converge in examples you will see,
it might converge to 1, in which case the Root Test tells us nothing. Again
this can be see through the examples

X 1
= 1,
n=1
n(n + 1)

X 1
= ∞.
n=1
n

However, in both cases, the n an converges to 1.
Our next test concerns alternating series, which is a series in which the
signs of the terms alternate between positive and negative. We set up an al-
ternating series by taking a sequence (an )∞
n=1 of positive numbers and forming

(−1)n+1 an .
P
n=1

Proposition 3.2.9 (Alternating Series Test). Let (an )∞


n=1 be a sequence of
real numbers. Suppose that:
(i) (an )∞
n=1 is an decreasing sequence, and

(ii) lim an = 0.
n→∞

Then ∞
X
(−1)n+1 an
n=1

48
converges. Moreover, for any N ,
2N
X ∞
X 2N
X −1
(−1)n+1 an ≤ (−1)n+1 an ≤ (−1)n+1 an .
n=1 n=1 n=1

Proof. Note that an ≥ 0 for all n, because the sequence is decreasing and
converges to 0.

Set
N
X
sN := (−1)n+1 an .
n=1

Note that an ≥ 0 for all n, because Since (an )∞


n=1 is a decreasing sequence,
we have
sN +2 = sN + aN +1 − aN2 ≥ sN if N is even.
sN +2 = sN − aN +1 + aN2 ≥ sN if N is odd.
Thus, the subsequence of even terms, (s2N )∞ N =1 is an increasing sequence,

and the subsequence of odd terms, (s2N −1 )N =1 is decreasing. To show that
the sequence of even terms (s2N )∞
N =1 is bounded, observe that for each N ,
since a2N +1 ≥ 0,
s2N ≤ s2N + a2N +1 = s2N +1 ≤ s1 .
So, s1 is an upper bound for (s2N )∞N =1 . By the Monotone Convergence Cri-
terion (Theorem 2.4.2), (s2N )∞
N =1 converges to L := sup{s2N : N ∈ N≥1 }.
Next, for the subsequence of odd terms, since s2N = s2N −1 − a2N , we get
lim s2N −1 = lim s2N + a2N = lim s2N + lim a2N = L + 0.
N →∞ N →∞ N →∞ N →∞

By Exercise 2.2.4, since both the even and odd subsequences converge to the
same value, we conclude that
lim sN = L,
N →∞

49

(−1)n+1 an converges and equals L.
P
i.e.,
n=1
Finally, since L is the supremum of {s2N : N ∈ N≥1 }, we have

X 2N
X
(−1)n+1 an = L ≥ s2N = (−1)n+1 an .
n=1 n=1

By a similar argument as above, we can show that L = inf{s2N −1 : N ∈ N≥1 },


and from this,
X∞ 2N
X −1
(−1)n+1 an ≤ (−1)n+1 an .
n=1 n=1

In the Alternating Series Test, it is crucial that the sequence (an )∞ n=1 is
decreasing – see Exercise 3.2.3.
The last test that we will state uses the integral. Since we haven’t formally
defined the integral, we will not be able to prove this result yet; we will prove
it later (Exercise 7.4.4 when we study integration. However, it is a very
powerful test, particularly as one can often use the Fundamental Theorem of
Calculus to easily analyse convergence of an integral.
Proposition 3.2.10 (Integral Test). Let f : [1, ∞) → R be a function.
Suppose that:
(i) f (x) ≥ 0 for all x ∈ [1, ∞), and

(ii) f is decreasing: f (x) ≥ f (y) whenever x ≤ y.


Then the series ∞
X
f (n)
n=1

converges if and only if the improper integral


Z ∞
f (x) dx
1

converges.
Exercise 3.2.1. For each of the following, determine whether the series con-
verges.

50

n
P
(a) 2n
.
n=1


n2 −3
P
(b) (n+2)(n+5)
.
n=1


P (−1)n n2 −3
(c) (n+2)(n+5)
.
n=1


P (n2 +3)n
(d) (2n2 −1)n
.
n=1


P sin(n)
(e) n2 +n
.
n=1


n2 +2
P
(f) n4 +4
.
n=1


n3 +3
P
(g) n6 +6
.
n=1


n!
P
(h) 2n
.
n=1

Exercise 3.2.2. [Leb16, Exercise 2.5.7] Suppose that (an )∞


n=1 is a decreasing

P
sequence. If the series an converges, prove that lim nan = 0.
n=1 n→∞

Exercise 3.2.3. Give an example of a sequence (an )∞n=1 such that an ≥ 0 for
all n and lim an = 0, such that the alternating series
n→∞


X
(−1)n+1 an
n=1

does not converge.


Exercise 3.2.4. [TBB08, Exercise 3.5.1] Suppose that (an )∞ n=1 is a sequence

P
of real numbers such that an ≥ 0 for all n. If we know that an converges,
n=1

a2n
P
does it follow that converges?
n=1

51
Exercise 3.2.5. [TBB08, Exercise 3.4.7] Suppose that (an )∞ ∞
n=1 and (bn )n=1

P ∞
P
are sequences of real numbers such that an and bn converge. Does it
n=1 n=1

P
follow that an bn converges? What about if we assume in addition that
n=1
an ≥ 0 for all n?
Exercise 3.2.6. Suppose that (an )∞ ∞
n=1 and (bn )n=1 are sequences of real num-
bers such that:

(i) bn > 0 for all n,



P
(ii) bn converges, and
n=1
 ∞
an
(iii) The sequence bn
converges.
n=1

P
Prove that an converges.
n=1

P
Exercise 3.2.7. [TBB08, Exercise 3.5.11] Show that a series an is abso-
n=1

P
lutely convergent if and only if every “subseries” ank converges.
k=1

Exercise 3.2.8. Produce an example of a sequence (an )∞


n=1 of numbers in
(0, ∞) such that
an+1 an+1
lim sup > 1, lim inf < 1,
n→∞ an n→∞ an

an converges. Can you even arrange that lim sup an+1
P
and the series an
= ∞?
n=1 n→∞

3.3 Cauchy convergence criterion for series


The following is an important theoretical device, although for examples of
sequences you may encounter, it is rarely the right tool to prove convergence.

Proposition 3.3.1 (Cauchy Convergence Criterion). Let (an )∞n=1 be a se-



P
quence of real numbers. Then converges if and only if, for every  > 0
n=1

52
there exists N0 such that, for all N ≥ M ≥ N0 ,

XN
an < .



n=M

Proof. Again we use the partial sums


N
X
sN := an .
n=1

By the Cauchy Convergence Criterion for sequences (Theorem 2.6.2), (sN )∞ N =1



an converges) if and only if (sN )∞
P
converges (i.e., N =1 is Cauchy. Since
n=1

N
X
sN − sM −1 = an ,
n=M

it is easy to see that (sN )∞


N =1 is Cauchy if and only if, for every  > 0 there
exists N0 such that for all N ≥ M ≥ N0 ,

XN
an < .



n=M

Exercise 3.3.1. Let (an )∞ ∞ ∞


n=1 , (bn )n=1 , and (cn )n=1 are sequences such that for
all n,
an ≤ b n ≤ c n .

P ∞
P P∞
Prove that if an and cn converge, then so does bn . Can this be
n=1 n=1 n=1
proven using the Squeeze Theorem (Theorem 2.3.5)?

53
Chapter 4

Topology of Rd

In this chapter we will look at Euclidean space Rd , enabling us to consider


sequences of points in this space (and, correspondingly, limits of such se-
quences).

4.1 Norms
The set Rd is a vector space over R: it is equipped with addition,

(a1 , . . . , ad ) + (b1 , . . . , bd ) := (a1 + b1 , . . . , an + bn ),

and scalar multiplication,

c · (a1 , . . . , ad ) := (ca1 , . . . , cad )

(here, (a1 , . . . , ad ), (b1 , . . . , bd ) ∈ Rd and c ∈ R).


A norm encapsulates a way of measuring distances in vector spaces.

Definition 4.1.1. A norm on Rd is a function k · k : Rd → [0, ∞) (written


a 7→ kak) satisfying the following, for a, b ∈ Rd and c ∈ R:

(i) kak = 0 if and only if a = (0, . . . , 0); (positive definiteness)

(ii) kcak = |c| · kak; (homogeneity)

(iii) ka + bk ≤ kak + kbk. (triangle inequality)

54
For us, the most important norm is the Euclidean norm,
q
k(a1 , . . . , ad )k2 := a21 + · · · + a2d .

Proving properties (i) and (ii) of a norm for k·k2 is a straightforward exercise
(Exercise 4.1.1). The third property takes a bit more work. For this, it is
useful to observe that the norm is given by

kak2 = a · a,

where we employ the dot product:

(a1 , . . . , ad ) · (b1 , . . . , bd ) := a1 b1 + · · · + ad bd .

Proposition 4.1.2. Let a, b ∈ Rd and let k · k denote the Euclidean norm.

(i) (Cauchy–Schwarz Inequality) |a · b| ≤ kak2 · kbk2 .

(ii) (Triangle Inequality) ka + bk2 ≤ kak2 + kbk2 .

(iii) Therefore, k · k2 is a norm on Rd .

Proof. (i): This is clearly true if b = 0, so assume otherwise. Consider

p(t) := ka + tbk22
= (a + tb) · (a + tb)
= a · a + 2t(a · b) + t2 (b · b)
kak22 + 2(a · b)t + kbk22 t2

This is a quadratic polynomial which is always ≥ 0 (evident from the first


line), and therefore has at most one root. Hence its discriminant must be
≤ 0, that is, 2
0 ≥ 2(a · b) − 4kak22 kbk22 .
Rearranging this yields
|a · b|2 ≤ kak22 kbk22 ,
and then taking square roots yields the desired inequality.

55
(ii): Using the same expansion (with t = 1) we have

ka + bk22 = kak22 + 2a · b + kbk22


≤ kak22 + 2|a · b| + kbk22
≤ kak22 + 2kak2 · kbk2 + kbk22 t2 (Cauchy–Schwarz)
= (kak2 + kbk2 )2 ,

and then taking square roots yields the desired inequality.


(iii): We just proved the triangle inequality, while Exercise 4.1.1 is to
show conditions (i) and (ii).
We will mostly use the Euclidean norm when working with Rd , and will
often write k · k for k · k2 ; however, the next two most important norms are
the l1 and l∞ norms, defined as follows.
Example 4.1.3. The l1 -norm on Rd is given by

k(a1 , . . . , ad )k1 := |a1 | + · · · + |ad |.

The l∞ -norm on Rd is given by

k(a1 , . . . , ad )k∞ := max{|a1 |, . . . , |ad |}.

These are norms (Exercise 4.1.2).


Interesting fact 4.1.4. In fact, there are numerous other norms that one can
put on Rd ; however, in a sense that is very relevant to this course, they are
all equivalent. To be precise, for any two norms k · k and k · k0 on Rd , there
exist α, β > 0 such that, for all x ∈ Rd ,

αkxk ≤ kxk0 ≤ βkxk.

We won’t prove this in this course. However, when using two norms out of
k · k1 , k · k2 , and k · k∞ , we can find α and β explicitly: for all a ∈ Rd ,

kak∞ ≤ kak2 ≤ kak1 ≤ dkak∞

(Exercise 4.1.3).
Exercise 4.1.1. Show that the Euclidean norm k · k2 satisfies conditions (i)
and (ii) of a norm.
Exercise 4.1.2. Show that k · k1 and k · k∞ are norms.

56
Exercise 4.1.3. Prove that for all a ∈ Rd ,
kak∞ ≤ kak2 ≤ kak1 ≤ dkak∞ .
Exercise 4.1.4. [Sav17, Exercise 4.1.5] Let k · k and k · k0 be norms on Rd .
Define k · k00 : Rd → [0, ∞) by
kak00 := kak + kak0 .
Prove that k · k00 is a norm.

4.2 Convergence in Rd
We now make use of a norm on Rd to define convergence of a sequence in Rd .
Definition 4.2.1. Let (an )∞ d d
n=1 be a sequence in R (that is, an ∈ R for each
d ∞
n), and let L ∈ R . We say that (an )n=1 converges to L if lim kan −Lk2 = 0.
n→∞
In this case, we may also write
lim an = L or an → L as n → ∞.
n→∞

We have used a particular norm, k · k2 , in the above definition; however,


the next proposition says that we could have used any other norm and arrived
at the same concept.
Proposition 4.2.2. Let (an )∞ d d
n=1 be a sequence in R and let L ∈ R . Let
0 d
k · k be a norm on R . Then an → L if and only if
lim kan − Lk0 = 0.
n→∞

Proof. Exercise 4.2.1 (use Interesting Fact 4.1.4).


When working with sequences in Rd , we sometimes want to look at the
components of each term of the sequence. This gets notationally a bit messy,
and we just have to bear with it. If (an )∞ d
n=1 is a sequence in R , we write

an = (a(1) (d)
n , . . . , an ) for each n ∈ N≥1 .

Note that we then get d different sequences of real numbers, one for each
component: ∞ (d) ∞
a(1)

n n=1 , . . . , an n=1 .

Through these sequences, we can relate convergence in Rd to convergence in


R.

57
Proposition 4.2.3. Let (an )∞ d
n=1 be a sequence in R , with

an = (a(1) (d)
n , . . . , an ) for each n ∈ N≥1 ,

and let L = (L1 , . . . , Ld ) ∈ Rd . Then

lim an = L (in Rd )
n→∞

if and only if, for each i = 1, . . . , d,

lim a(i)
n = Li (in R).
n→∞

Proof. ⇒: We assume that lim an = L. Fix i ∈ {1, . . . , d}, and we must


n→∞
(i)
show that lim an = Li . Note that from the formula for k · k2 , for any
n→∞
d
x = (x1 , . . . , xd ) ∈ Rd , we have |xi |2 ≤ x2i ≤ kxk22 , and thus |xi | ≤ kxk2 .
P
i=1
Therefore,
−kan − Lk2 ≤ a(i)
n − Li ≤ kan − Lk2 .
(i)
By the Squeeze Theorem, it follows that lim an − Li = 0, which is the same
n→∞
(i)
as lim an = Li .
n→∞
(i)
⇐: We assume that lim an = Li for all i = 1, . . . , d. Then we have
n→∞

lim kan − Lk22 = lim (a(1) 2 (d)


n − L1 ) + · · · + (an − Ld )
2
n→∞ n→∞
= 0.

It follows from Exercise 2.2.5 that kan − Lk2 → 0, as required.


Remark 4.2.4. Since limits are unique in R, a consequence of the previous
proposition is that limits are also unique in Rd .

Definition 4.2.5. A sequence (an )∞ d


n=1 in R is Cauchy if for every  > 0
there exists n0 such that for all m, n ≥ n0 ,

kam − an k2 < .

Theorem 4.2.6 (Cauchy Convergence Criterion for Rd ). Let (an )∞


n=1 be a
d
sequence in R . Then it converges if and only if it is Cauchy.

58
Proof. ⇒: This proof is almost the same as the argument for sequences in R.
Assume that (an )∞ d
n=1 converges to some L ∈ R . Let  > 0 be given. There
exists n0 such that for all n ≥ n0 ,

kan − Lk2 < .
2
Now, if m, n ≥ n0 then

kam − an k2 ≤ kam − Lk2 + kL − an k2 (∆-inequality)


 
< + = .
2 2
Hence (an )∞
n=1 is Cauchy.
(1) (d)
⇐: Suppose that (an )∞ is Cauchy. Write an = (an , . . . , an ). Let us
n=1 

(i)
show that each sequence an is Cauchy (for i = 1, . . . , d). Fix i and let
n=1
 > 0 be given. Then since (an )∞
n=1is Cauchy, there exists n0 such that for
all m, n ≥ n0 , kam − an k2 < . Then for m, n ≥ n0 , we have

|a(i) (i)
m − an | ≤ kam − an k2 < ,
 ∞
(i)
as required. Thus an is Cauchy, and so it converges by the Cauchy
n=1
Convergence Criterion for R (Theorem 2.6.2); we let Li ∈ R be its limit.
Consequently by Proposition 4.2.3, (an )∞
n=1 converges to (L1 , . . . , Ld ).

Definition 4.2.7. A subset S of Rd is bounded if there exists some M > 0


such that for all x ∈ S,
kxk2 ≤ M.
A sequence (an )∞ d
n=1 in R is bounded if the set of its terms, {an : n ∈ N≥1 }
is bounded.

Note that in Rd , we do not have a concept of bounded above or below,


since there is no ordering.
Next, we generalize the Bolzano–Weierstrass Theorem to Rd . We define
a subsequence of a sequence in Rd in exactly the same way as for a sequence
in R.

Theorem 4.2.8 (Bolzano–Weierstrass Theorem for Rd ). Let (an )∞ n=1 be a


d
bounded sequence in R . Then it has a subsequence which converges.

59
Proof. We prove this by induction in d. The base case is d = 1, and in this
case it just the Bolzano–Weierstrass Theorem for R (Corollary 2.5.6).
We now assume that it holds for Rd−1 . Let (an )∞ n=1 be a bounded sequence
d (1) (d)
in R . For each n, write an = (an , . . . , an ) and define

bn := (a(1) (d−1)
n , . . . , an ) ∈ Rd−1 .

Since kbn k2 ≤ kan k2 , the sequence (bn )∞


n=1 is bounded. By the inductive hy-
pothesis, (bn )n=1 has a subsequence (bnk )∞

k=1 converging to some (L1 , . . . , Ld−1 ) ∈
  ∞
(d)
Rd−1 . Next, we consider the subsequence ank of the last-component
 ∞ k=1
(d)
sequence an . This subsequence is also bounded, so it has a subsubse-
 ∞n=1
(d)
quence, ankl which converges to some Ld ∈ R.
l=1
Now, (ankl )∞ ∞
l=1 is a subsequence of (an )n=1 . We are done once we prove
the following claim.
Claim. (ankl )∞l=1 converges to (L1 , . . . , Ld ).
To prove the claim, by Proposition 4.2.3, it is equivalent to show that
(i)
(ankl )∞
l=1 converges to Li for each i = 1, . . . , d.
For i = d, we already know this converges. For i < d, since (bnk )∞ k=1
(i)
converges to (L1 , . . . , Ld−1 ), we know from Proposition 4.2.3 that lim ank =
k→∞
(i)
Li . Then by Proposition 2.5.4, it follows that lim ankl = Li as well.
l→∞

Exercise 4.2.1. Prove Proposition 4.2.2 (using Interesting Fact 4.1.4).


Exercise 4.2.2. Let (an )∞ ∞ d
n=1 and (bn )n=1 be sequences in R that converge,
and let c ∈ R.
   
(a) Prove that lim (an + bn ) = lim an + lim bn .
n→∞ n→∞ n→∞
 
(b) Prove that lim can = c lim an .
n→∞ n→∞

(c) Prove that if (cn )∞


n=1 is a sequence in R which converges, then lim cn an =
   n→∞

lim cn lim an . (Here we are using scalar multiplication.)


n→∞ n→∞
   
(d) Prove that lim an · bn = lim an · lim bn . (Here we are using dot-
n→∞ n→∞ n→∞
product).

60
4.3 Open and closed sets
Definition 4.3.1. Let a ∈ Rd and let r > 0. The open ball centred at a with
radius r is
B(a; r) := {x ∈ Rd : kx − ak2 < r}.

Example 4.3.2. For d = 1,

B(a; r) = {x ∈ Rd : |x − a| < r} = (a − r, a + r).

In fact, every bounded open interval (a, b) is an open ball:


 
a+b b−a
(a, b) = B ; .
2 2

For d = 2, the open ball B(a; r) is an open disc (whose boundary is the circle
centred at a with radius r).

For d = 3, the open ball B(a; r) is interior of a sphere – the name “ball”
is most appropriate here.

Definition 4.3.3. Let A ⊆ Rd be a set.

(i) A is open if for every x ∈ A, there exists  > 0 such that

B(x; ) ⊆ A.

(ii) A is closed if its complement,

Rd \ A = {x ∈ Rd : x 6∈ A}

is open.

61
It is not the case that a set is either open or closed. For exam-
ple, the set (0, 1] is neither open (since 1 ∈ (0, 1] but there is no open ball
containing 1 that is in (0, 1]) nor closed (similar reasoning with 0 ∈ R\(0, 1]).
Example 4.3.4. Given a ∈ Rd and r > 0, the open ball B(a; r) is an open set.
To see this, let x ∈ B(a; r), so that kx − ak2 < r. Define

 := r − ka − xk2 > 0.

To show that B(x; ) ⊆ B(a; r), let y ∈ B(x; ), so that ky − xk2 < . Then
by the triangle inequality,

ky − ak2 ≤ ky − xk2 + kx − ak <  + ka − xk2 = r.

Thus, y ∈ B(a; r), as required.

Proposition 4.3.5. (i) The sets ∅ and Rd are open.

(ii) For any finite collection of open sets, U1 , . . . , Um ⊆ Rd , their intersec-


tion,
U1 ∩ · · · ∩ Um ,
is open.

(iii) For any arbitrary collection of open sets {Uα : α ∈ I}, their union,
[
Uα ,
α∈I

is open.

Proof. Parts (i) and (ii) are Exercise 4.3.1.


(iii): Set [
U := Uα ,
α∈I

and let x ∈ U . By definition of union, this means that x ∈ Uα for some


α ∈ I. Since Uα is open, there is some  > 0 such that B(x; ) ⊆ Uα . Since
Uα ⊆ U , it follows that
B(x; ) ⊆ U,
as required.

62
Example 4.3.6. Although we have that finite intersections of open sets are
open, it is not true that infinite intersections of open sets are open. For
example,
∞  
\ 1 1
{0} = − , ,
n=1
n n
but {0} isn’t an open set because it contains no open ball centred at 0.
Example 4.3.7. For any a ∈ R, the sets (a, ∞) and (−∞, a) are open. To see
this, we may write them as unions of open balls:

[ ∞
[
(a, ∞) = (a, a + n), (−∞, a) = (a − n, a).
n=1 n=1

Alternatively, here is a proof that (a, ∞) is open using the definition of an


open set directly. Let x ∈ (a, ∞). Then x > a, and we may set r := x−a > 0.
We have
B(x; r) = (x − r, x + r) = (a, x + r) ⊂ (a, ∞),
as required.
Example 4.3.8. For any a, b ∈ R, the closed interval [a, b] is closed, since

R \ [a, b] = (−∞, a) ∪ (b, ∞),

which is open by the previous example and since unions of open sets are
open.
Proposition 4.3.9. Let F ⊆ Rd . Then F is closed if and only if, for every
sequence (an )∞ ∞
n=1 in F (i.e., an ∈ F for all n), if (an )n=1 converges then

lim an ∈ F.
n→∞

Proof. ⇒: We assume that F is closed, i.e., Rd \ F is open. Let (an )∞ n=1


be a sequence in F which converges, and suppose for a contradiction that
L := lim an is not in F . That means L ∈ Rd \ F . By openness, there exists
n→∞
 > 0 such that B(L; ) ⊆ Rd \ F . Using the definition of convergence, we
may find n such that kan − Lk2 < . But this means that

an ∈ B(L; ) ⊆ Rd \ F,

so that an 6∈ F , which is a contradiction.

63
⇐: We assume that for every sequence in F that converges, the limit is
in F . We need to show that Rd \ F is open.
Take a point x ∈ Rd \ F , and suppose for a contradiction that there isno
 > 0 such that B(x; ) ⊆ Rd \ F . Then for each n ∈ N≥1 , since B x; n1 is
not contained in Rd \ F , this means that there must be some point in this
ball and in F . Let an be such a point, so an ∈ F and kan − xk2 < n1 .
Choosing such an for each n, we arrive at a sequence (an )∞
n=1 in F . Since
0 ≤ kan − xk2 < n1 , we get lim kan − xk2 = 0 by the Squeeze Theorem. In
n→∞
other words,
lim an = x.
n→∞

By hypothesis, it follows that x ∈ F , which is a contradiction.


Definition 4.3.10. Let A ⊆ Rd be a set and let a ∈ Rd .
(i) a is an interior point of A if there exists  > 0 such that B(a; ) ⊆ A.
The interior of A is

A◦ := {x ∈ Rd : x is an interior point}.

(ii) a is an accumulation point of A if there is a sequence (an )∞


n=1 in A
such that a = lim an . The closure of A is
n→∞

A := {x ∈ Rd : x is an accumulation point}.

(iii) a is a boundary point of A if it is an accumulation point of A and it


is not an interior point. The boundary of A is

∂A := {x ∈ Rd : x is a boundary point} = A \ A◦ .

(iv) a is an isolated point of A if there exists  > 0 such that B(a; ) ∩ A =


{a}.
(v) a is a limit point of A if it is an accumulation point and it is not an
isolated point of A.
Example 4.3.11. Let A := (0, 1] ∪ {2}.
• The interior points of A are all points in (0, 1):

A◦ = (0, 1).

64
• The accumulation points of A are all points in [0, 1] ∪ {2}:

A = [0, 1] ∪ {2}.

• The boundary points of A are 0, 1, and 2:

∂A = {0, 1, 2}.

• A has only one isolated point, namely 2.

• The limit points of A are all points in [0, 1].

Example 4.3.12. Let A := Q.

• A◦ = ∅.

• A = ∂A = R.

• A has no isolated points.

• Every point of R is a limit point of A.

Example 4.3.13. Let A := Z.

• A◦ = ∅.

• A = ∂A = Z.

• Every point of A is an isolated point.

• A has no limit points.

Exercise 4.3.1. Prove parts (i)-(ii) of Proposition 4.3.5.


Exercise 4.3.2. Prove the following facts about closed sets.

(a) ∅, Rd are closed sets.

(b) For any finite collection of closed sets, F1 , . . . , Fm ⊆ Rd , their union,

F1 ∪ · · · ∪ Fm ,

is closed.

65
(c) For any arbitrary collection of closed sets {Fα : α ∈ I}, their intersection,
\
Fα ,
α∈I

is closed.

Exercise 4.3.3. [Sav17, Exercise 4.3.1] Prove that a set A ⊆ Rd is bounded if


and only if there exists a ∈ Rd and r > 0 such that A ⊆ B(a; r).
Exercise 4.3.4. [Sav17, Exercise 4.3.2 (b)] Prove that any finite set is closed.
Exercise 4.3.5. Let (an )∞ d d
n=1 be a sequence in R which converges to L ∈ R .
Prove that the set
{an : n ∈ N≥1 } ∪ {L}
is closed.
Exercise 4.3.6. Let A ⊆ Rd with A 6= ∅ and A 6= Rd . Take points a ∈ A and
b ∈ Rd \ A, and for t ∈ [0, 1], define

γ(t) := (1 − t)a + tb.

γ parametrizes a line from a to b – a line that starts in A and ends out of A.

(a) Define
T := {t ∈ [0, 1] : (1 − t)a + tb ∈ A}.
Show that t0 := sup T exists.

(b) Show that with t0 as in part (a), the point γ(t0 ) is in the boundary of A.

(c) Conclude that the only sets in Rd whose boundary is ∅ are ∅ and Rd .

Exercise 4.3.7. Let A ⊆ Rd .

(a) Prove that the closure, A, is closed.

(b) Prove that the interior, A◦ , is open.

(c) Prove that the boundary, ∂A, is closed.

(d) Prove that A◦ = Rd \ (Rd \ A).

66
Exercise 4.3.8. Let (an )∞ d
n=1 be a sequence in R , and define A := {an : n ∈
d
N≥1 }. Show that if x ∈ R is a limit point of A, then there is a subsequence
(ank )∞
k=1 which converges to x.

Exercise 4.3.9. [Sav17, Exercise 4.3.13] Let A1 , . . . , Ad ⊆ R be closed sets.


Show that

A1 × · · · × Ad = {(a1 , . . . , ad ) : ai ∈ Ai for i = 1, . . . , d} ⊆ Rd

is closed.
Exercise 4.3.10. Let A ⊆ Rd and let a ∈ Rd . Prove that the following are
equivalent.

(i) a is a limit point of A.

(ii) For every  > 0,


B(a; A) ∩ A \ {a} =
6 ∅.

(iii) There exists a sequence (an )∞


n=1 in A \ {a} which converges to a.

Exercise 4.3.11. Construct a set A ⊆ R which has some limit points, but
such that none of its limit points are in A itself.

4.4 Compactness
Definition 4.4.1. Let K ⊆ Rd be a set. We say that K is (sequentially)
compact if every sequence (an )∞
n=1 in K has a subsequence that converges to
a point a ∈ K.

In this course, we will frequently abbreviate “sequential compactness” to


just “compactness”. However, there is a different concept which is called
“compactness”, which is equivalent to sequential compactness for subsets of
Rd , though the two aren’t equivalent in some more general settings. This is
discussed further in Interesting Fact 4.4.7.
Example 4.4.2. Let F be a finite set,

F = {x1 , . . . , xk }.

67
Then F is compact. To see this, let (an )∞
n=1 be a sequence in K. Then there
must be some i such that an = xi for infinitely many n. Consequently, we
can realize the constant sequence

(xi , xi , . . . )

as a subsequence (ank )∞
k=1 .
Example 4.4.3. The set R is not compact. To see this, take the sequence
(1, 2, 3, 4, . . . ). This sequence diverges to ∞, and it is not hard to see that
any subsequence of it also diverges to ∞. Hence it has no subsequence which
converges.
Example 4.4.4.
∞ The set A := (0, 1] is not compact. To see this, take the
sequence n1 n=1 in A. This sequence converges to 0, and hence so does
every subsequence of it (Proposition 2.5.4). Hence, it has no subsequence
converging to a point in A.
Theorem 4.4.5 (Heine–Borel Theorem). Let K be a subset of Rd . Then K
is compact if and only if K is closed and bounded.
Proof. ⇒: Suppose that K is compact. To see that K is closed, suppose for
a contradiction that it is not closed. By Proposition 4.3.9, it follows that
there is some sequence (an )∞
n=1 in K which converges, yet such that

lim an 6∈ K.
n→∞

Then by Proposition 2.5.4 (generalized to Rd ), every subsequence of (an )∞ n=1


converges to the same point, and hence no subsequence converges to a point
in K. This contradicts that K is compact.
Next, to see that K is bounded, again suppose for a contradiction that
it is not. Then for each n ∈ N≥1 , we may find some an ∈ K such that
kan k2 ≥ n. This gives us a sequence (an )∞ ∞
n=1 . Any subsequence (ank )k=1 will
be unbounded, and therefore won’t converge. Again, this contradicts that K
is compact.
⇐: We assume that K is closed and bounded, and we let (an )∞ n=1 be a
sequence from K. Since K is bounded, so is the sequence (an )∞ n=1 . Therefore
by the Bolzano–Weierstrass Theorem (Theorem 4.2.8), there is a subsequence
(ank )∞ d
k=1 which converges to some point L ∈ R . Since K is closed and the
sequence (ank )∞
k=1 is in K, it follows from Proposition 4.3.9 that L ∈ K. Thus

(an )n=1 has a subsequence converging to a point in K, as required.

68
Proposition 4.4.6. (i) For any finite collection of compact sets, K1 , . . . , Km ⊆
Rd , their union,
K1 ∪ · · · ∪ Km ,
is compact.
(ii) For any arbitrary collection of compact sets {Kα : α ∈ I}, their inter-
section, \
Kα ,
α∈I
is compact.
Proof. Exercise 4.4.1
Interesting fact 4.4.7. The correct definition of compactness is as follows:
K ⊆ Rd is compact if for any family {Ui }i∈I of open sets in Rd (indexed by
any set I) such that [
K⊆ Ui
i∈I

({Ui }i∈I is an “open cover” of K), there are i1 , . . . , im ∈ I such that


K ⊆ U i1 ∪ · · · ∪ U im
({Ui1 , . . . , Uim } is a “finite subcover”). This definition makes sense in a much
more general setting of topological spaces. Metric spaces are certain special
topological spaces, which include all subsets of Rd (and much more), and for
these, sequential compactness is equivalent to compactness, a result which
you will see in MAT 3120. The Heine–Borel Theorem, however, does not
generalize to this setting. In the more general setting of topological spaces,
compactness and sequential compactness are different notions, neither im-
plying the other.
Exercise 4.4.1. Prove Proposition 4.4.6
Exercise 4.4.2. In the proof of the Heine–Borel Theorem, we use a general-
ization of Proposition 2.5.4 to Rd . State and prove this generalization.
Exercise 4.4.3. [Sav17, Exercise 4.4.6] Let A1 , . . . , Ad ⊆ R be compact sets.
Show that
A1 × · · · × Ad = {(a1 , . . . , ad ) : ai ∈ Ai for i = 1, . . . , d} ⊆ Rd
is compact.

69
Exercise 4.4.4. Let K ⊆ Rd be a compact set. Let f : K → R be a function.
Suppose that for every x ∈ K there exists an open set U containing x and
some M ∈ R such that for all y ∈ U ,

f (y) ≤ M.

Prove that f (K) = {f (x) : x ∈ K} is bounded above.


[Hint. Proof by contradiction. Suppose it is not bounded above, use this to
construct a sequence (an )∞
n=1 in K with certain properties, and then use a
convergent subsequence.]
Exercise 4.4.5. Let K ⊆ Rd . Prove that the following are equivalent.

(i) K is compact.

(ii) Every infinite set E ⊆ K has a limit point z which is in K.

Exercise 4.4.6. Let K ⊆ R be a compact set. Prove that min(K) and max(K)
exist (recall that this means that K contains a lower and upper bound for
itself).
[Hint. Use the Heine–Borel Theorem! ]

70
Chapter 5

Continuous functions

In this chapter, we develop another notion of a limit: here it is for a func-


tion defined on a subset of the real numbers, and the concept concerns the
behaviour of the function as the variable tends to a particular point. This
concept enables us to talk about continuous functions; it will also (in the
next chapter) play a key role in establishing differentiation.

5.1 The limit of a function


Definition 5.1.1. Let X ⊆ Rd and let a ∈ Rd be a limit point of X. Let
f : X → Rm and let L ∈ Rm . We write
lim f (x) = L
x→a

if for every  > 0 there exists δ > 0 such that, if x ∈ X \{a} and kx−ak2 < δ
then kf (x) − Lk2 < .
The idea in the above definition is that when x is close (but not equal) to
a, then f (x) should be close (and possibly equal) to L. It makes sense that
we restrict to a being a limit point, because that is exactly the condition that
tells us that there are points (arbitrarily) close to a inside X (Exercise 4.3.10).
Recall that a limit point a may or may not be inside X itself; however, if
a ∈ X, then the value f (a) does not matter at all to the limit lim f (x) (or
x→a
whether this limit exists).
Example 5.1.2. Define f : R → R by f (x) := 2x + 5. Then for any a ∈ R,
we have lim f (x) = 2a + 5.
x→a

71
To see this, set L := 2a + 5, and let  > 0 be given. Set δ := 2 . Then if
x ∈ R \ {a} and |x − a| < δ = 2 , then

|f (x) − L| = |2x + 5 − (2a + 5)| = 2|x − a| < 2 = .
2
Example 5.1.3. Define g : R → R by g(x) := x2 . Then for any a ∈ R, we
have lim g(x) = a2 .
x→a
To see this, set L := a2 and let  > 0 be given. Now, for any x ∈ R, we
have
|g(x) − L| = |x2 − a2 | = |x − a| · |x + a|.
We want to make this < . We can make the term |x − a| arbitrarily small,
but for the |x + a| term, it might not be so small but we can at least try to
bound it by something fixed. Indeed, if |x − a| < 1 then

|x + a| = |x − a + 2a| ≤ |x − a| + 2|a| < 1 + 2|a|.



So, we now see that we want |x − a| < 1+2|a| .
n o

Now we set δ := min 1, 1+2|a| . If |x − a| < δ then

|g(x) − L| = |x − a| · |x + 1|
≤ |x − a|(1 + 2|a|)

< (1 + 2|a|)
1 + 2|a|
= ,

as required.
We next establish uniqueness of limits of a function, in much the same
way as we did for sequences.
Proposition 5.1.4. Let X ⊆ Rd and let a ∈ Rd be a limit point. Let
f : X → Rm and let L, L0 ∈ Rm . If lim f (x) = L and lim f (x) = L0 then
x→a x→a
L = L0 .
0
Proof. Suppose for a contradiction that L 6= L0 . Set  := kL−L
2
k2
. Using the
definition of limit twice simultaneously, we may find δ > 0 such that for all
x ∈ X \ {a} with kx − ak2 < δ,

kf (x) − Lk2 <  andkf (x) − L0 k2 < .

72
Since a is a limit point of X, we may find x ∈ X ∩ B(a; δ) \ {a} (Exercise
4.3.10); that is, x ∈ X, x 6= a, and kx − ak2 < δ.
Consequently, kf (x) − Lk2 <  and kf (x) − L0 k2 < . But then by using
the Triangle Inequality,
kL − L0 k2 ≤ kL − f (x)k2 + kf (x) − L0 k2 <  +  = kL − L1k2 ,
a contradiction.
Next, we shall connect limits of a function with limits of a sequence.
This will be instrumental towards quickly proving basic properties (such as
algebra of limits).
Proposition 5.1.5 (Sequential Characterization of Limits). Let X ⊆ Rd
and let a ∈ Rd be a limit point. Let f : X → Rm and let L ∈ Rm . Then
lim f (x) = L if and only if for every sequence (an )∞
n=1 in X \ {a} which
x→a
converges to a, we have
lim f (an ) = L.
n→∞

Proof. ⇒: We assume that lim f (x) = L. To show that lim f (an ) = L, let
x→a n→∞
 > 0 be given. Choose δ > 0 such that if x ∈ X \ {a} and kx − ak2 < δ then
kf (x) − Lk2 < .
Next, since lim an = a, there exists n0 ∈ N≥1 such that if n ≥ n0 then
n→∞
kan − ak2 < δ.
Then for n ≥ n0 , we have an ∈ X \ {a} and kan − ak2 < δ, and thus,
kf (an ) − Lk2 < ,
as required.
⇐: We now assume that for every sequence (an )∞
n=1 in X \ {a} which
converges to a, we have lim f (an ) = L.
n→∞
Suppose for a contradiction that it is not true that lim f (x) = L. This
x→a
means that there is some  > 0 such that for every δ > 0, there exists
x ∈ X \ {a} such that kx − ak2 < δ and kf (x) − Lk2 ≥ . We will now
construct a sequence using this.
Namely, for each n ∈ N≥1 , using δ := n1 , we may choose some an ∈ X \{a}
such that
1
kan − ak2 < and kf (an ) − Lk2 ≥ .
n
73
This gives us a sequence (an )∞ 1
n=1 in X \ {a}. Since 0 ≤ kan − ak2 < n for all
n, the Squeeze Theorem ensures that lim kan − ak2 = 0, i.e., lim an = a.
n→∞ n→∞
By hypothesis, it follows that lim f (an ) = L. But this is a contradiction
n→∞
since we have kf (an ) − Lk2 ≥  for all n.
Proposition 5.1.6 (Algebra of Limits). Let X ⊆ Rd and let a ∈ Rd be a
limit point. Let f, g : X → Rm and γ : X → R be functions which all have
limits at a. Let c ∈ R. Then:
   
(i) lim (f (x) + g(x)) = lim f (x) + lim g(x) .
x→a x→a x→a

(ii) lim (cf (x)) = c lim f (x).


x→a x→a
  
(iii) lim (γ(x)f (x)) = lim γ(x) lim f (x) .
x→a x→a x→a

1 1
(iv) If γ(x) 6= 0 for all x ∈ X and lim γ(x) 6= 0 then lim = .
x→a x→a γ(x) lim γ(x)
x→a

Proof. Exercise 5.1.1. (Use the Sequential Characterization of Limits and


Exercise 4.2.2.)
Theorem 5.1.7 (Squeeze Theorem). Let X ⊆ Rd and let a ∈ Rd be a limit
point. Let f, g, h : X → R be functions which satisfy
f (x) ≤ g(x) ≤ h(x) for all x ∈ X.
If
lim f (x) = lim h(x) = L
x→a x→a
then
lim g(x) = L
x→a
as well.
Proof. Exercise 5.1.2.
Given a function f : X → Rm , we may define functions f1 , . . . , fm : X →
R by
f (a) = (f1 (a), . . . , fm (a)), a ∈ X.
(Equivalently, fi is the composition of f with the ith coordinate projection
Rm → R.) We call f1 , . . . , fm the component functions of f , and we write
f = (f1 , . . . , fm ).

74
Proposition 5.1.8. Let X ⊆ Rd and let a ∈ Rd be a limit point. Let
f = (f1 , . . . , fm ) : X → Rm be a function and let L = (L1 , . . . , Lm ) ∈ Rm .
Then lim f (x) = L if and only if lim fi (x) = Li for each i = 1, . . . , m.
x→a x→a

Proof. Exercise 5.1.3. (Use the Sequential Characterization of Limits and


Proposition 4.2.3.)
The order structure in R allows us to talk of one-sided limits when we
have a function of one variable.
Definition 5.1.9. Let X ⊆ R, let f : X → Rd be a function and let L ∈ Rd .
If a ∈ R is a limit point of X ∩ (a, ∞), then we write
lim f (x) = L
x→a+

if for every  > 0 there exists δ > 0 such that, if x ∈ X, x > a, and
|x − a| < δ
then
|f (x) − L| < .
Likewise, if a ∈ R is a limit point of X ∩ (−∞, a), then we write
lim f (x) = L
x→a−

if for every  > 0 there exists δ > 0 such that, if x ∈ X, x < a, and
|x − a| < δ
then
|f (x) − L| < .
Example 5.1.10. Define f : R → R by

−1,
 x < 0;
f (x) := 0, x = 0;

1, x > 0.

Then one can see that


lim f (x) = 1
x→0+
and
lim f (x) = −1.
x→0−

75
Exercise 5.1.1. Prove Proposition 5.1.6.
Exercise 5.1.2. Prove Theorem 5.1.7.
Exercise 5.1.3. Prove Proposition 5.1.8.
Exercise 5.1.4. Define f : R → R by
x+1
f (x) := .
x+3
Prove directly using the definition that
1
lim f (x) = .
x→0 3
Exercise 5.1.5. Suppose f : R → R is a function, L ∈ R, and
 
1
lim f = L.
n→∞ n

Does it follow that lim+ f (x) = L? (Give a proof or counterexample.)


x→0

Exercise 5.1.6. Let (an )∞


n=1 be a sequence in R
m
and let L ∈ Rm . Define a
function  
1 1
f : 1, , , . . . → Rm
2 3
by  
1
f = an .
n
Prove that lim an = L if and only if lim f (x) = L.
n→∞ x→0
Exercise 5.1.7. Prove that the following limits do not exist.

(i) lim x1 .
x→0

1

(ii) lim sin x
.
x→0

Exercise 5.1.8. Let X ⊆ R, let a ∈ R be a limit point of X ∩ (a, ∞). Let


f : X → Rm and let L ∈ Rm . Define g := f |X∩(a,∞) : X ∩ (a, ∞) → Rm .
Prove that
lim+ f (x) = L iff lim g(x) = L.
x→a x→a

76
Exercise 5.1.9. (cf. [Sav17, Exercise 5.1.8]) Let X ⊆ R and let a ∈ R be a
limit point of both X ∩ (a, ∞) and X ∩ (−∞, a). Let f : X → Rd be a
function and let L ∈ Rd . Prove that

lim f (x) = L
x→a

if and only if both

lim f (x) = L and lim f (x) = L.


x→a+ x→a−

5.2 Continuity
We now formulate the notion of a continuous function by making use of the
concept of a limit.

Definition 5.2.1. Let X ⊆ Rd and let a ∈ X be a point which is not isolated.


Let f : X → Rm . We say f is continuous at a if

lim f (x) = f (a).


x→a

If a ∈ X is an isolated point, then we say that any function f : X → Rm


is continuous at a.

Making use of the definition of limit, we find that f : X → Rm is contin-


uous at a ∈ X if for any  > 0 there exists δ > 0 such that, if x ∈ X and
ka − xk2 < δ then kf (a) − f (x)k2 < . This reformulation says that f is
continuous if nearby points get sent to nearby images; it works whether or
not a is an isolated point.
Example 5.2.2. Define f : R → R by f (x) := 2x + 5. Then using Example
5.1.2, we have for any a ∈ R,

lim f (x) = f (a),


x→a

and thus f is continuous at every a ∈ R.


Example 5.2.3. Define f : [0, ∞) → R by
(
sin x1 ,

if x > 0;
f (x) :=
0, if x = 0.

77
Then f is not continuous at 0. To see this, we need to show that lim f (x) = 0
x→0
does not hold. Consider the sequence
 ∞
∞ 1
(an )n=1 =
π(2n + 0.5) n=1
which converges to 0. Note that the corresponding sequence (f (an ))∞
n=1 is
the constant sequence
(1, 1, . . . ),
which does not converge to 0.
Example 5.2.4. Define g : [0, ∞) → R by
(
x sin x1 ,

if x > 0;
g(x) :=
0, if x = 0.

This function oscillates similarly to the previous example, but the magnitude
of the oscillation is tempered by the factor of x. We shall prove that this
function is continuous at 0.
For this, consider functions f, h : [0, ∞) → R given by f (x) := x, h(x) :=
−x. Then we have f (x) ≤ g(x) ≤ h(x) for all x, and
lim f (x) = 0 = lim h(x).
x→∞ x→∞

Therefore, by the Squeeze Theorem, lim g(x) = 0 = g(0), as required.


x→∞

Proposition 5.2.5. Let X ⊆ Rd and let Y ⊆ Rm . Let f : X → Y ⊆ Rm


and g : Y → Rn be functions. Let a ∈ X. Suppose that f is continuous at a
and g is continuous at f (a). Then g ◦ f : X → Rn is continuous at a.
Proof. Let  > 0 be given. Since g is continuous at f (a), there exists η > 0
(we’re using η instead of δ) such that for all y ∈ Y , if ky − f (a)k2 < η then
kg(y) − g(f (a))k2 < . Next, since f is continuous at a, using η in place of
 in that definition, we obtain δ > 0 such that for all x ∈ X, if kx − ak2 < δ
then kf (x) − f (a)k2 < η.
Now, putting these together, if x ∈ X and kx − ak2 < δ then kf (x) −
f (a)k2 < η and so (viewing f (x) as y),
kg(f (x)) − g(f (a))k2 < ,
as required.

78
Proposition 5.2.6. Let X ⊆ Rd and let a ∈ Rd be a limit point. Let
f, g : X → Rm and γ : X → R be functions which are all continuous at a.
Let c ∈ R. Then:
(i) f + g is continuous at a.

(ii) c · f is continuous at a.

(iii) γ · f is continuous at a.
1
(iv) If γ(x) 6= 0 for all x ∈ X then γ
is continuous at a.
Proof. These follow from Proposition 5.1.6.
Exercise 5.2.1. [Leb16, Exercise 3.2.3] Define f : R → R by
(
x, if x ∈ Q;
f (x) = 2
x, if x 6∈ Q.

Prove that f is continuous at 1 and discontinuous at 2.


Exercise 5.2.2. Let X ⊆ Rd , let f : X → Rm , and let a ∈ X. Let r > 0 and
define
g := f |X∩B(a;r) : X ∩ B(a; r) → Rm .
Prove that f is continuous at a if and only if g is continuous at a.
Exercise 5.2.3. [Sav17, Exercise 5.2.3] Let f : Rd → Rm . Recall that for
B ⊆ Rm the preimage of B under f is

f −1 (B) := {x ∈ Rd : f (x) ∈ B}.

Prove that f (x) is continuous for all x ∈ Rd if and only if for every open set
U of Rm , the preimage f −1 (U ) is open. Prove also that f is continuous if
and only if for every closed set F of Rm , the preimage f −1 (F ) is closed.
Exercise 5.2.4. Define the Dirichlet function f : R → R by
(
0, if x 6∈ Q;
f (x) := 1
q
, if x = pq in lowest terms (with p ∈ Z and q ∈ N≥1 ).

Prove that for any a ∈ R, lim f (x) = 0. From this, determine the points at
x→a
which f is continuous.

79
5.3 Properties of continuous functions
In the previous section, we defined what it means for a function to be con-
tinuous at a single point. Here we consider functions that are “globally”
continuous, or continuous at every point of their domain. Such functions
have some strong and useful properties, as we shall see.
Definition 5.3.1. Let X ⊆ Rd and let f : X → Rm be a function. We say
that f is continuous (on X) if f is continuous at a for every a ∈ X.
Theorem 5.3.2. Let K ⊆ Rd be compact and let f : K → Rm be a continu-
ous function. Then its image, f (K), is also compact.
Proof. Let (yn )∞
n=1 be a sequence in f (K); we need to find a subsequence that
converges to a point in f (K). By definition of f (K), we can find xn ∈ K
for each n such that yn = f (xn ). Since K is compact, there is a subsequence
(xnk )∞ ∞
k=1 (of the sequence (xn )n=1 ) which converges to a point a ∈ K. Since
f is continuous at a, we have

lim f (x) = f (a),


x→a

and therefore by the Sequential Characterization of Limits (Proposition 5.1.5),


lim f (xnk ) = f (a). This says that the subsequence (ynk )∞
k=1 converges to the
k→∞
point f (a) ∈ f (K), as required.
This has an important consequence for the problem of optimizing func-
tions.
Corollary 5.3.3 (Extreme Value Theorem). Let K ⊂ Rd be compact and
nonempty, and let f : K → R be a continuous function. Then there exists
xmin , xmax ∈ K such that for all x ∈ K,

f (xmin ) ≤ f (x) ≤ f (xmax ).

In other words, the image of f is bounded above and below, and it attains its
bounds.
Proof. By the previous theorem, f (K) is compact, By Exercise 4.4.6, min(f (K))
and max(f (K)) exist: that is, there are ymin , ymax ∈ f (K) such that

ymin ≤ z ≤ ymax

80
for all z ∈ f (K). We may take xmin , xmax ∈ K such that
f (xmin ) = ymin and f (xmax ) = ymax .
Then it follows that for all x ∈ K,
f (xmin ) ≤ f (x) ≤ f (xmax ).

Remark 5.3.4. If X ⊆ Rd is not compact, then it is possible that f (X) is not


bounded, or that it is bounded but it does not attain its bounds. Here are
some examples.
Define f : (0, 1] → R by f (x) := x1 . Then f ((0, 1]) = [1, ∞) is not
bounded above.
2
Define g : R → R by g(x) := x2x+1 . Then g(R) = [0, 1), which is bounded,
but has no maximum.
Theorem 5.3.5 (Intermediate Value Theorem). Let f : [a, b] → R be a
continuous function. Let y ∈ R be any value between f (a) and f (b). Then
there exists z ∈ [a, b] such that f (z) = y.
Proof. We may assume without loss of generality that f (a) ≤ f (b), since in
the other case, we may replace f with −f (and y with −y). Define
S := {x ∈ [a, b] : f (x) ≤ y}.
This set is nonempty (it contains a) and bounded above (by b). Therefore,
z := sup(S) exists.
We need only prove that f (z) = y.
To show f (z) ≥ y, assume for a contradiction that f (z) < y. Then using
 := y − f (z) we can find δ such that for all x ∈ [a, b] with |x − z| < δ, we
have |f (x) − f (z)| < . We may find x ∈ [a, b] with |x − z| < δ and x > z
(since z 6= b), and for such x,
f (x) < f (z) +  = f (z) + y − f (z) = y.
Hence, x ∈ S, contradicting that z is an upper bound.
Next, to show f (z) ≤ y, assume for a contradiction that f (z) > y. Then
using  := f (z) − y we can find δ such that for all x ∈ [a, b] with |x − z| < δ,
we have |f (x) − f (z)| < . Then for all x ∈ (z − δ, z], we have
f (x) > f (z) −  = f (z) − (f (z) − y) = y,

81
so that x 6∈ S. Since z is an upper bound for S, this shows that z − δ is also
an upper bound. But that contradicts that z is the least upper bound.
The Intermediate Value Theorem is extremely powerful, since it gives us
criteria (often easy to check) for an equation to have a solution. For example,
for any p ∈ N≥0 and α > 0, the function f : [0, ∞) → R defined by

f (x) := xp − α

is continuous. We have f (0) = −α < 0 and (since np → ∞ as n → ∞) we can


find b ∈ [0, ∞) such that f (b) > 0. Then the Intermediate Value Theorem
immediately tells us that there exists z ∈ [0, ∞) such that f (z) = 0, i.e.,
z p = α. In this way, we are immediately able to define pth roots.

Proposition 5.3.6. Let f : [a, b] → R be a continuous function. Then


f ([a, b]) = [c, d] for some c, d ∈ R.

Proof. By the Extreme Value Theorem, there exist xmin , xmax ∈ [a, b] such
that
f ([a, b]) ∈ [f (xmin ), f (xmax )].
Set c := f (xmin ) and d := f (xmax ).
We thus have one containment, f ([a, b]) ⊆ [c, d], and we only need to
prove the other containment. Let y ∈ [c, d]. Then by the Intermediate Value
Theorem, there exists z between xmin and xmax such that f (z) = y. Thus,
y ∈ f ([a, b]), as required.
Exercise 5.3.1. [TBB08, Exercise 5.7.1] Give an example of a function f :
R → R such that for all a ∈ R, f is not continuous at a, yet such that the
conclusion of the Extreme Value Theorem holds, i.e., there exists xmin , xmax ∈
R such that for all x ∈ R,

f (xmin ) ≤ f (x) ≤ f (xmax ).

Exercise 5.3.2. Let f : R → R be a periodic function, i.e., such that there


exists P > 0 such that f (P + x) = f (x) for all x ∈ R. Suppose that f is
continuous. Prove that the conclusion of the Extreme Value Theorem holds,
i.e., there exists xmin , xmax ∈ R such that for all x ∈ R,

f (xmin ) ≤ f (x) ≤ f (xmax ).

82
Exercise 5.3.3. Let f : [a, b] → [a, b] be a continuous function. Prove that
there exists y ∈ [a, b] such that f (y) = y (such y is called a fixed point).
Exercise 5.3.4. Let I ⊆ R be an interval and let f : I → R be a continuous
function. Prove that f (I) is an interval.
Exercise 5.3.5. Let f : Rd → R be a continuous function. Suppose there
exists x+ , x− ∈ Rd such that

f (x− ) < 0 < f (x+ ).

Prove that there exists x0 ∈ Rd such that f (x0 ) = 0. For a bigger challenge,
prove that there are infinitely many such points.
Exercise 5.3.6. [Leb16, Exercise 3.3.3] Let f : (0, 1) → R be a continuous
function such that lim f (x) = lim f (x). Prove that f attains a minimum or
x→0 x→1
f attains a maximum (but possibly not both): that is, either there exists
xmin ∈ (0, 1) such that f (xmin ) ≤ f (x) for all x ∈ (0, 1), or there exists
xmax ∈ (0, 1) such that f (x) ≤ f (xmax ) for all x ∈ (0, 1).

5.4 Inverses of continuous functions


Recall that a function f : A → B is injective or one-to-one if for all x, y ∈ A,

f (x) = f (y) ⇒ x = y.

A function f : A → B is surjective or onto if

f (A) = B.

When f is both injective and surjective, it is called bijective, and it follows


that it has an inverse: a function f −1 : B → A such that

f −1 ◦ f = idA , f ◦ f −1 = idB .

Namely, for y ∈ B, one defines

f −1 (y) := x

where x ∈ A is the unique element satisfying f (x) = y.

83
Often we are given an injective function f : A → B, and by replacing the
codomain B with f (B), we get a function f : A → f (A) which is surjective
for free, and thus bijective.
When we start with a continuous injective function f : A → Rd , it would
be nice if its inverse f −1 were also continuous; unfortunately, this is not true
in this level of generality, and we need to put restrictions on the domain A.
Example 5.4.1. Define f : [0, 1) ∪ [2, 3] → R by
(
t, t ∈ [0, 1);
f (t) :=
t − 1, t ∈ [2, 3].

This function is injective, so it has an inverse f −1 : [0, 2] → R which is given


by (
t, t ∈ [0, 1);
f −1 (t) =
t + 1, t ∈ [1, 2].
The function f is continuous but f −1 is not.
For positive results, we will focus on maps from an interval into R; how-
ever see also Exercise 5.4.2 for a different general result.

Definition 5.4.2. Let X ⊆ R and let f : X → R be a function.

(i) We say f is (weakly) increasing if for x, y ∈ X,

x ≤ y =⇒ f (x) ≤ f (y).

(ii) We say f is strictly increasing if for x, y ∈ X,

x < y =⇒ f (x) < f (y).

(iii) We say f is (weakly) decreasing if for x, y ∈ X,

x ≥ y =⇒ f (x) ≥ f (y).

(iv) We say f is strictly decreasing if for x, y ∈ X,

x > y =⇒ f (x) > f (y).

84
(Note that when X = N≥1 then the above concepts for a function f :
N≥1 → R agree with those already defined in Definition 2.4.1.)
Lemma 5.4.3. Let a < b and let f : [a, b] → R be a continuous function.
The following are equivalent.
(i) f is either strictly increasing or strictly decreasing.

(ii) f is injective.
Proof. (i) ⇒ (ii): This is immediate from the definitions.
(ii) ⇒ (i): Assume without loss of generality that f (a) < f (b), and let’s
prove that f is strictly increasing. If it is not then there exists x1 , x2 ∈ [a, b]
such that x1 < x2 and f (x1 ) ≥ f (x2 ).
Consider two cases:

If f (x1 ) ≥ f (b) then we have

f (a) ≤ f (b) ≤ f (x1 ),

so by applying the Intermediate Value Theorem to f |[a,x1 ] (with y := f (b)),


there exists z ∈ [a, x1 ] such that f (z) = f (b). Since f is injective, this would
mean that z = b, but z ≤ x1 < x2 ≤ b, so this is a contradiction.
If f (x1 ) ≤ f (b), then we have

f (x2 ) ≤ f (x1 ) ≤ f (b),

so by applying the Intermediate Value Theorem to f |[x2 ,b] (with y := f (x1 )),
there exists z ∈ [x2 , b] such that f (z) = f (x1 ). Again, since f is injective,
this would mean that z = x1 , but z ≥ x2 > x1 , so this is a contradiction.
Theorem 5.4.4. Let I ⊆ R be an interval and let f : I → R be an injective
continuous function. Then f −1 : f (I) → R is continuous.

85
Proof. On every bounded closed subinterval [a, b] of I, f is either strictly
increasing or strictly decreasing. From this it is clear that f is either strictly
increasing or strictly decreasing on all of I. Assume, without loss of gener-
ality, that f is strictly increasing.
Let b ∈ f (I); to show that f −1 is continuous at b, we must show lim f −1 (y) =
y→b
−1
f (b). So, let  > 0 be given.
Set a := f −1 (b) (so that f (a) = b). Let us assume that a ∈ I ◦ – in the
other case, a is an endpoint of the interval I and then we will need to adjust
the following argument slightly. Since there is no harm in decreasing , we
may further assume that a − , a +  ∈ I. Since f is strictly increasing, we
have
f (a − ) < f (a) < f (a + ).
We then set
δ := min{f (a) − f (a − ), f (a + ) − f (a)} > 0.
We are done once we prove the following claim.
Claim. If y ∈ f (I) and |y − b| < δ, then |f −1 (y) − f −1 (b)| < .
To prove this claim, suppose it is false, so either
f −1 (y) ≤ f −1 (b) −  or f −1 (y) ≥ f −1 (b) + .
Set x := f −1 (y) (so that f (x) = y), so the above reads
x ≤ a −  or x ≥ a + .
In the first case, since f is strictly increasing, we get
y = f (x) ≤ f (a − ) ≤ f (a) − δ = b − δ.
In the second case, we similarly obtain
y = f (x) ≥ f (a + ) ≥ f (a) + δ = b + δ.
In either case, we get a contradiction to the assumption that |y − b| < δ.
This concludes the proof of the claim.
Exercise 5.4.1.
√ Let p ∈ N≥2 . Prove that the function f : [0, ∞) → R given
by f (x) := x is continuous.
p

Exercise 5.4.2. Let K ⊆ Rd be a compact set and let f : K → Rm be a


continuous injective function. Prove that f −1 : K → Rd is continuous.
[Hint. Generalize Exercise 5.2.3 to say that if X ⊆ Rd is closed then a
function f : X → Rm is continuous if and only if for every closed set F of
Rm , the preimage f −1 (F ) is closed.]

86
5.5 Uniform continuity
Let X ⊆ Rd and let f : X → Rm be a function. If we unwind the definition
of “f is a continuous function”, it says:
For any x ∈ X and  > 0 there exists δ > 0 such that if y ∈ X and
|x − y| < δ then |f (x) − f (y)| < .
In this definition, the δ is allowed to depend on  and on x. It is crucial
that δ is allowed to depend on , to capture the idea that “close points get
sent to close images” (see Exercise 5.5.3). However, it is less clear, at first,
what role the dependence on x plays.
Here we explore this subtlety, by first introducing a variant on continuity
where δ is not allowed to depend on x.
Definition 5.5.1. Let X ⊆ Rd and let f : X → Rm be a function. We say
that f is uniformly continuous (on X) if for any  > 0 there exists δ > 0
such that for any x, y ∈ X, if kx − yk2 < δ then kf (x) − f (y)k2 < .
Example 5.5.2. Define f : R → R by f (x) := |x|. This is uniformly continu-
ous.
To see this, given  > 0, we set δ := . Then if x, y ∈ R and |x − y| < δ,
then
|x| − |y| ≤ |x − y| < δ = .
Example 5.5.3. Define f : R → R by f (x) := x2 Then f is continuous (by
Proposition 5.2.6 (iii)) but not uniformly continuous.
To see that it is not uniformly continuous, suppose for a contradiction
that it is. Then using  := 1 there would be some δ > 0 such that for all
x, y ∈ R, if |x − y| < δ then |f (x) − f (y)| < 1.
However, we may take
1 δ
x := , y := x + ,
δ 2
so that |x − y| = 2δ < δ. However,
|f (x) − f (y)| = y 2 − x2
= (y − x)(y + x)
δ
≥ (x + x)
2
δ 2
= · = 1,
2 δ

87
a contradiction.
The obstruction to uniform continuity in the previous example came from
points that are large. In fact, in the next example, we should that if we
eliminate large points from the domain then the function becomes uniformly
continuous.
Example 5.5.4. Consider the restriction of the previous example to [a, b]
where a, b ∈ R with a < b. That is, f : [a, b] → R given by f (x) := x2 .
This function is uniformly continuous.
To see this, let  > 0 be given. Set M := max{|a|, |b|}, so that M is an

upper bound for {|x| : x ∈ [a, b]}. Take δ := 2M . Then for x, y ∈ [a, b] with
|x − y| < δ, we have

|f (x) − f (y)| = |x2 − y 2 |


= |x − y||x + y|
≤ |x − y|(|x| + |y|)
< δ(M + M )
= ,

as required.
In fact, the previous example is a special case of a general phenomenon:
continuous functions on compact sets are automatically uniformly continuous.

Theorem 5.5.5. Let K ⊆ Rd be compact and let f : K → Rm be continuous.


Then f is uniformly continuous.

Proof. We prove this by contradiction: suppose that f is not uniformly con-


tinuous, so there exists  > 0 such that for every δ > 0 there exist x, y ∈ K
with kx − yk < δ and kf (x) − f (y)k ≥ .
In particular, for each n, we may choose some xn , yn ∈ X such that
kxn − yn k2 < n1 yet kf (xn ) − f (yn )k2 ≥ . This gives us two sequences,
(xn )∞ ∞
n=1 and (yn )n=1 , in K.
Since K is compact, there is a subsequence (xnk )∞k=1 which converges to

a point a ∈ K. The subsequence (ynk )k=1 must also converge to a, since

kynk − ak2 ≤ kynk − xnk k2 + kxnk − ak2 → 0,

and so by the Squeeze Theorem, kynk − ak2 → 0.

88
Since f is continuous and by using the Sequential Characterization of
Limits (Proposition 5.1.5), it follows that

lim f (xnk ) = f (a) = lim f (ynk ),


k→∞ k→∞

and in particular,
lim kf (xnk ) − f (ynk )k2 = 0.
k→∞

But this is a contradiction since kf (xnk ) − f (ynk )k2 ≥  for all k.


Exercise 5.5.1. Determine which of the following functions are uniformly
continuous.

(a) f : (0, ∞) → R defined by f (x) := x1 .

(b) f : [1, ∞) → R defined by f (x) := x1 .


1
(c) f : R → R defined by f (x) := 1+x2

1

(d) f : (0, ∞) → R defined by f (x) := sin x
.

(e) f : [1, ∞) → R defined by f (x) := ln(x).

(f) f : R → R defined by f (x) := sin(x2 ).

Exercise 5.5.2. Let f : Rd → Rm be continuous. Prove that for any bounded


set X ⊂ Rd , the restricted function f |X is uniformly continuous.
Exercise 5.5.3. Consider modifying the definition of continuity so that δ may
not depend on  (although it can depend on x): for any x there exists δ > 0
such that for any  > 0, if y ∈ X and |x − y| < δ, then |f (x) − f (y)| < .
Show that if X = R and f : X → Rm satisfies this definition then f must be
a constant function.
Exercise 5.5.4. Let X = X1 ∪ X2 ⊆ Rd and let f : X → Rm be a func-
tion. Suppose that f |X1 and f |X2 are both uniformly continuous. Must
f be uniformly continuous? [Hint. Consider the case X1 = (−∞, 0) and
X2 = (0, ∞).]
Exercise 5.5.5. Let X ⊆ Rd be bounded and let f : X → Rm be uniformly
continuous. Prove that f (X) is bounded.

89
Exercise 5.5.6. Let f : Rd → Rm be a function and let L ∈ Rm . Let us write
lim f (x) = L if for every  > 0 there exists M such that if x ∈ Rd and
kxk2 →∞
kxk2 ≥ M then kf (x) − Lk2 < .
Prove that if f is continuous and lim f (x) = L then f is uniformly
kxk2 →∞
continuous.
Exercise 5.5.7. Let X ⊆ Rd and let f : X → Rm be a function. f is Lipschitz
if there is some M > 0 such that for all x, y ∈ X,

kf (x) − f (y)k2 ≤ M kx − yk2 .

(i) Prove that if f is Lipschitz then it is uniformly continuous.



(ii) Prove that f : [0, 1] → R defined by f (t) := t is uniformly continuous
but not Lipschitz.

(iii) Prove that f : [1, ∞) → R defined by f (t) := t is Lipschitz.

Exercise 5.5.8. (Challenging!) Let X ⊆ Rd and let f : X → Rm be uniformly


continuous. Prove that there is a continuous function g : X → Rm such that
g|X = f .
[For a point a ∈ X \ X, take a sequence (xn )∞n=1 in X that converges to a.
Prove that (f (xn ))∞
n=1 is Cauchy, so define g(a) := lim f (xn ). Prove that
n→∞
this makes g a well-defined function and that it is continuous.]

5.6 Infinite limits and limits at infinity


It is often useful to have the concept of ∞ at our disposal when speaking of
limits. We have a few definitions related to this concept.

Definition 5.6.1. Let A ⊆ Rd and let f : A → Rm .

• If m = 1 and a ∈ Rd is a limit point of A, then we write lim f (x) = ∞


x→a
if for every R > 0 there exists δ > 0 such that if x ∈ A \ {a} and
kx − ak2 < δ then
f (x) > R.

90
• Similarly, if m = 1 and a ∈ Rd is a limit point of A, then we write
lim f (x) = −∞ if for every R > 0 there exists δ > 0 such that if
x→a
x ∈ A \ {a} and kx − ak2 < δ then

f (x) < −R.

• If d = 1, A is not bounded above, and L ∈ Rm , we write lim f (x) = L


x→∞
if for every  > 0 there exists R > 0 such that if x ∈ A and x > R then

kf (x) − Lk2 < .

• Similarly, if d = 1, A is not bounded below, and L ∈ Rm , we write


lim f (x) = L if for every  > 0 there exists R > 0 such that if x ∈ A
x→−∞
and x < −R then
kf (x) − Lk2 < .

• If A is not bounded and L ∈ Rm , we write lim f (x) = L if for every


kxk2 →∞
 > 0 there exists R > 0 such that if x ∈ A and kxk2 > R then

kf (x) − Lk2 < .

One can also define things such as lim f (x) = ∞, by splicing these
kxk2 →∞
definitions appropriately (Exercise 5.6.1).
Let f : N≥1 → Rm , and form the associated sequence (an )∞
n=1 . Observe
that for L ∈ Rm ,
lim f (x) = L iff lim an = L.
x→∞ n→∞

Exercise 5.6.1. Splice the definitions in order to define lim f (x) = ∞, lim f (x) =
x→∞ x→−∞
∞, lim f (x) = −∞, lim f (x) = −∞, lim f (x) = −∞, and lim f (x) =
x→∞ x→−∞ kxk2 →∞ kxk2 →−∞
−∞.
Exercise 5.6.2. Let A ⊆ Rd , B ⊆ Rm , and let f : A → Rm , g : B →
Rn , such that f (A) ⊆ B. Let a ∈ Rd be a limit point of A. Suppose
that lim kf (x)k2 = ∞ (this makes sense since x 7→ kf (x)k2 is a real-valued
x→a
function) and lim g(y) = L ∈ Rn . Prove that
kyk2 →∞

lim g(f (x)) = L.


x→a

91
Exercise 5.6.3. Let f : Rd → Rm be a continuous function. Suppose that
lim f (x) exists. Prove that f is bounded.
kxk2 →∞
[Use the Extreme Value Theorem.]

92
Chapter 6

Differentiation

We will now use the concept of a limit in order to define differentiability and
the derivative of a real-variable function. Differentiation is a fundamental
operation in calculus; the machinery built in the previous chapter will allow
us to rigorously establish the key properties of the derivative.

6.1 The derivative


Definition 6.1.1. Let X ⊆ R, let f : X → R be a function, let a ∈ X be a
non-isolated point. We write

f (x) − f (a)
f 0 (a) := lim
x→a x−a
(either as a real number, as ±∞, or as “not existing”). When this limit
exists and is finite, we say f is differentiable at a. If all points of X are
non-isolated and f is differentiable at every point of X, then we say f is
differentiable on X. In this case, we get a function f 0 : X → R, also written
df
as dx .

As you have seen in Calculus, f 0 (a) is the slope of the tangent line to the
graph of y = f (x) at the point (a, f (a)).

93
Example 6.1.2. Define f : R → R by f (x) = x2 . Then for a ∈ R,

f (x) − f (a)
f 0 (a) = lim
x→a x−a
x − a2
2
= lim
x→a x − a
(x + a)(x − a)
= lim
x→a x−a
= lim x + a = 2a.
x→a

Thus f is differentiable on R.
Example 6.1.3. Define f : R \ {0} → R by
1
f (x) := .
x
Then for a ∈ R \ {0},

f (x) − f (a)
f 0 (a) = lim
x→a x−a
1
−1
= lim x a
x→a x − a
a−x 1
= lim ·
x→a xa x−a
−1
= lim
x→a xa
−1
= 2.
a
Example 6.1.4. Define f : R → R by f (x) := |x|. If a > 0, note that for x
close to a we have f (x) = x. Thus,

f (x) − f (a) x−a


f 0 (a) = lim = lim = 1.
x→a x−a x→a x − a

If a < 0, for x close to a we have f (x) = −x, and thus,

f (x) − f (a) −x − (−a)


f 0 (a) = lim = lim = −1.
x→a x−a x→a x−a

94
However for a = 0, we have
f (x) − f (0) x−0
lim+ = lim+ = 1,
x→0 x−0 x→0 x − 0

whereas
f (x) − f (0) −x − 0
lim− = lim+ = −1.
x→0 x−0 x→0 x−0
Since these one-sided limits are different, f 0 (0) does not exist (Exercise 5.1.9).
Proposition 6.1.5. Let X ⊆ R, let f : X → R be a function, let a ∈ X be
a non-isolated point. If f is differentiable at a then f is continuous at a.
Proof. We need to show that lim f (x) = f (a). We have
x→a
 
f (x) − f (a)
lim f (x) − f (a) = lim · (x − a)
x→a x→a x−a
 
f (x) − f (a)  
= lim lim x − a (Algebra of Limits)
x→a x−a x→a
0
= f (a) · 0 = 0.

In the above, we were able to use Algebra of Limits (Proposition 5.1.6)


because the limits exist.
We therefore have lim f (x) = f (a), as required.
x→a

The following example shows that, even if f is differentiable everywhere,


the function f 0 might not be continuous.
Example 6.1.6. Define f : R → R by
(
1

x2 sin x
, x 6= 0;
f (x) :=
0, x = 0.

This function is differentiable everywhere; for a 6= 0, this will follow from the
rules of differentiation (Chain Rule, Product Rule, Quotient Rule), and we
have
   
0 1 2 1 −1
f (a) = 2a sin + a cos · 2
a a a
   
1 1
= 2a sin − cos .
a a

95
For a = 0, we have

f (x) − f (0)
f 0 (a) = lim
x→0 x− 0
1
= lim x sin
x→0 x
=0

by Example 5.2.4.
Similarly,
   
0 1 1
lim f (x) = lim 2x sin − cos
x→0 x→0 x x
 
1
= 0 − lim cos ,
x→0 x

but this limit does not exist (just like Exercise 5.1.7 (ii)). Hence, f 0 is not
continuous at 0.
If f is differentiable and f 0 is differentiable, we write the derivative of f 0
as f 00 or f (2) . Similarly we define f 000 = f (3) , and so on.
Exercise 6.1.1. Let n ∈ N≥1 and define f : R → R by f (x) := xn . Directly
compute f 0 (a) for a ∈ R.
Exercise 6.1.2. [TBB08, Exercise 7.2.1] Let X ⊆ R, let f : X → R be a
function, let a ∈ X be a non-isolated point. Show that

f (a + h) − f (a)
f 0 (a) = lim .
h→0 h
Exercise 6.1.3. [TBB08, Exercise 7.2.3] Determine which of the following
functions f : R → R are differentiable at 0.

(i) f (x) := x|x|.


(
1

x sin x
, x 6= 0;
(ii) f (x) :=
0, x = 0.
(
x2 , x ∈ Q;
(iii) f (x) :=
0, x 6∈ Q.

96
Exercise 6.1.4. [Leb16, Exercise 4.1.9] Let X ⊆ R be a set in which all points
are non-isolated. Suppose that f : X → R is differentiable function which
is Lipschitz (see the definition in Exercise 5.5.7). Prove that f 0 : X → R is
bounded.
Exercise 6.1.5. [TBB08, Exercise 7.2.13] Let f : R → R be strictly increasing
and differentiable. Does this imply that f 0 (x) ≥ 0 for all x ∈ R? Does it
imply that f 0 (x) > 0 for all x ∈ R?

6.2 Computation rules for derivatives


In Calculus, you have seen a number of tools for computing derivatives. In
this section, we shall prove these rules.

Proposition 6.2.1. Let X ⊆ R, let f, g : X → R be functions, let a ∈ X be


a non-isolated point. Suppose that f and g are both differentiable at a, and
let c ∈ R. Then

(i) (cf )0 (a) = c(f 0 (a)) and (f + g)0 (a) = f 0 (a) + g 0 (a); (Linearity)

(ii) (f g)0 (a) = f 0 (a)g(a) + f (a)g 0 (a). (Product Rule)

We don’t do the Quotient Rule here; it can be proven using the Chain
Rule and the Product Rule (Exercise 6.2.2).
Proof. Part (i) is Exercise 6.2.1
(iii): We compute

(f g)(x) − (f g)(a) f (x)g(x) − f (a)g(a)


lim = lim
x→a x−a x→a x−a
f (x)g(x) − f (a)g(x) + f (a)g(x) − f (a)g(a)
= lim
 x−a 
x→a
  
f (x) − f (a)  g(x) − g(a)
= lim lim g(x) + f (a) lim
x→a x−a x→a x→a x−a
0 0
= f (a)g(a) + f (a)g (a),

using Algebra of Limits in the third line, and that g is continuous at a


(Proposition 6.1.5) for the last equality.

97
Proposition 6.2.2 (Chain Rule). Let X, Y ⊆ R, let f : X → R and g : Y →
R be functions, let a ∈ X be a non-isolated point. Suppose that f (X) ⊆ Y and
that f (a) is a non-isolated point of Y . Suppose also that f is differentiable
at a and g is differentiable at f (a). Then g ◦ f is differentiable at a, and

(g ◦ f )0 (a) = g 0 (f (a))f 0 (a).

Proof. For y ∈ Y , define


(
g(y)−g(f (a))
y−f (a)
, y 6= f (a);
h(y) := 0
g (f (a)), y = f (a).

Since lim h(y) = g 0 (f (a)) = h(f (a)), h is continuous at f (a).


y→f (a)
Claim. For all x ∈ X \ {a},

g(f (x)) − g(f (a)) f (x) − f (a)


= h(f (x)) .
x−a x−a
To see this claim, we consider two cases. If f (x) 6= f (a), then we have

g(f (x)) − g(f (a)) g(f (x)) − g(f (a)) f (x) − f (a)
= ·
x−a f (x) − f (a) x−a
f (x) − f (a)
= h(f (x)) .
x−a
On the other hand, if f (x) = f (a), then both sides are 0, so the claim holds.
Now, using the claim, we compute

g(f (x)) − g(f (a))


(g ◦ f )0 (a) = lim
x→a x−a
f (x) − f (a)
= lim h(f (x))
x→a x−a
0
= h(f (a))f (a)
= g 0 (f (a))f 0 (a).

In the second-last line we used the fact that h ◦ f is continuous at a, which


is a consequence of Proposition 5.2.5.

98
Proposition 6.2.3 (Inverse Rule). Let X ⊆ R be an interval, let f : X → R
be a continuous, injective function. Let a ∈ X. If f is differentiable at a and
f 0 (a) 6= 0 then f −1 : f (X) → R is differentiable at f (a) and
1
(f −1 )0 (f (a)) = .
f 0 (a)
Proof. For convenience, set b := f (a).
Similarly to the previous proof, we define h : X → R by
(
f (x)−f (a)
x−a
, x=6 a;
h(x) := 0
f (a), x = a.

This function is continuous, and h(a) 6= 0. It follows that h is nonzero


everywhere on some open interval containing a; so by shrinking X, we may
assume that h(x) 6= 0 for all x ∈ X.
Now we observe that, for y ∈ Y \ {b}, we have
f −1 (y) − f −1 (b) f −1 (y) − a
=
y−b f (f −1 (y)) − f (a)
1
= .
h(f −1 (y))
Since h and f −1 are continuous (the latter by Theorem 5.4.4), so is their
composition. Hence by the Algebra of Limits, we get
f −1 (y) − f −1 (b) 1 1 1 1
lim = −1
= −1
= = 0 .
y→b y−b lim h(f (y)) h(f (b)) h(a) f (a)
y→b

Exercise 6.2.1. Prove Proposition 6.2.1 (i).


Exercise 6.2.2. Let X ⊆ R, let f, g : X → R be a function, let a ∈ X be a
non-isolated point. Suppose that f and g are both differentiable at a, and
that g(x) 6= 0 for all x ∈ X, and g 0 (a) 6= 0.
 0 0 (a)
(a) Using the Chain Rule, prove that g1 (a) = −gg(a)2
.
 0
f
(b) Using (a) and the Product Rule, prove the Quotient Rule: g
(a) =
f 0 (a)g(a)−f (a)g 0 (a)
g(a)2
.

99
Exercise 6.2.3. (cf. [Leb16, Exercise 4.1.11]) Let X ⊆ R, let f, g : X → R be
a function, let a ∈ X be a non-isolated point. Suppose that f is bounded
and that g(a) = g 0 (a) = 0. Consider the function h = f g.
(a) Prove that h0 (a) = 0.
(b) Is it possible to use the product rule to do part (a)?
Exercise 6.2.4. [TBB08, Exercise 7.3.17] Using the fact that the derivative
−1
of sin(x) is cos(x), find a formula
 for the derivative of sin (x) (considered
π π
as a function [−1, 1] → − 2 , 2 ).

6.3 Optimizing differentiable functions


Derivatives are often used for optimizing functions: finding their maximum
and/or minimum (extrema). The precise relationship between derivatives
and extrema has to do with the concept of “local extrema”: the minimum
or maximum of a point in a (possibly) small neighbourhood.
Definition 6.3.1. Let X ⊆ R, let f : X → R, and let a ∈ X be an interior
point.
(i) a is a local minimum of f if there exists r > 0 such that (a−r, a+r) ⊆ X
and
f (a) ≤ f (x) for all x ∈ (a − r, a + r).

(ii) a is a local maximum of f if there exists r > 0 such that (a−r, a+r) ⊆
X and
f (a) ≥ f (x) for all x ∈ (a − r, a + r).

Theorem 6.3.2. Let X ⊆ R, let f : X → R, and let a ∈ X be an interior


point. If f has a local maximum or local minimum at a and f is differentiable
at a, then f 0 (a) = 0.

100
Proof. Assume that f has a local minimum at a (the other case is almost
identical). Let r > 0 be such that (a − r, a + r) ⊆ X and
f (a) ≤ f (x) for all x ∈ (a − r, a + r).
Since f is differentiable, the limit
f (x) − f (a)
lim
x→a x−a
exists, and by the Sequential Characterization of Limits, for any sequence
(xn )∞
n=1 in X converging to a, we will have

f (xn ) − f (a)
f 0 (a) = lim .
n→∞ xn − a
Now, let us first take some sequence (xn )∞
n=1 in (a, a + r) converging to a
from the right: for example,
r
xn := a + .
n+1
Then we have xn − a > 0 and f (xn ) − f (a) ≥ 0. Thus,
f (xn ) − f (a)
≥ 0,
xn − a
which implies that
f (xn ) − f (a)
f 0 (a) = lim ≥ 0.
n→∞ xn − a
However, we may similarly take some sequence (xn )∞
n=1 in (a − r, a) con-
verging to a from the left: for example,
r
xn := a − .
n+1
In this case we have xn − a < 0 and f (xn ) − f (a) ≥ 0, so that
f (xn ) − f (a)
≤ 0,
xn − a
and thus,
f (xn ) − f (a)
f 0 (a) = lim ≤ 0.
n→∞ xn − a
Putting these two inequalities together, we obtain f 0 (a) = 0 as required.

101
Suppose f : [a, b] → R is continuous. By the Extreme Value Theorem, it
attains a maximum and a minimum. Let’s say we want to find a point xmax ∈
[a, b] where it achieves its maximum; the point xmax will in particular be a
local maximum, so if xmax is an interior point of [a, b] and f is differentiable
at xmax , then by the above theorem, f 0 (xmax ) = 0. It is possible, however,
that xmax doesn’t satisfy the hypotheses of the above theorem (it might not
be differentiable, or it might not be an interior point). Altogether, we get
the following three possibilities:

(i) f is differentiable at xmax and f 0 (xmax ) = 0,

(ii) xmax = a or xmax = b (it is not an interior point), or

(iii) f is not differentiable at xmax .

Exercise 6.3.1. Give examples of continuous functions f : [a, b] → R which


attain a maximum at a unique point xmax such that

(a) in the first example, f is not differentiable at xmax ,

(b) in the second example, xmax is a and f is differentiable at xmax ,

Exercise 6.3.2. [TBB08, Exercise 7.5.1] Give an example of a differentiable


function f : R → R such that f 0 (0) = 0 but 0 is not a local maximum nor
minimum.
Exercise 6.3.3. [TBB08, Exercise 7.5.2] Define f : R → R by
(
x4 2 + sin x1 ,

x 6= 0;
f (x) :=
0, x = 0.

(i) Prove that f is differentiable on R.

(ii) Prove that f has a minimum at 0.

(iii) Prove that for every r > 0, f 0 ((−r, r)) contains both (strictly) positive
and negative numbers.

102
6.4 The Mean Value Theorem
The Mean Value Theorem tells us that the average change of a differentiable
function is attained as the derivative of the function at some point. This
will be derived from Rolle’s Theorem, which is the special case in which the
average change of a function is zero.

Theorem 6.4.1 (Rolle’s Theorem). Let f : [a, b] → R be a continuous


function that is differentiable on (a, b). If f (a) = f (b) then there exists
x0 ∈ (a, b) such that
f 0 (x0 ) = 0.

Proof. If f is constant, then f 0 (x0 ) = 0 for all x0 ∈ (a, b). Otherwise, by the
Extreme Value Theorem, let xmin , xmax be the minimal and maximal points
of f , so that
f (xmin ) ≤ f (x) ≤ f (xmax )
for all x ∈ [a, b]. Since x is not constant, we know that either

f (xmin ) < f (a) = f (b) or f (xmax ) > f (a) = f (b),

(or both) and in particular, either xmin ∈ (a, b) or xmax ∈ (a, b). We set x0
equal to one of these two points which is in (a, b).
Then f is differentiable at x0 , and since x0 is a global extreme point for
f , it follows from Theorem 6.3.2 that f 0 (x0 ) = 0, as required.
Next we do a generalization of the Mean Value Theorem. This general-
ization can be used to prove L’Hôpital’s rule (Exercise 6.4.5).

Theorem 6.4.2 (Cauchy’s Mean Value Theorem). Suppose that f, g : [a, b] →


R are continuous functions that are differentiable on (a, b). Then there exists
x0 ∈ (a, b) such that

(f (b) − f (a))g 0 (x0 ) = (g(b) − g(a))f 0 (x0 ).

Proof. Define h : [a, b] → R by

h(x) := (f (b) − f (a))g(x) − (g(b) − g(a))f (x).

Our strategy is to apply Rolle’s Theorem to h.

103
The function h is continuous, and it is differentiable on (a, b) with
h0 (x) = (f (b) − f (a))g 0 (x) − (g(b) − g(a))f 0 (x).
Also, we compute
h(a) = (f (b) − f (a))g(a) − (g(b) − g(a))f (a)
= f (b)g(a) − f (a)g(a) − g(b)f (a) + g(a)f (a)
= f (b)g(a) − g(b)f (a).
Likewise, h(b) = f (b)g(a) − g(b)f (a).
Hence by Rolle’s Theorem, there exists x0 ∈ (a, b) such that h0 (x0 ) = 0.
Looking at the computation for h0 (x), it follows that
(f (b) − f (a))g 0 (x0 ) = (g(b) − g(a))f 0 (x0 ).

Corollary 6.4.3 (Mean Value Theorem). Let f : [a, b] → R be a continuous


function that is differentiable on (a, b). Then there exists x0 ∈ (a, b) such
that
f (b) − f (a)
f 0 (x0 ) = .
b−a
Proof. Define g : [a, b] → R by g(x) := x, then apply Cauchy’s Mean Value
Theorem to f, g to get x0 ∈ (a, b) such that
(f (b) − f (a))g 0 (x0 ) = (g(b) − g(a))f 0 (x0 ).
Since g 0 (x0 ) = 1 and g(b) − g(a) = b − a, this rearranges to become
f (b) − f (a)
f 0 (x0 ) = .
b−a

Exercise 6.4.1. [TBB08, Exercise 7.6.4] Suppose that f : R → R is a differ-


entiable function and there is some c > 0 such that f 0 (x) ≥ c for all x ∈ R.
Prove that lim f (x) = ∞.
x→∞
Exercise 6.4.2. [TBB08, Exercise 7.6.5] Let f : R → R be a twice-differentiable
function (i.e., f 0 (x) and f 00 (x) exist everywhere). Prove that if f has at
least three zeroes (three distinct points x1 , x2 , x3 such that f (x1 ) = f (x2 ) =
f (x3 ) = 0) then there exists x0 ∈ R such that f 00 (x0 ) = 0.

104
Exercise 6.4.3. Suppose that f : [a, b] → R is a continuous function which is
differentiable on (a, b), such that f 0 (x) = 0 for all x ∈ (a, b). Prove that f is
the constant function.
Exercise 6.4.4. Let I ⊆ R be an interval and let f : I → R be a differentiable
function. Recall from Exercise 6.1.4 that if f is Lipschitz (see Exercise 5.5.7
for the definition) then f 0 is bounded. Use the Mean Value Theorem to prove
the converse: that if f 0 is not Lipschitz then f is not bounded.
Exercise 6.4.5. Let I be an interval containing a as an interior point, and
let f, g : I → R be continuous functions, such that g(x) > 0∀x ∈ I \ {a},
g 0 (x) 6= 0∀x ∈ I \ {a}, and f (a) = g(a) = 0.

(i) Let  > 0. Suppose that b ∈ I satisfies

f (b) ≥ g(b)(L + ).

Prove that there is some x ∈ I such that

f 0 (x) ≥ g 0 (x)(L + ).

[Use Cauchy’s Mean Value Theorem.]

(ii) Prove that if

g(x)(L − ) < f (x) < g(x)(L + ) for all x ∈ I.

then
g 0 (x)(L − ) < f 0 (x) < g 0 (x)(L + ) for all x ∈ I.

(iii) Prove that if


f 0 (x)
L := lim exists,
x→a g 0 (x)

then
f (x)
lim = L.
x→a g(x)

(This is one case of L’Hôpital’s Rule.)

105
Exercise 6.4.6. [TBB08, Exercise 7.6.15] Let f : [0, ∞) → R be a function
which is differentiable, such that f 0 (x) is decreasing and positive. By writing
N
P
f (N ) − f (0) as f (n) − f (n − 1), prove that the series
n=1


X
f 0 (n)
n=1

converges if and only if f is bounded.


[You may not use the Integral Test. In fact, this is essentially a restatement
of the Integral Test through the Fundamental Theorem of Calculus, but the
task here is to prove it by just using the Mean Value Theorem.]

106
Chapter 7

Riemann integration

In this chapter we define integration and study its properties. Integrating a


positive function can be thought of as measuring the area under its graph
(and above the x-axis). We will prove some important properties of the in-
tegral, including the Fundamental Theorem of Calculus (relating integration
to differentiation).

7.1 The Riemann integral


To formally define the integral, we proceed by estimating this area. Fix
a < b.
A partition of an interval [a, b] is a finite set {t0 , t1 , . . . , tn } such that

a = t0 < t1 < · · · < tn = b.1

A partition breaks up the interval [a, b] into n subintervals

[t0 , t1 ], [t1 , t2 ], . . . , [tn−1 , tn ].

Let P = {t0 , t1 , . . . , tn } be a partition and let f : [a, b] → R be a bounded


function. For i = 1, . . . , n, define

mi (P, f ) := inf f ([ti−1 , ti ]) = inf{f (t) : t ∈ [ti−1 , ti ]},


Mi (P, f ) := sup f ([ti−1 , ti ]) = sup{f (t) : t ∈ [ti−1 , ti ]}.
1
Of course, sets are not ordered: {t0 , t1 , t2 , t3 } = {t1 , t3 , t0 , t2 }. However, when we
write out a partition this way as {t0 , t1 , . . . , tn }, we will understand that the elements are
in order (unless otherwise noted).

107
Definition 7.1.1. Let P = {t0 , t1 , . . . , tn } be a partition and let f : [a, b] → R
be a bounded function. The lower Darboux sum of f for P is
n
X
L(P, f ) := mi (P, f )(ti − ti−1 ).
i=1

The upper Darboux sum of f for P is


n
X
U (P, f ) := Mi (P, f )(ti − ti−1 ).
i=1

The idea is the following. Suppose that f is a positive function. We


start with a partition P , which corresponds to a way of breaking up [a, b]
into intervals [t0 , t1 ], [t1 , t2 ], . . . , [tn−1 , tn ]. If we then ask for the smallest
rectangles which have these intervals on one side, and which cover the graph
of f , then the rectangles we will come up with are precisely

[t0 , t1 ] × [0, M1 (P, f )], . . . , [ti−1 , ti ] × [0, Mi (P, f )], . . . [tn−1 , tn ] × [0, Mn (P, f )].

The area of these rectangles is precisely the upper Darboux sum, and in
this sense, the upper Darboux sum gives an estimation from above of the
area under f . Likewise, the lower Darboux sum corresponds to the largest
rectangles that can be fit under the graph of f , so the lower Darboux sum
gives an estimation from below of the area under f .
Example 7.1.2. Let f : [0, 1] → R be given by f (t) := t, and let P :=
t0 := 0, t1 := 31 , t2 := 1 . Then


1 1
m1 (P, f ) = 0, m2 (P, f ) = , M1 (P, f ) = , M2 (P, f ) = 1.
3 3
108
Hence,
   
1 1 1 2
L(P, f ) = m1 (P, f )(t1 −t0 )+m2 (P, f )(t2 −t1 ) = 0 −0 + 1− =
3 3 3 9

and
   
1 1 1 7
U (P, f ) = M1 (P, f )(t1 −t0 )+M2 (P, f )(t2 −t1 ) = − 0 +1 1 − = .
3 3 3 9

It is clear that mi (P, f ) ≤ Mi (P, f ) always holds, and therefore,

L(P, f ) ≤ U (P, f )

for any partition P .

Definition 7.1.3. Let P, P 0 be partitions. We say that P 0 refines P if P ⊆ P 0


(as sets).

Lemma 7.1.4. Let f : [a, b] → R be a bounded function and let P, P 0 be


partitions of [a, b] such that P 0 refines P . Then

L(P, f ) ≤ L(P 0 , f ) and U (P 0 , f ) ≤ U (P, f ).

Proof. Since P refines P 0 , it is obtained from P by adding finitely many


points. We will prove that if P 0 contains exactly one more point than P ,
then
L(P, f ) ≤ L(P 0 , f ) and U (P 0 , f ) ≤ U (P, f ).
The general case will then follow by induction, using this special case as the
inductive step.
We will only prove L(P, f ) ≤ L(P 0 , f ), as the case for upper sums is
similar.
Let P = {t0 , t1 , . . . , tn } and let P 0 = {t0 , . . . , tk−1 , s, tk , . . . , tn }, with

a = t0 < t1 < · · · < tk−1 < s < tk < · · · < tn = b.

Let us compare

L(P, f ) = m1 (P, f )(t1 −t0 )+· · ·+mk (P, f )(tk −tk−1 )+· · ·+mn (P, f )(tn −tn−1 )

109
with
L(P 0 , f ) = m1 (P 0 , f )(t1 − t0 ) + · · · + mk (P 0 , f )(s − tk−1 ) + mk+1 (P 0 , f )(tk − s)
+ · · · + mn+1 (P 0 , f )(tn − tn−1 ).
Note that for i < k, we have
mi (P, f ) = mi (P 0 , f ),
and likewise, for i > k,
mi (P, f ) = mi+1 (P 0 , f ).
Hence,
L(P 0 , f )−L(P, f ) = mk (P 0 , f )(s−tk−1 )+mk+1 (P 0 , f )(tk −s)−mk (P, f )(tk −tk−1 ).
Nett, observe that since f ([tk−1 , s]) ⊆ f ([tk−1 , tk ], we have
mk (P, f ) = inf f ([tk−1 , tk ]) ≤ inf f ([tk−1 , s]) = mk (P 0 , f ),
and likewise,
mk (P, f ) ≤ mk+1 (P 0 , f ).
Thus,
L(P 0 , f ) − L(P, f )
= mk (P 0 , f )(s − tk−1 ) + mk+1 (P 0 , f )(tk − s) − mk (P, f )(tk − tk−1 )
≥ mk (P, f )(s − tk−1 ) + mk (P, f )(tk − s) − mk (P, f )(tk − tk−1 )
= 0,
as required.
Corollary 7.1.5. Let f : [a, b] → R be a bounded function and let P, P 0 be
partitions of [a, b]. Then
L(P, f ) ≤ U (P 0 , f ).
Proof. Let P 00 = P ∪ P 0 (as sets), so that P 00 is a partition of [a, b] which
refines both P and P 0 . Then
L(P, f ) ≤ L(P 00 , f ) (previous lemma)
≤ U (P 00 , f )
≤ U (P 0 , f ) (previous lemma).

110
Definition 7.1.6. A bounded function f : [a, b] → R is (Riemann) integrable
if

sup{L(P, f ) : P a partition of [a, b]} = inf{U (P, f ) : P a partition of [a, b]}.

In this case, we set Z b Z b


f (t) dt or f
a a

equal to this number, called the integral of f (on [a, b]).

In other words, Z b
f (t) dt = I
a
if we can approximate I both from above and from below arbitrarily well by
upper and lower Darboux sums (respectively).
If [a, b] ⊆ A ⊆ R and f : A → R is a function such that f |[a,b] is integrable,
then we continue to write Z b
f (t) dt
a

for the value of the integral of f |[a,b] . We also write


Z a Z b
f (t) dt = − f (t) dt.
b a

To shorten notation, we will sometimes write

sup L(P, f )
P

to mean
sup{L(P, f ) : P a partition of [a, b]},
and similarly
inf U (P, f )
P

for
inf{U (P, f ) : P a partition of [a, b]}.
Note that by Corollary 7.1.5, it follows that

111
sup L(P, f ) ≤ inf U (P, f ).
P P

Example
R 1 7.1.7. Let f : [0, 1] → R be defined by f (t) := t. This is integrable
1
and 0 f (t) dt = 2 .
To see this, consider the partition P = {0, n1 , n2 , . . . , nn = 1}. Then
   
i−1 i i−1 i i−1
mi (f, P ) = inf f , = inf , = ,
n n n n n
and so
n  
X i i−1
L(f, P ) = mi (f, P ) −
i=1
n n
n
X i−1 1
= ·
i=1
n n
n
1 X
= (i − 1)
n2 i=1
n(n − 1) n−1
= = .
2n2 2n
i
Similarly, Mi (f, P ) = n
and so
n  
X i i−1
U (f, P ) = Mi (f, P ) −
i=1
n n
n
X i 1
= ·
i=1
n n
n
1 X
= 2 i
n i=1
n(n + 1) n+1
= 2
= .
2n 2n
It follows that
 
n−1 1
sup L(P, f ) ≥ sup : n ∈ N≥1 = ,
P 2n 2

112
and on the other hand,
 
n+1 1
inf U (P, f ) ≤ inf : n ∈ N≥1 = .
P 2n 2
Therefore, these must both equal 12 , which means that
Z 1
1
f (t) dt = .
0 2
Example 7.1.8. Consider the function f : [0, 1] → R defined by
(
0, t ∈ Q;
f (t) :=
1, t∈ 6 Q.

For any partition P , we will show that


L(P, f ) = 0 and U (P, f ) = 1.
Consequently,
sup L(P, f ) = 0 6= 1 = inf U (P, f ),
P P

so that f is not integrable.


Let P = {t0 , . . . , tn } be a partition. Then for each i, the set [ti−1 , ti ]
contains both rational and irrational numbers. Hence,
f ([ti−1 , ti ]) = {0, 1},
so that mi (P, f ) = 0 and Mi (P, f ) = 1. From this we get
n
X
U (P, f ) = Mi (P, f )(ti − ti−1 )
i=1
n
X
= (ti − ti−1 )
i=1
= tn − t0 (by telescoping)
= 1.
On the other hand,
n
X n
X
L(P, f ) = mi (P, f )(ti − ti−1 ) = 0(ti − ti−1 ) = 0.
i=1 i=1

113
Example 7.1.9. Consider the function f : [0, 2] → R defined by
(
0, t = 1;
f (t) :=
1, t=6 1.
R1
This is integrable and 0 f (t) dx = 2.
To see this, given  > 0, consider the partition

P := {t0 = 0, t1 = 1 − , t2 = 1 + , t3 = 2}.

We compute
m1 (P, f ) = 1, m2 (P, f ) = 0, m3 (P, f ) = 1,
and therefore
3
X
L(f, P ) = mi (f, P )(ti − ti−1 )
i=1
= 1(1 −  − 0) + 0(1 +  − (1 − )) + 1(2 − (1 + ))
= 1 −  + 1 −  = 2 − 2.

Therefore,
sup L(f, P ) ≥ 2 − 2,
P

and since  > 0 is arbitrary,

sup L(f, P ) ≥ 2.
P

On the other hand, taking the “trivial” partition P := {0, 2}, we have
M1 (f, P ) = 1, and so

U (f, P ) = M1 (f, P )(2 − 0) = 2,

and therefore
inf U (f, P ) ≤ 2.
P

Hence, supP L(f, P ) = inf P U (f, P ) = 2, so f is integrable and


Z 1
f (t) dt = 2.
0

114
Here is a way of rephrasing integrability.

Proposition 7.1.10. Let f : [a, b] → R be a bounded function. Then f is


integrable if and only if, for every  > 0 there exists a partition P such that

U (P, f ) − L(P, f ) < .

Proof. ⇒: Assume that f is integrable and let  > 0. Set

I := sup L(P, f ) = inf U (P, f ).


P P

It follows that there are partitions PL and PU such that


 
L(PL , f ) > I − , U (PU , f ) < I + .
2 2
Let P = PL ∪ PU , so by Lemma 7.1.4,

L(P, f ) ≥ L(PL , f ) and U (P, f ) ≤ U (PU , f ).

Therefore,
  
U (P, f ) − L(P, f ) ≤ U (PU , f ) − L(PL , f ) < I − − I+ = .
2 2
⇐: For any  > 0 there is a partition P such that U (P, f ) − L(P, f ) < .

0 ≤ inf U (Q, f ) − sup L(Q, f ) ≤ U (P, f ) − L(P, f ) < .


Q Q

Since  > 0 is arbitrary, it follows that inf Q U (Q, f ) = supQ L(Q, f ), so that
f is integrable.

Theorem 7.1.11. If f : [a, b] → R is continuous then f is integrable.

Proof. Let  > 0 be given. Since f is continuous and [a, b] is compact, by


Theorem 5.5.5, there exists δ > 0 such that if x, y ∈ [a, b] and |x − y| < δ

then |f (x) − f (y)| < b−a .
Let P = {t0 , . . . , tn } be a partition such that |ti − ti−1 | < δ for all i.
By Proposition 5.3.6, f ([ti−1 , ti ]) is a closed interval, and then by the uni-
form continuity condition, we see that its maximum and minimum cannot

115
be separated by more than /(b − a).2 In other words,

Mi (P, f ) − mi (P, f ) < .
b−a
Consequently,
n
X
U (P, f ) − L(P, f ) = (Mi (P, f ) − mi (P, f ))(ti − ti−1 )
i=1
n
X 
< (ti − ti−1 )
i=1
b−a

= (tn − t0 ) = .
b−a
This verifies the condition in the previous proposition, so we conclude that
f is integrable.
Exercise 7.1.1. Define f : [−1, 1] → R by f (x) := x2

(i) Let P := −1, − 21 , 13 , 1 . Compute L(P, f ) and U (P, f ).




(ii) Find a partition P such that U (P, f ) − L(P, f ) < 21 .

Exercise 7.1.2. [Leb16, Exercise 5.1.5] Define f : [−1, 1] → R by


(
1, x > 0;
f (x) :=
0, x ≤ 1.
R1
Prove that f is integrable and compute −1
f (t) dt.
Exercise 7.1.3. Let f : [0, 1] → R be an increasing function.

(a) Prove that for any partition P and any i, Mi (P, f ) = mi+1 (P, f ).

(b) For n ∈ N≥1 , set Pn := {0, n1 , n2 , . . . , nn }. Find a short expression for


U (Pn , f ) − L(Pn , f ).
2
In fact, we don’t really need to use Proposition 5.3.6 here:

Mi (P, f )−mi (P, f ) = sup f ([ti−1 , ti ])−inf f ([ti−1 , ti ]) = sup{f (x)−f (y) : x, y ∈ [ti−1 , ti ]} ≤ .
b−a

116
(c) Prove that f is Riemann integrable.
Exercise 7.1.4. Recall the Dirichlet function from Exercise 5.2.4:
(
0, if x 6∈ Q;
f (x) := 1
q
, if x = pq in lowest terms (with p ∈ Z and q ∈ N≥1 ).

Let  > 0.
(a) Prove that for any partition P of [0, 1],

L(P, f ) = 0.

(b) Prove that there is a number N such that for any partition P of [0, 1],
the set
{i : Mi (P, f ) ≥ }
has size at most N .

(c) Let N be as in part (b). Suppose that P = {x0 , x1 , . . . , xn } is a partition


of [0, 1], and let

δ := max{x1 − x0 , x2 − x1 , . . . , xn − xn−1 }.

Prove that
U (f, P ) ≤  + N δ.

(d) Prove that there is a partition P such that

U (f, P ) < 2.


R1
(e) Prove that f is integrable and compute 0
f (t) dt.
Exercise 7.1.5. [Leb16, Exercise 5.1.7] Let f : [a, b] → R be Riemann inte-
grable. Prove that given  > 0 there is a partition P = {t0 , . . . , tn } of [a, b]
such that, for any ξ1 ∈ [t0 , t1 ], . . . , ξn ∈ [tn−1 , tn ],
Z
b X n
f (t) dt − f (ξi )(ti − ti−1 ) < .


a
i=1

n
P
[The sum f (ξi )(ti − ti−1 ) is called a Riemann sum for f .]
i=1

117
Exercise 7.1.6. [Leb16, Exercise 5.1.8] Let α > 0, β ∈ R. Suppose that
f : [β, α + β] → R is Riemann integrable. Define g : [0, 1] → R by

g(x) := f (αx + β).


R1 R α+β
Prove that g is Riemann integrable and compute 0
g in terms of β
f.
Exercise 7.1.7. Let f : [a, b] → R be a bounded function and let P be a
partition of [a, b]. Show that for c > 0,

U (P, cf ) = cU (P, f ) and L(P, cf ) = cL(P, f ).

What happens if c < 0?

7.2 Properties of the integral


Proposition 7.2.1 (Additive Property). Let f : [a, b] → R be a bounded
function and let c ∈ (a, b). Then f is integrable if and only if f |[a,c] and f |[c,b]
are both integrable. In this case,
Z b Z c Z b
f (t) dt = f (t) dt + f (t) dt.
a a c

Proof. ⇒: Let  > 0 be given. There exists a partition P of [a, b] such


that U (P, f ) − L(P, f ) < . By possibly adding a point (which leads to a
refinement of the original P ), we may assume that c ∈ P . Break up the
partition P into two partitions corresponding to [a, c] and [c, b]: define

P1 := P ∩ [a, c] and P2 := P ∩ [c, b].

Then one sees that

U (P, f ) = U (P1 , f |[a,c] ) + U (P2 , f |[c,b] )

and likewise
L(P, f ) = L(P1 , f |[a,c] ) + L(P2 , f |[c,b] ).
Since U (P2 , f |[c,b] ) − L(P2 , f |[c,b] ) ≥ 0, it follows that

U (P1 , f |[a,c] ) − L(P1 , f |[a,c] ) ≤ U (P, f ) − L(P, f ) < .

118
Likewise,

U (P2 , f |[c,b] ) − L(P2 , f |[c,b] ) ≤ U (P, f ) − L(P, f ) < .

Since  is arbitrary, we see that both f |[a,c] and f |[c,b] are integrable.
Rb Rc Rb
⇐ and showing a f = a f + c f are Exercise 7.2.1
Proposition 7.2.2 (Linearity of the integral). Let f, g : [a, b] → R be
bounded integrable functions and let c ∈ R. Then cf + g is integrable, and
Z b Z b Z b
cf (t) + g(t) dt = c f (t) dt + g(t) dt.
a a a

Proof. The case c = 0 is trivial. For c > 0, it is an exercise (Exercise 7.2.2)


to show that

mi (P, cf +g) ≥ cmi (P, f )+mi (P, g) and Mi (P, cf +g) ≤ cMi (P, f )+Mi (P, g).

It follows that

L(P, cf + g) ≥ cL(P, f ) + L(P, g) and U (P, cf + g) ≤ cU (P, f ) + U (P, g).

Consequently,

sup L(P, cf + g) ≥ sup cL(P, f ) + L(P, g)


P P
≥ c sup L(P, f ) + sup L(P, g)
P P
Z b Z b
=c f+ g.
a a

and likewise Z b Z b
inf U (P, cf + g) ≤ c f+ g.
P a a
Rb Rb Rb
It follows that cf + g is integrable with a cf + g = c a f + a g.
For the case c < 0, it suffices (by combining with the case we just did) to
Rb Rb
show that −f is integrable and a −f = − a f . This is Exercise 7.2.3.
Proposition 7.2.3. Let f, g : [a, b] → R be integrable. If f (t) ≤ g(t) for all
t ∈ [a, b] then
Z b Z b
f (t) dt ≤ g(t) dt.
a a

119
Proof. Let P be a partition. Then it is straightforward to see that mi (P, f ) ≤
mi (P, g) for all i, and thus

L(P, f ) ≤ L(P, g).

Therefore, Z b Z b
f = sup L(P, f ) ≤ sup L(P, g) = g.
a P P a

Corollary 7.2.4. Let f : [a, b] → R be integrable. If m, M ∈ R are such that

m ≤ f (t) ≤ M

for all x ∈ [a, b] then


Z b
m(b − a) ≤ f (t) dt ≤ M (b − a).
a

Exercise 7.2.1. Prove the remaining part of Proposition 7.2.1: that if f :


[a, b] → R is a bounded function such that f |[a,c] and f |[c,b] is integrable then
f is integrable and
Z b Z c Z b
f (t) dt = f (t) dt + f (t) dt.
a a c

Exercise 7.2.2. Let f, g : [a, b] → R be bounded functions, let P be a partition


of [a, b], and let c > 0.
(a) Show that mi (P, f + g) ≥ mi (P, f ) + mi (P, g) and Mi (P, f + g) ≤
Mi (P, f ) + Mi (P, g) for all i.
(b) Show that mi (P, cf ) = cmi (P, f ) and Mi (P, cf ) = cMi (P, f ).
(c) Show that mi (P, cf + g) ≥ cmi (P, f ) + mi (P, g) and Mi (P, cf + g) ≤
cMi (P, f ) + Mi (P, g).
(d) Show that mi (P, −f ) = −Mi (P, f ).
Exercise 7.2.3. Let f : [a, b] → R be an integrable function. Prove that −f
is integrable and Z b Z b
−f (t) dt = − f (t) dt.
a a

120
Exercise 7.2.4. Let f : [a, b] → R be a continuous function such that f (t) ≥ 0
Rb
for all t ∈ [a, b]. Suppose that a f (t) dt = 0. Prove that f (t) = 0 for all
t ∈ [a, b].
[Hint. If f (x) > 0 then use continuity to obtain γ and c < d such that
f (t) ≥ γ for all t ∈ [c, d].]
Rb
Exercise 7.2.5. Let f : [a, b] → R be continuous. Prove that if a f (t)g(t) dt =
0 for every continuous g : [a, b] → R, then f (t) = 0 for all t ∈ [a, b].
[Hint. Find a way to use the previous exercise.]
Exercise 7.2.6. Let f : [a, b] → R be an integrable function (hence bounded).
Define F : [a, b] → R by
Z x
F (x) := f (t) dt.
a

Prove that F is Lipschitz (see Exercise 5.5.7) and therefore (uniformly) con-
tinuous.
Exercise 7.2.7. Let f : [a, b] → R be an integrable function. Prove that |f |
is integrable and that
Z b Z b

f (t) dt ≤ |f (t)| dt.

a a

7.3 The Fundamental Theorem of Calculus


Theorem 7.3.1 (Fundamental Theorem of Calculus). Let f : [a, b] → R be
an integrable function. Define F : [a, b] → R by
Z x
F (x) := f (t) dt.
a

For any x ∈ [a, b], if f is continuous at x then F is differentiable at x and

F 0 (x) = f (x).

Proof. Fix x ∈ [a, b] such that f is continuous at x. We will show that (if
x < b) lim+ F (y)−F
y−x
(x)
= f (x) using the δ- definition of limit (with ≤ in place
y→x

121
F (y)−F (x)
of <). A similar argument will show that (if x > a) lim− y−x
= f (x),
y→x
and thus by Exercise 5.1.9,

F (y) − F (x)
F 0 (x) = lim = f (x).
y→x y−x
Ry
Note that by the Additive Property, F (y) − F (x) = x f .
Given  > 0, choose δ > 0 such that if t ∈ [a, b] and |t − x| ≤ δ then
|f (t) − f (x)| ≤ . Let y ∈ (x, x + δ]. Then for t ∈ [x, y], we have

f (x) −  ≤ f (t) ≤ f (x) + .

Hence by Corollary 7.2.4 (applied to f |[x,y] ),


Z y
(f (x) − )(y − x) ≤ f ≤ (f (x) + )(y − x),
x

so that
F (y) − F (x)
f (x) −  ≤ ≤ f (x) + .
y−x
F (y)−F (x)
This proves that lim+ y−x
= f (x), as required.
y→x

The following is the form in which the Fundamental Theorem of Calculus


is typically used: it enables computing integrals via antidifferentiation.

Theorem 7.3.2. Let F : [a, b] → R be a differentiable function, such that


F 0 : [a, b] → R is continuous. Then
Z b
F 0 (t) dt = F (b) − F (a).
a

Proof. Define G : [a, b] → R by


Z x
G(x) := F 0 (t) dt.
a

Then by the Fundamental Theorem of Calculus, G0 = F 0 . It follows from


Exercise 6.4.3 that G − F is a constant function, i.e.,

F (x) = G(x) + C

122
for some C ∈ R. Using x = a, we find
F (a) = G(a) + C = 0 + C,
so that C = F (a). Hence,
Z b
F (b) − F (a) = G(b) = F 0,
a

as required.
Interesting fact 7.3.3. The above theorem can be generalized somewhat: in-
stead of assuming that F 0 is continuous, it suffices to assume that F 0 exists
(i.e., F is differentiable). The proof for this case is a bit longer (and can’t be
reduced to Theorem 7.3.1).
Exercise 7.3.1. [Sav17, Exercise 7.3.1] Let f : [a, b] → R be continuous and
define F : [a, b] → R by
Z b
F (x) := f (t) dt.
x
Prove that F is differentiable and F 0 (x) = −f (x) for all x ∈ [a, b].
Exercise 7.3.2. [Leb16, Exercise 5.3.2] Define G : R → R by G(x) :=
R x2
0
sin(t2 ) dt. Compute G0 (x).
Exercise 7.3.3. [Sav17, Exercise 7.3.6] Let f : [a, b] → R be continuous, where
Rx Rb
a < b. Prove that if a f (t) dt = x f (t) dt for all x ∈ [a, b], then f (t) = 0 for
all t ∈ [a, b].
Exercise 7.3.4. Prove the Integration by Parts formula: if f, g : [a, b] → R
are differentiable functions for which f 0 , g 0 : [a, b] → R are continuous, then
Z b Z b
0
f (t)g (t) dt = f (b)g(b) − f (a)g(a) − f 0 (t)g(t) dt.
a a

Exercise 7.3.5. Prove the Change of Variables formula: if f : [c, d] → R is a


continuous function and g : [a, b] → R is a differentiable function such that
g([a, b]) ⊆ [c, d] and g 0 : [a, b] → R is continuous, then
Z b Z g(b)
0
f (g(t))g (t) dt = f (t) dt.
a g(a)
Rx R g(x)
[Hint. Define F (t) := a f (g(t))g 0 (t) dt and G(t) := g(a)
f (t) dt. Use FTC
and the Chain Rule to compute F 0 and G0 .]

123
7.4 Improper integrals
So far we have only defined integration for bounded functions on bounded
intervals. It is desirable to extend the theory to unbounded functions and/or
unbounded intervals. The approach of approximating by Darboux sums,
however, will not work: if f is not bounded above then for every partition
P , Mi (P, f ) will be infinite for some i, and thus

U (P, f ) = ∞.

Instead, to deal with this case, we use limits.

Definition 7.4.1. Let f : (a, b] → R be a function such that, for every


x ∈ (a, b], f |[x,b] is Riemann integrable. Then we define
Z b Z b
f (t) dt := lim+ f (t) dt,
a x→a x

provided that this limit exists. Likewise, if f : [a, b) → R is such that f |[a,x]
is integrable for all x ∈ [a, b), then
Z b Z x
f (t) dt := lim− f (t) dt,
a x→b a

provided that this limit exists.


Let f : [a, ∞) → R be a function such that, for every x ∈ [a, ∞), f |[a,x] is
integrable. Then we define
Z ∞ Z x
f (t) dt := lim f (t) dt.
a x→∞ a

Likewise, if f : (−∞, b] → R is a function such that f |[x,b] is integrable for


all x ∈ (−∞, b] then
Z b Z b
f (t) dt := lim f (t) dt.
−∞ x→−∞ x

Whenever one of the above limits exist, we say that the improper integral
converges.

124
Note that if f : [a, b] → R is integrable then by Exercise 7.2.6,
Z b Z x
f (t) dt = lim− f (t) dt
a x→b a
Rb Rb
(and similarly, one can prove a
f (t) dt = lim+ x
f (t) dt), so there is no
x→a
ambiguity in the above notation.
The above definition allows integrals to be defined where there is un-
boundedness at one endpoint. To allow unboundedness at both endpoints,
we cut in the middle and combine. For example, suppose f : (a, ∞) → R is
a function for which f |[x,y] is Riemann integrable for all a < x < y. Then we
choose some c ∈ (a, ∞) and define
Z ∞ Z c Z ∞
f (t) dt := f (t) dt + f (t) dt,
a a c

provided both of these exist (using the above definition).


Exercise
R∞ 1 7.4.1. [TBB08, Exercise 8.5.3] For what values of p is the integral
1 tp
dt convergent? (You may use the Fundamental Theorem of Calculus.)
R ∞ Exercise 7.4.5] Suppose that f : [0, ∞) → R is a
Exercise 7.4.2. [Sav17,
function such that 0 f (t) dt converges. Prove that for every  > 0 there
exists M such that if a > b > M then
Z b


f (t) dt < .

a

Exercise 7.4.3. Define f : R → R by f (t) := t. Show that the limit


Z x
lim f (t) dt
x→∞ −x
R∞
exists, but explain why −∞
f (t) dt doesn’t converge.
Exercise 7.4.4. In this question, we prove the Integral Test (Proposition
3.2.10). Let f : [1, ∞) → R be an decreasing function.
(i) Prove that for all n ∈ N≥1 ,
N
X Z N +1 N
X
f (n) ≤ f (t) dt ≤ f (n).
n=2 1 n=1

125
R∞ ∞
P
(ii) Prove that if 1
f (t) dt converges, then f (n) converges.
n=1

(iii) Prove that if F : [1, ∞) → R is a bounded increasing function then


lim F (x) = sup F ([0, ∞).
x→∞


P R∞
(iv) Using (c), prove that if f (n) converges then 1
f (t) dt converges.
n=1

126
Chapter 8

Sequences and series of


functions

Our aim in this chapter is to make sense of and analyze infinite sums of

fn , where fn : X → Rm . Inherent in trying to define such a
P
functions:
n=1
thing is a notion of limit, and therefore it makes sense to start with sequences
of functions and develop the notion of a limit in that setting. From there,
we define convergence of series of functions in terms of partial sums.

8.1 Pointwise limits


The most obvious way to define a limit of functions is the pointwise limit.
We will illustrate with examples that, unfortunately, pointwise limits do not
behave well with respect to calculus operations.

Definition 8.1.1. Let X be a set, let (fn : X → Rm )∞ n=1 be a sequence of


m ∞
functions, and let f : X → R . We say that (fn )n=1 converges pointwise to
f if for every x ∈ X,
lim fn (x) = f (x).
n→∞

In this case, we may write

pw− lim fn = f.
n→∞

127
Example 8.1.2. Define fn : [0, 1] → R by fn (x) := xn . Then for x ∈ [0, 1],
(
0, x ∈ [0, 1);
lim fn (x) = lim xn =
n→∞ n→∞ 1, x = 1.

Hence, (fn )∞
n=1 converges pointwise to f : [0, 1] → R defined by
(
0, x ∈ [0, 1);
f (x) :=
1, x = 1.

Note that each fn is continuous, but the pointwise limit f is not.


xn
Example 8.1.3. Define fn : [0, 1] → R by fn (x) := n
. For x ∈ [0, 1], we can
see (using the Squeeze Theorem) that

lim fn (x) = 0,
n→∞

so that (fn )∞n=1 converges pointwise to the constant zero function (which
we’ll write as g here). How do the derivatives behave? fn0 is differentiable
and fn0 (x) = xn−1 . Hence, (fn0 )∞
n=1 converges pointwise to the function f from
the previous example – which is not the same as g 0 (which is also the zero
function). This shows that
 0
pw− lim fn 6= pw− lim fn0 .
n→∞ n→∞

Example 8.1.4. Define fn : [0, 1] → R such that


   
1 2
fn (0) = 0, fn = n, fn = 0, fn (1) = 0,
n n
and fn is linear in between.

128
An explicit formula for fn is

2
n x,
 x ∈ [0, n1 ];
fn (x) := 2n − n2 x, x ∈ [ n1 , n2 ];

0, x ∈ [ n2 , 1].

Claim. The sequence (fn )∞ n=1 converges pointwise to the zero function g.
To see this, note that for x = 0, we have fn (x) = 0 for all n, and thus
lim fn (0) = 0. On the other hand, for x > 0, we have fn (x) = 0 provided
n→∞
n > x2 , and thus (fn (x))∞
n=1 is eventually zero, which implies its limit is 0.
R1
However, to compute 0 fn (t) dt, note that this gives the area of a triangle
with base length n2 and height n, so
Z 1
fn (t) dt = 1.
0

The integral of g = pw− lim fn , however, is 0. This shows that


n→∞
Z 1 Z 1
lim fn (t) dt 6= lim fn (t) dt.
0 n→∞ n→∞ 0

Exercise 8.1.1. Give an example of a sequence of differentiable functions


(fn : R → R)∞ n=1 which converge pointwise to the function f (x) := |x|, which
is not differentiable at 0.
Exercise 8.1.2. (cf. [TBB08, Exercise 9.2.1]) Define fn : R → R by fn (x) :=
x2n
1+x2n
. Determine the pointwise limit of the sequence (fn )∞
n=1 .
Exercise 8.1.3. [TBB08, Exercise 9.2.3] There is a sequence (xn )∞n=1 in [0, 1]
such that
{xn : n ∈ N≥1 } = Q ∩ [0, 1].
(In other words, Q ∩ [0, 1] is countable.) Define fn : [0, 1] → R by
(
x, x ∈ {x1 , . . . , xn };
fn (x) :=
0, otherwise,
and (
x, x ∈ Q;
f (x) :=
0, otherwise,
Show that fn converges pointwise to f , and that each fn is integrable. (From
Example 7.1.8, we know that f is not integrable.)

129
Exercise 8.1.4. [TBB08, Exercise 9.2.7] Let (fn : [a, b] → R)∞
n=1 be a sequence
which converges pointwise to f : [a, b] → R. Which of the following are true:

(a) If each fn is (weakly) increasing then so is f .

(b) If each fn is strictly increasing then so is f .

(c) If each fn is bounded then so is f .

(d) If each fn is constant then so is f .

(e) If fn (x) ≥ 0 for all x ∈ [a, b] then f (x) ≥ 0 for all x ∈ [a, b].

(f) If each fn is linear (meaning, of the form fn (x) = αn x + βn ) then so is f .

(g) If each fn is convex (meaning fn (tx + (1 − t)y) ≤ tfn (x) + (1 − t)fn (y)
for all x, y ∈ [a, b] and all t ∈ [0, 1]) then so is f .

8.2 Uniform convergence


In this section, we define a stronger notion of convergence of a sequence of
functions. Using this notion, we can overcome the bad behaviour with mere
pointwise convergence, seen in the last section.
To contextualize the definition of uniform convergence, we unwind the
definition of “a sequence (fn : X → Rm )∞n=1 converges pointwise to f : X →
Rm ” (using the δ- definition of convergence in Rm ), we get:

For any x ∈ X and  > 0 there exists n0 ∈ N≥1 such that for all n ≥ n0
then kfn (x) − f (x)k2 < .

Here, n0 is allowed to depend on both x and  (but not, of course, on


n). The idea of “uniformizing” this definition which we will do presently is
similar to what we did when we defined uniform continuity (Section 5.5): we
modify the definition so that n0 is not allowed to depend on x.

Definition 8.2.1. Let X be a set, let (fn : X → Rm )∞ n=1 be a sequence of


functions, and let f : X → Rm . We say that (fn )∞ n=1 converges uniformly to
f if for every  > 0 there exists n0 such that for all n ≥ n0 and x ∈ X,

kfn (x) − f (x)k2 < .

130
In this case, we may write

u− lim fn = f.
n→∞

By comparing this definition to the characterization of pointwise conver-


gence above, it is clear that

u− lim fn = f implies pw− lim fn = f.


n→∞ n→∞

We also define uniform convergence of a series of functions.

Definition 8.2.2. Let X be a set, let (fn : X → Rm )∞


n=1 be a sequence of
functions, and let f : X → Rm . We define

X∞ N
X
u− fn := u− lim fn ,
N →∞
n=1 n=1

provided that this limit exists. When this limit exists, we say that the series
P∞
fn converges uniformly (on X).
n=1

sin(nx)
Example 8.2.3. Define fn : R → R by fn (x) := n
. Then u− lim fn = 0
n→∞
(the zero function).
To see this, let  > 0 be given. Then pick n0 > 1 . If n ≥ n0 and x ∈ R
then
| sin(nx)| 1
|fn (x) − 0| = ≤ < .
n n
Example 8.2.4. Let b ∈ (0, 1) and define fn : [0, b] → R by fn (x) := xn . Then
u− lim fn = 0 (the zero function).
n→∞
To see this, let  > 0 be given. Since lim bn = 0, there exists n0 such
n→∞
that if n ≥ n0 then |bn | < . Now, if n ≥ n0 and x ∈ [0, b] then x ≤ b so that
xn ≤ bn , and thus
|fn (x) − 0| = xn < .
Example 8.2.5. The sequence of functions (fn : [0, 1] → R)∞ n=1 defined by
n
fn (x) := x does not converge uniformly to the function g in Example 8.1.2.
This will follow from Theorem 8.3.1 below: if u−lim fn = g then this theorem
n→∞
would tell us that g is continuous, but g is not.

131
In practice, we will (more) often be interested in whether series of func-
tions converge uniformly, and for this, there is the following useful test.

Theorem 8.2.6 (The Weierstrass M -test). Let X be a set, let (fn : X →


R)∞ ∞
n=1 be a sequence of functions, and let (Mn )n=1 be a sequence of non-
negative real numbers. Suppose that the following hold:

(i) |fn (x)| ≤ Mn for all x ∈ X, and



P
(ii) Mn converges.
n=1


P
Then fn converges uniformly.
n=1

Proof. For each x ∈ X, by the Comparison Test, the series



X
|fn (x)|
n=1


P
converges, so that fn (x) converges absolutely. Define
n=1


X
g(x) := fn (x).
n=1


P
We will show that g = u− fn .
n=1

P
Let  > 0. Since Mn converges, we can find N0 such that
n=1

X∞ N0
X
Mn − Mn < .



n=1 n=1

132
Then if N ≥ N0 and x ∈ X,

XN X ∞ XN
g(x) − fn (x) = fn (x) − fn (x)


n=1
n=1 n=1
X ∞
= fn (x)


n=N +1

X
≤ |fn (x)|
n=N +1
X∞
≤ Mn
n=N +1

< ,

as required.
Exercise 8.2.1. (cf. [TBB08, Exercise 9.3.1]) For n ∈ N≥1 , define fn : R → R
by
x2n
fn (x) := lim .
n→∞ 1 + x2n

Prove that (fn |[b,∞) )∞n=1 converges uniformly, for any (fixed) b > 1, and that

(fn |[−c,c] )n=1 converges uniformly, for any (fixed) c ∈ (0, 1).
Exercise 8.2.2. [TBB08, Exercise 9.3.3] Let X be a finite set and suppose
that (fn : X → Rm )∞
n=1 is a sequence of functions which converge pointwise
m
to f : X → R . Prove that f = u− lim fn .
n→∞
Exercise 8.2.3. For a function f : X → R, define

kf k∞ := sup{|f (x)| : x ∈ X} ∈ [0, ∞].

Prove that a sequence (fn : X → R)∞n=1 converges uniformly to f : X → R if


and only if
lim kfn − f k∞ = 0.
n→∞

Exercise 8.2.4. Let X be a set and let (fn : X → R)∞ ∞


n=1 and (gn : X → R)n=1
be sequences which converge uniformly.

(a) [TBB08, Exercise 9.3.9] Prove that (fn + gn )∞


n=1 converges uniformly.

133
(b) [TBB08, Exercise 9.3.10] Must the sequence (fn gn )∞
n=1 converge uni-
formly? Give a proof or counterexample.
Exercise 8.2.5. [TBB08, Exercise 9.3.4] Let X = X1 ∪ X2 , let (fn : X →
Rd )∞ d
n=1 be a sequence of functions, and let f : X → R . Prove that if
f |X1 = u− lim fn |X1 and f |X2 = u− lim fn |X1 then
n→∞ n→∞

f = u− lim fn .
n→∞

Exercise 8.2.6. Show that the series



X sin(nx)
n=1
2n
converges uniformly on R.
Exercise 8.2.7. Let (fn : X → Rm )∞
n=1 be a sequence of functions. Prove that
the sequence converges uniformly if and only if it is “uniformly Cauchy”,
which means: for every  > 0 there exists n0 such that for all n, n0 ≥ n0 ,
kfn (x) − fm (x)k2 < .
[Hint. Use the Cauchy Convergence Criterion for Rm to first prove that
g = pw− lim fn exists. Then prove that g = u− lim fn by a similar proof to
n→∞ n→∞
the Cauchy Convergence Criterion.]
Exercise 8.2.8. (cf. [TBB08, Exercise 9.3.31]) Let X be a set and (fn : X →
R m )∞ ∞
n=1 is a sequence of functions. Suppose that for every sequence (xn )n=1
in X, we have
lim fn (xn ) = 0.
n→∞
Prove that
u− lim fn = 0 (the zero function).
n→∞
Exercise 8.2.9. Let f : R → R be a function and define a sequence of func-
tions (fn : R → R)∞n=1 by
1
fn (x) := f (x + ).
n
(a) Prove that if f is continuous then f = pw− lim fn .
n→∞

(b) Prove that if f is uniformly continuous then f = u− lim fn .


n→∞

(c) Give an example of a function f such that (fn )∞


n=1 doesn’t even converge
pointwise to f .

134
8.3 Properties of uniform convergence
Theorem 8.3.1. Let X ⊆ Rd be a set and a ∈ X. Let (fn : X → Rm )∞ n=1 a
sequence of functions which converges uniformly to f : X → Rm . If each fn
is continuous at a then so is f .
Hence, if each fn is continuous on X then so is f .
Proof. Let  > 0. Since f = u− lim fn , there exists n0 such that for all
n→∞
n ≥ n0 and all x ∈ X,

kfn (x) − f (x)k2 < .
3
Fix any n ≥ n0 Next, since fn is continuous at a, there exists δ > 0 such
that for all x ∈ X, if kx − ak2 < δ then

kfn (x) − fn (a)k2 < .
3
Now let x ∈ X be a point satisfying kx − ak2 < δ. Then
kf (x) − f (a)k2 = kf (x) − fn (x) + fn (x) − fn (a) + fn (a) − f (a)k2
≤ kf (x) − fn (x)k2 + kfn (x) − fn (a)k2 + kfn (a) − f (a)k2
  
< + + = ,
3 3 3
as required.
Applying the above theorem to a sequence of partial sums, we get the
following.
Corollary 8.3.2. Let X ⊆ Rd and suppose that (fn : X → Rm )∞ n=1 is

P
a sequence of continuous functions. If fn converges uniformly then the
n=1

P
function fn is continuous.
n=1

Example 8.3.3. Consider the series



X xn
.
n=0
n!
bn
If we fix some interval [−b, b] and set Mn := n!
, then for every x ∈ [−b, b],
we have n
x
≤ Mn .
n!

135

Mn+1 P
Since Mn
→ 0, it follows by the Ratio Test that Mn converges, so by
n=1
Weierstrass M -test,

X xn
n=0
n!
converges uniformly on [−b, b]. It follows that this defines a continuous func-
∞ n
x
P
tion on [−b, b]. Since b is arbitrary, it follows that n!
is continuous on
n=0
R.
Theorem 8.3.4. Let (fn : [a, b] → R)∞ n=1 be a sequence of continuous func-
tions which converges uniformly to f : [a, b] → R. Then
Z b Z b
f (t) dt = lim fn (t) dt.
a n→∞ a

Remark 8.3.5. In this theorem, we know that each fn and therefore also f is
continuous, and hence they are all Riemann integrable. In Exercise 8.3.4, this
theorem is generalized to the case that each fn is just Riemann integrable
(where the main challenge is to show that f is also Riemann integrable).
Proof. Define Mn := sup{|fn (t) − f (t)| : t ∈ [a, b]}. This is called kfn − f k∞
in Exercise 8.2.3, and by that same exercise, we know that
Mn → 0 as n → ∞.
For each n we have
Z b Z b

0 ≤ f (t) dt − fn (t) dt
Z a b a


= f (t) − fn (t) dt
a
Z b
≤ |f (t) − fn (t)| dt (Exercise 7.2.7)
a
≤ Mn (a − b). (Corollary 7.2.4)
By the Squeeze Theorem, it follows that
Z b Z b

lim f (t) dt − fn (t) dt = 0,
n→∞ a a

as required.

136
Applying the above theorem to a sequence of partial sums, we get the
following.
Corollary 8.3.6. Let (fn : [a, b] → R)∞
n=1 be a sequence of continuous func-

P
tions. If the series fn converges uniformly then
n=1

∞ Z
X b ∞
Z bX
fn (t) dt = fn (t) dt.
n=1 a a n=1

Example 8.3.7. Let x ∈ [0, 1). By the Weierstrass M -test (with Mn = xn ),



tn converges uniformly on [0, x]. At each value of t, this is a
P
the series
n=0
geometric series, so that we know that

X 1
tn = .
n=0
1−t

It follows that
x ∞ x ∞ ∞
xn+1 xn
Z Z
1 X
n
X X
dt = t dt = = .
0 1−t n=1 0 n=0
n + 1 n=1 n
Rx 1
We also know, from the Fundamental Theorem of Calculus, that 0 1−t
=
ln(1 − x), so this gives us an interesting formula:

X xn
ln(1 − x) = , x ∈ [0, 1).
n=1
n

Essentially the same argument also proves this formula for x ∈ (−1, 0].
For derivatives, the requirement for uniform convergence is slightly more
subtle. In Example 8.1.3, we had a sequence (fn : [0, 1] → R)∞ n=1 converg-
0 ∞
ing pointwise to the zero function, although (fn )n=1 converges to a nonzero
function. Looking closely at this example, we see that in fact 0 = u− lim fn ,
n→∞
which tells us that uniform convergence of the fn is not enough to improve
the behaviour with respect to differentiation. Instead, what we need is that
the sequence (fn ) converges uniformly. We also ask that the sequence con-
sists of continuously differentiable functions; this isn’t really necessary (see
[TBB08, Theorem 9.37]).

137
Theorem 8.3.8. Let (fn : [a, b] → R)∞ n=1 be a sequence of differentiable
0
functions such that fn is continuous for each n. Suppose that the sequence
(fn0 )∞ ∞
n=1 converges uniformly to some function g : [a, b] → R and that (fn )n=1
converges pointwise to f : [a, b] → R. Then f is differentiable and f 0 = g.
Proof. For x ∈ [a, b], we have

f (x) = lim fn (x)


n→∞
Z x
= lim fn (a) + fn0 (t) dt (Theorem 7.3.2)
n→∞ a
Z x
= f (a) + lim fn0 (t) dt
n→∞ a
Z x
= f (a) + g(t) dt (Theorem 8.3.4).
a

Hence by the Fundamental Theorem of Calculus (Theorem 7.3.1), f 0 = g.


Using partial sums we get the following.
Corollary 8.3.9. Let (fn : [a, b] → R)∞ n=1 be a sequence of differentiable

functions such that fn0 is continuous for each n, and let f =
P
fn (converging
n=1

fn0
P
pointwise). If the series converges uniformly then
n=1


X
0
f = fn0 .
n=1


1
xn for x ∈ (−1, 1]. We
P
Example 8.3.10. We already know that 1−x
=
n=0
1
would like to differentiate both sides: on the left we get (1−x)2 . On the right,

P n−1
differentiating term-by-term would give nx – but we don’t know yet
n=0
whether (or where) this converges and whether it agrees with the derivative
on the left. ∞
nxn−1 converges uniformly on
P
Fix some b ∈ (0, 1). We’ll show that
n=0
[−b, b]. Indeed, setting Mn := nbn−1 , then for all x ∈ [−b, b] we have

|nxn−1 | ≤ Mn .

138

Mn+1 P
Moreover, Mn
→ b < 1, so by the Ratio Test, Mn converges. Hence by
n=0
the Weierstrass M -test,

X
nxn−1
n=0
converges uniformly.
We may now apply the above corollary to conclude that

1 X
2
= nxn−1 , for x ∈ [−b, b].
(1 − x) n=0

Since b is arbitrary, it follows that



1 X
= nxn−1 , for x ∈ (−1, 1).
(1 − x)2 n=0

Exercise 8.3.1. [TBB08, Exercise 9.4.1] Does there exist a sequence of dis-
continuous functions (fn : [a, b] → R)∞n=1 which converges uniformly to a
continuous function f : [a, b] → R?
Exercise 8.3.2. Let X ⊆ Rd and let (fn : X → Rd )∞ n=1 be a sequence of
uniformly continuous functions which converges uniformly to f : X → Rd .
Prove that f is uniformly continuous.
Exercise 8.3.3. [TBB08, Exercise 9.5.2] Prove that
Z πX∞ ∞
sin(nt) X 2
2
dt = .
0 n=1 n n=1
(2n − 1)3

Exercise 8.3.4. Let (fn : [a, b] → R)∞ n=1 be a sequence of functions which
converges uniformly to f : [a, b] → R.
(a) Let P be a partition of [a, b], fix n ∈ N≥1 , and let  > 0 be such that
 > kfn − f k∞ = sup{|fn (x) − f (x)| : x ∈ [a, b]}.
Prove that
mi (P, f ) > mi (P, fn ) − , Mi (P, f ) < Mi (P, fn ) + ,
for all i and
L(P, f ) > L(P, fn ) − , U (P, f ) < U (P, fn ) + .

139
(b) Prove that if each fn is Riemann integrable then so is f , and
Z b Z b
f (t) dt = lim fn (t) dt.
a n→∞ a

[Exercise 8.2.3 may be handy.]

Exercise 8.3.5. [TBB08, Exercise 9.5.3] Suppose (fn : [a, b] → R)∞ n=1 is a
sequence of continuous functions which converges uniformly to f : [a, b] → R.
For each n, define Fn : [a, b] → R by
Z x
Fn (x) := fn (t) dt,
a

and define F : [a, b] → R by


Z x
F (x) := f (t) dt.
a

Prove that (Fn )∞


n=1 converges uniformly to F .

Exercise 8.3.6. Using the previous exercise, prove that if (fn : [a, b] → R)∞n=1
is a sequence satisfying the hypotheses of Theorem 8.3.8, then (fn )∞ n=1 con-
verges uniformly.
Exercise 8.3.7. Define f : R → R by

x2 x 4 x6
f (x) := 1 + + + + ··· .
1! 2! 3!
(a) Verify that the series defining f converges.

(b) [TBB08, Exercise 9.6.2] Prove that

f 0 (x) = 2xf (x), x ∈ R.

(c) Find a function (defined using a series) g : R → R which satisfies

g 0 (x) = 3x2 g(x), x ∈ R.

140
Chapter 9

Power series

In this chapter we look at a particularly fruitful way of writing a function as


a series. The series we consider can be considered, in a sense, as polynomials
of (possibly) infinite degree. We will make use of the theory developed in
previous chapters to help us do calculus with these series.

9.1 Convergence of a power series


Definition 9.1.1. A power series is a series of the form

X
an (x − c)n ,
n=0

where (an )∞
n=1 is a sequence of real numbers, c ∈ R, and x is a variable. The
numbers a0 , a1 , . . . are the coefficients of the power series, and c is called the
centre of the power series.

If we substitute a real number for x, then either the series converges or it


doesn’t; if I is a set of real numbers at which the series does converge, then we
can view the power series as a function I → R. We will, additionally, often

P
view a power series as a series of functions (i.e., as fn where fn : R → R
n=0
is given by fn (x) = an xn ).

an (x − c)n be a power series. The interval of
P
Definition 9.1.2. Let
n=0

141
convergence of this power series is the set

X
{b ∈ R : an (b − c)n converges}.
n=0

It is clear that the interval of convergence always contains the centre, c.


Example 9.1.3. If we take c = 0 and all coefficients equal to 1, we get the
power series

X
xn .
n=0

When we substitute a real number for x, we get the Geometric Series (Ex-
ample 3.1.8). We analyzed earlier exactly what real numbers this series
converges for, and from that analysis we may conclude that the interval of
convergence is (−1, 1). For x in this interval of convergence, we have

X 1
xn = .
n=0
1−x

nn xn . For an x 6= 0, we use the
P
Example 9.1.4. Consider the power series
n=0
ratio test:
n
(n + 1)n+1 xn+1

= (n + 1)|x| n + 1 ≥ (n + 1)|x|,
nn xn n
and therefore,
(n + 1)n+1 xn+1

lim = ∞.
n→∞ nn xn
Consequently, the series does not converge. The interval of convergence of
this power series is therefore {0}.

an (x − c)n be a power series, and define
P
Theorem 9.1.5. Let
n=0

1
R := p
lim sup n
|an |
n→∞

(interpreted as 0 if the lim sup is ∞, and as ∞ if the lim sup is 0). Then for
b ∈ R,

142

an (b − c)n converges, while
P
(i) if |b − c| < R then
n=0

an (b − c)n diverges.
P
(ii) if |b − c| > R then
n=0

Remark 9.1.6. The number R in the above proposition is called the radius

an (x − c)n .
P
of convergence of the power series
n=0
We can rephrase the conclusion of the above proposition in terms of the
interval of convergence of the power series. If R ∈ (0, ∞), it tells us that the
interval of convergence is one of
(c − R, c + R), (c − R, c + R], [c − R, c + R), or [c − R, c + R].
If R = 0 then the interval of convergence is {c}, whereas if R = ∞ then
the interval of convergence is R. In particular, in all cases, the interval of
convergence is an interval.
Proof. Let b ∈ R, b 6= c (since of course the series converges if b = c). We
will use the Root Test (Proposition 3.2.7). We compute
p
n
p |b − c|
lim sup |an (b − c)n | = lim sup |b − c| n |an | = .
n→∞ n→∞ R

|b−c|
an (b − c)n converges if
P
Hence by the Root Test, the series R
< 1, i.e, if
n=0
|b − c| < R, and it diverges if |b−c|
R
> 1, i.e., if |b − c| > R. (Note that these
arguments are valid even in the cases R = 0 or R = ∞, if we allow |b−c| 0
to
|b−c|
mean ∞ and ∞ to mean 0.)
As noted in the remark before the proof, if the radius of convergence

an (x − c)n is R ∈ (0, ∞), then we can conclude that the interval of
P
of
n=0
convergence is one of (c−R, c+R), (c−R, c+R], [c−R, c+R), or [c−R, c+R].
We already saw in Example 9.1.3 a situation where we get (c − R, c + R). In
the following examples, we demonstrate that the other cases are possible.
∞ n
x
P
Example 9.1.7. Consider the power series n
. The radius of convergence
n=1
is
1 1
R= p = = 1.
n
lim sup 1/n 1
n→∞

143
For the endpoint b = −1, we have that

X (−1)n
n=1
n

converges by the Alternating Series Test. For the endpoint b − 1, we have


that ∞
X 1
n=1
n
is the Harmonic Series, and therefore it diverges. Hence the interval of con-
vergence is [−1, 1).

xn
P
Example 9.1.8. Consider the power series n(n+1)
. The radius of conver-
n=1
gence is
1 1
R= p = = 1.
n
lim sup 1/n(n + 1) 1
n→∞

For the endpoint b = −1, once again we can use the Alternating Series Test
to conclude that ∞
X (−1)n
n=1
n(n + 1)
converges. For the endpoint b = 1, we have that

X 1
n=1
n(n + 1)

converges by Example 3.1.5.


We have seen one example where the radius of convergence is 0 (Example
9.1.4. As a simple example of the opposite case, any polynomial can be
viewed as a power series with infinite radius of convergence (it is a power
series in which only finitely many coefficients are nonzero). There are also
examples with infinitely many nonzero coefficients, such as the following.
∞ n
x
P
Example 9.1.9. Consider the power series nn
. The radius of convergence
n=1
is
1 1 1
R= p = = = 0.
lim sup 1/nn
n lim sup 1/n ∞
n→∞ n→∞

144
Hence the interval of convergence is R. (This power series converges to the
exponential function – this is Exercise 9.3.1 in a later section.)
As we saw in Chapter 8, we cannot conclude much about a function just
from knowing that it is a pointwise limit. To advance the theory of power
series, we will want to have uniform convergence; this is the purpose of the
next result.

an (x − c)n be a power series and let R be its
P
Proposition 9.1.10. Let
n=0
radius of convergence. Let [a, b] be any closed bounded interval contained in
(c − R, c + R) (which is R in the case R = ∞). Then the series converges
uniformly on [a, b].

Proof. We use the Weierstrass M -test (Theorem 8.2.6). Set M := max{|a −


c|, |b − c|}, so that M < R. Let Mn := |an M n |. Then using the Root Test as
P∞
in the previous proof, we know that Mn converges.
n=0
For every x ∈ [a, b], we have |an xn | ≤ Mn , and therefore it follows by the
Weierstrass M -test that the series

X
an x n
n=0

converges uniformly on [a, b].


Interesting fact 9.1.11. The above proposition can be strengthened, to say
that for any closed bounded interval [a, b] contained in the interval of conver-

an (x − c)n converges uniformly on [a, b] ([TBB08,
P
gence, the power series
n=0
Theorem 10.10]). This makes a difference when the interval of convergence
includes endpoints; for example, in Example 9.1.8, we saw that the power
series ∞
X xn
n=1
n(n + 1)
has interval of convergence [−1, 1]. The series therefore converges uniformly
on [−1, 1], although Proposition 9.1.10 is not sufficient to tell us this.
∞ n
x
P
Likewise, the series n
converges uniformly on [−1, b) for any b ∈
n=1
(−1, 1).

145
Exercise 9.1.1. Determine the interval of convergence of the following power
series.

n2
− 3)n .
P
(a) 2n
(x
n=0


2n
+ 3)n .
P
(b) n2
(x
n=0


xn
P
(c) n!
.
n=0


P (x−2)n
(d) n ln(n)
(you might use the Integral Test to help sort out convergence
n=2
at the endpoints).

xn
P
(e) n!
.
n=1


an xn where
P
(f)
n=0
(
1, if n = 4k or n = 4k + 1 for some k ∈ N≥0 ;
an =
−1, otherwise.

Exercise 9.1.2. Give an example of a power series whose interval of conver-


gence is (0, π].

an
Exercise 9.1.3. [TBB08, Exercise 10.2.2] Prove that if the limit lim an+1
n→∞
exists, then it is equal to the radius of convergence of the power series

an (x − c)n .
P
n=0
[Warning: this limit doesn’t always exist, whereas the lim sup used to define
the radius of convergence does always exist.]
∞ ∞
an xn and bn xn have radii of convergence Ra and
P P
Exercise 9.1.4. Let
n=0 n=0
Rb respectively.

(an +bn )xn is at least min{Ra , Rb }.
P
(a) Show that the radius of convergence of
n=0

146

an xn+1 .
P
(b) Determine the radius of convergence of
n=0


an xkn where k ∈ N≥1 .
P
(c) Determine the radius of convergence of
n=0

(d) [TBB08, Exercise 10.2.11] If |an | ≤ |bn | for all n, determine what the
relationship is between Ra and Rb .

an xn be a power series with
P
Exercise 9.1.5. [TBB08, Exercise 10.2.12] Let
n=0

an x2n .
P
radius of convergence R. Determine the radius of convergence of
n=0

an xn be a power series with
P
Exercise 9.1.6. [TBB08, Exercise 10.2.13] Let
n=0
radius of convergence R ∈ (0, ∞). Prove that the radius of convergence of

k
an xn is 1.
P
n=0

9.2 Continuity, integration, and differentia-


tion

an (x − c)n be a power series with interval of con-
P
Theorem 9.2.1. Let
n=0
vergence I, and define f : I → R by

X
f (x) := an (x − c)n .
n=0

Then:

(i) f is continuous on I, and

(ii) For any a, b ∈ I,


Z b ∞
X an
(b − c)n+1 − (a − c)n+1 .

f (t) dt =
a n=0
n+1

Proof. We define fn : I → R by fn (x) := an (x − c)n .

147
(i): Let x ∈ I, and first suppose that x − c is less than the radius of
convergence. Then we can find r > 0 such that x ∈ (c − r, c + r). By
Proposition 9.1.10,
X∞
f |[c−r,c+r] = u− fn |[c−r,c+r] .
n=0
Therefore, by Theorem 8.3.1, f |[c−r,c+r] is continuous, and so (by Exercise
5.2.2) f is continuous at x.
If x − c is not less than the radius of convergence, then x is an endpoint of
I, and we use Interesting Fact 9.1.11 to see that the series converges uniformly
on a closed interval containing x, and so f is continuous at x.
(ii): Similarly, using either Proposition 9.1.10 or (if one or both of a, b
are endpoints of I) Interesting Fact 9.1.11, we get that
X∞
f |[a,b] = u− fn |[a,b] ,
n=0

Therefore, by Theorem 8.3.4,


Z b ∞ Z b
X
f (t) dt = fn (t) dt
a n=0 a
X∞ Z b
= an (t − c)n dt
n=0 a

X an
(b − c)n+1 − (a − c)n+1 .

=
n=0
n+1


an (x − c)n be a power series with radius of conver-
P
Theorem 9.2.2. Let
n=0
gence R > 0, and define f : (c − R, c + R) → R by

X
f (x) := an (x − c)n .
n=0

nan (x − c)n−1 also has radius of convergence R,
P
Then the power series
n=1
and for x ∈ (c − R, c + R),

X
f 0 (x) = nan (x − c)n−1 .
n=0

148

Proof. Let R0 be the radius of convergence of the power series
P
nan (x −
√ n=1
c)n−1 . Then since lim n−1 n = 11
n→∞

1
R0 = √
lim sup n−1 nan
n→∞
1
= √ √
lim sup n−1
n n−1 an
n→∞
1
= √
lim sup n−1 an
n→∞
= R.

Now, let x ∈ (c − R, c + R). We may find a closed interval [a, b] contained in


(c − R, c + R) such that x ∈ (a, b).
Let fn (x) := an (x − c)n . Using the radius of convergence we just com-

fn0 converges uniformly on [a, b]. Hence it follows
P
puted, we know that
n=0

from Theorem 8.3.8 that f 0 = fn0 on [a, b], and in particular, at x.
P
n=0

an (x − c)n be a power series with radius of conver-
P
Corollary 9.2.3. Let
n=0
gence R > 0, and define f : (c − R, c + R) → R by

X
f (x) := an (x − c)n .
n=0

Then for each n,


f (n) (c)
an = .
n!
Proof. By induction on n. For n = 0, we have

f (0) (c) f (c)


= = a0 .
0! 1
1 rn √
This follows from the fact that n → ∞ for any r > 1: for if n > 1 then there
n−1

would exist r > 1 such that n−1 n > r for all n sufficiently large, and from this, we get
rn−1 /n < 1, a contradiction.

149
Now assume that it is true for n and let us prove it for n + 1. Set g := f 0 , so
that ∞
X
g(x) = (n + 1)an+1 xn .
n=0

Since the radius of convergence of this power series is the same as R > 0, we
may apply the inductive hypothesis to g, which tells us

g (n) (c)
(n + 1)an+1 = .
n!
Noting that f (n+1) = g (n) , and dividing both sides by n + 1, we get

f (n+1) (c) f (n+1) (c)


an+1 = = .
(n + 1)n! (n + 1)!

Exercise 9.2.1. [TBB08, Exercise 10.4.2] Find power series expansions for
x x
and
1 + x2 (1 + x2 )2

(centred at 0).

an xn be a power series
P
Exercise 9.2.2. (cf. [TBB08, Exercise 10.4.6]) Let
n=0
with radius of convergence R > 0, and define f : (−R, R) → R by

X
f (x) := an x n .
n=0

(a) Prove that if f is even, meaning that f (x) = f (−x) for all x, then an = 0
for all n odd.

(b) Prove that if f is odd , meaning that f (x) = −f (x) for all x, then an = 0
for all n even.

Exercise 9.2.3. In this question, we want to prove that there is a function


f : (−1, 1) → R which satisfies

f (0) = 1, f 0 (x) = f (x2 ).

150
(a) Suppose that there were such a function, and that we could write it as a
power series

X
f (x) = an x n .
n=0
Find recurrence equations for the coefficients (an )∞
n=0 (i.e., an equation
expressing an in terms of a0 , . . . , an−1 ).
(b) With (an )∞
n=0 as in (a), prove that an ≤ 1 for all n.

(c) With (an )∞


n=0 as in (a), prove that the series

X
an x n
n=0

converges on (−1, 1), and that the function f it defines does satisfy the
conditions we began with.
(d) (Difficult) Compute exactly the radius of convergence of this power series.

9.3 Taylor series


The final corollary of the previous section tells us that if a function f can be
“represented” as a power series centred at c (here by “represented” we mean,
that the power series converges to f at all points in some open ball around
c), then we can recover the coefficients of the power series by taking repeated
derivatives of f and evaluating them at c. This suggests the question: when
can a function be represented as a power series (centred at a point c)? Here
we develop tools to answer this question.
Definition 9.3.1. Let I ⊆ R be an open interval and f : I → R be a
function which is infinitely differentiable (meaning that f (n) exists for all n).
For c ∈ I, the Taylor series of f (centred at c) is the power series

X f (n) (c)
(x − c)n .
n=0
n!

For N ∈ N≥0 , the N th Taylor polynomial of f (centred at c) is


N
X f (n) (c)
PN (x) := (x − c)n .
n=0
n!

151
In other words, the N th Taylor polynomial is the unique polynomial of
degree at most N such that
(i)
PN (c) = f (i) (c) for i = 0, . . . , N.

Theorem 9.3.2 (Lagrange Remainder Theorem). Let f : (a, b) → R be a


function for which f 0 , f (2) , . . . , f (N +1) all exist on (a, b), let c ∈ (a, b), and let
PN (x) be the N th Taylor polynomial of f centred at c. Then for x ∈ (a, b),
there exists z between c and x such that
f (N +1) (z)
f (x) − PN (x) = (x − c)N +1 .
(N + 1)!

Proof. Fix x and assume (without loss of generality) that x > c. Define
M := f(x−c)
(x)−PN (x)
N +1 , so that

f (x) − PN (x) = M (x − c)N +1 .


(N +1)
We need to prove that M = f (N +1)!(z) for some z ∈ (x, c). Define g : (a, b) → R
by
g(t) := f (t) − PN (t) − M (t − c)N +1 .
(i)
For i = 0, . . . , N , we have f (i) (c) = PN (c), while the ith derivative of M (t −
c)N +1 , evaluated at c, is 0. Hence,

g (i) (c) = 0.

We also have g(x) = 0 (by definition of M ) so that we may apply Rolle’s


Theorem to g on [c, x], to obtain z1 such that

g 0 (z1 ) = 0.

Now, since g 0 (c) = g 0 (z1 ) = 0, we may apply Rolle’s Theorem to the function
g 0 on [c, z1 ] to obtain z2 such that

g 00 (z2 ) = 0.

We continue in this way, obtaining z1 , . . . , zN +1 with zi ∈ [c, zi−1 ] ⊆ [c, x]


such that
g (i) (zi ) = 0 for i = 1, . . . , N + 1.

152
Now, since PN is a polynomial of degree at most N , its (N + 1)th derivative
is zero, whereas the (N + 1)th derivative of (t − c)N +1 is (N + 1)!. Hence,

0 = g (N +1) (zN +1 )
= f (N +1) (zN +1 ) − M (N + 1)!,

f (N +1) (zN +1 )
and thus M = (N +1)!
, as required.

Example 9.3.3. Let f : R → R be the function f (x) := sin(x). Then we have


for k ∈ N≥0

f (2k) (x) = (−1)k sin(x), f (2k+1) (x) = (−1)k cos(x),

and from this we see that the Taylor series for f centred at 0 is

x3 x 5 x7 X (−1)k x2k+1
x− + − + ··· = .
3! 5! 7! k=0
(2k + 1)!

Let us use the Lagrange Remainder Theorem to prove that the Taylor se-
ries converges to f everywhere. If we let PN (x) be the N th Taylor polynomial,
our task is to prove that lim PN (x) = f (x) for all x ∈ R.
N →∞
So, fix x ∈ R. For each N , by the Lagrange Remainder Theorem, there
exists z between 0 and x such that
f (N +1) (z) N +1
f (x) − PN (x) = x .
(N + 1)!

Since f (N +1) (z) is either ± sin(z) or ± cos(z), we see that |f (N +1) (z)| ≤ 1,
and so
|x|N +1
|f (x) − PN (x)| ≤ .
(N + 1)!
|x| N +1
Now, for any x, we have (N +1)!
→ 0 as N → ∞, and therefore by the
Squeeze Theorem, PN (x) → f (x), as required.
Exercise 9.3.1. Compute the Taylor series for the function f (x) := ex centred
at 0, and use the Lagrange Remainder Theorem to prove that the Taylor
series converges to f (x) for all x ∈ R.

153
Exercise 9.3.2. Fix α ∈ R. For n ∈ N≥1 , define
 
α α(α − 1) · · · (α − n − 1)
:= .
n n!
Consider the function f : [0, ∞) → R given by f (x) := xα .
(a) Prove that for all n ∈ N≥1 and all x ∈ (0, ∞),
 
α 1
(n)
f (x) = n! x 2 −n .
n
(You may use the “Power Rule” for differentiation.)
(b) Determine the Taylor polynomial Pn (x) and the Taylor series for f cen-
tred at 1.
(c) Assuming α > 0, prove that
 
α
(x − 1)n+1 xα ,
|f (x) − Pn (x)| ≤ for x ≥ 1,
n+1
1 − x n+1
   
α
|f (x) − Pn (x)| ≤
, for x ≤ 1.
n+1 x

(d) Prove that  


α
n <1

for all α ∈ (0, 1) and all n ∈ N≥1 .


(e) Prove that if α ∈ (0, 1) and x ∈ ( 21 , 1) then the Taylor series for f (centred
at 1) is equal to f at x.
Exercise 9.3.3. Define
( 2
e−1/x , x 6= 0;
f (x) :=
0, x = 0.

(a) Prove that for each n, there is a polynomial qn (x) such that
 
(n) 1
f (x) = qn f (x),
x
for x 6= 0.

154
(b) Prove that f is infinitely differentiable and that

f (n) (0) = 0
f (x)
for all n. You may use (without proof) that lim k = 0, for any k ∈ N≥1 .
x→0 x

(c) Determine the Taylor series for f centred at 0. What is the radius of
convergence of this power series? For what values of x is f equal to its
Taylor series?

155
Review

Items marked ∗ : you definitely won’t be asked to prove these on the final
exam.
Chapter 1 – The real numbers

• Should be comfortable with working with R (field structure, order struc-


ture, | · |), but don’t need to know the axioms per se.

• Important definition: supremum, infimum.

Chapter 2 – Sequences

• Definitions: boundedness, convergence/divergence, increasing/decreasing/monotone,


subsequence, Cauchy sequence, lim sup/lim inf.

• Should be well-versed at -n0 proofs (proving that a sequence converges,


proving that a sequence does not converge, proving some other property
assuming that a sequence converges).

• Uniqueness of limits

• If a sequence converges then it is bounded.

• Algebra of limits∗ .

• If an ≤ bn for all n then lim an ≤ lim bn (assuming the limits exist).


n→∞ n→∞

• Squeeze Theorem∗ .

• Every real number is a limit of rationals.

• Monotone convergence criterion.

156
• If lim an = L then lim ank = L.
n→∞ k→∞

• Every sequence has a monotone subsequence∗ .

• Bolzano–Weierstrass Theorem.

• Cauchy Convergence Criterion∗ .

• lim inf an ≤ lim sup an .


n→∞ n→∞

• lim inf an = lim inf{an , an+1 , an+2 , . . . } (and likewise for lim sup)∗ .
n→∞ n→∞

• lim an = L if and only if lim sup an = lim inf an = L (Theorem 2.7.6).


n→∞ n→∞ n→∞

• Some good practice exercises: Exercise 2.4.1, 2.6.1, 2.7.6.

Chapter 3 – Series

• Definition: convergence.

• Examples: harmonic series, geometric series.

• Linearity.

P ∞
P
• Order-preserving: if an ≤ bn then an ≤ bn (assuming both con-
n=1 n=1
verge).

P ∞
P
• an converges iff an converges.
n=1 n=m

• Boundedness Test.

• Comparison Test∗ .

• Absolute Convergence Test.

• Ratio Test.

• Root Test∗ .

• Alternating Series Test.

157
• Cauchy Convergence Criterion for Series.

• Some good practice exercises: Exercise 3.2.3, 3.2.6.

Chapter 4 – Topology of Rd

• Definition: norms, k · k2 , convergence in Rd , boundedness, open/closed


sets, A◦ , A, ∂A, limit points, (sequential) compactness.

• Cauchy–Schwarz and ∆-inequalities∗ .


(1) (d) (i)
• lim (an , . . . , an ) = (L1 , . . . , Ld ) iff lim an = Li for i = 1, . . . , d.
n→∞ n→∞

• Cauchy Convergence Criterion for Rd .

• Bolzano–Weierstrass Theorem for Rd .∗

• B(a; r) is open.

• Permanence properties of open sets, closed sets.

• F is closed if and only if F = F (Proposition 4.3.9).

• R, (0, 1] are not compact. Every finite set is compact. (You should be
able to prove these without using the Heine–Borel Theorem.)

• Heine–Borel Theorem∗ .

• Permanence properties of compact sets.

• Some good practice exercises: Exercise 4.3.1, 4.4.6.

Chapter 5 – Continuous functions

• Most of the things in this chapter are done at the general level of func-
tions f : X → Rm , where X ⊆ Rd . However, the most important thing
is to understand these concepts even for functions f : A → R where
A ⊆ R. If you still aren’t comfortable with topology of Rd , then don’t
allow yourself to get bogged down on that aspect; just focus on this case
where we work in R.

158
• Definitions: limit of a function, one-sided limits, continuity, (strictly/weakly)
increasing/decreasing, uniform continuity, infinite limits and limits at
∞.

• Uniqueness of the limit.

• Sequential Characterization of Limits∗ .

• Algebra of Limits, Squeeze Theorem.

• Composition of continuous functions is continuous.

• Algebra of continuous functions (Proposition 5.2.6).

• If K is compact and f is continuous then f (K) is compact.

• Extreme Value Theorem.

• Intermediate Value Theorem.

• If I is an interval and f : I → R is continuous and injective, then it is


strictly increasing or strictly decreasing, and its inverse is continuous∗ .

• If K is compact and f : K → Rd is continuous then it is uniformly


continuous.

• Some good practice exercises: Exercise 5.1.7, 5.3.3, 5.5.5.

Chapter 6 – Differentiation
• Definitions: the derivative/differentiability, local minima/maxima.

• If f is differentiable at a then f is continuous at a.

• Linearity, Product Rule, Chain Rule∗ , Inverse Rule.

• If f is differentiable at a and has a local min/max at a then f 0 (a) = 0.

• Rolle’s Theorem.

• Mean Value Theorem (you don’t need to know Cauchy’s Mean Value
Theorem)∗ .

• Some good practice exercises: Exercise 6.1.5, 6.4.3.

159
Chapter 7 – Integration

• Definitions: partition, mi (P, f ), Mi (P, f ), upper/lower Darboux sums,


integrable/integral, improper integrals.

• Examples: a function that is constant except at one point, a function


that is not Riemann integrable.

• L(P, f ) ≤ U (P 0 , f ) for any partitions, P, P 0 .∗

• f is integrable iff ∀ > 0 ∃ a partition P such that U (P, f )−L(P, f ) < .

• If f is continuous then f is integrable.

• Additive Property.

• Linearity.
Rb Rb
• If f (x) ≤ g(x) for all x then a
f≤ a
g (assuming both exist).

• Fundamental Theorem of Calculus and its consequence (Theorem 7.3.2).

• Some good practice exercises: Exercise 7.1.6. 7.2.1, 7.2.7

Chapter 8 – Sequences and series of functions

• Definitions: pointwise convergence, uniform convergence.

• Examples: sequences (fn )∞


n=1 that converge pointwise to f such that:

– Each fn is continuous but f is not.


– f 0 6= lim fn0 .
n→∞
Rb Rb
– a f 6= lim a fn .
n→∞

• The Weierstrass M -test.

• If each fn is continuous then so is u− lim fn .


n→∞
Rb Rb
• If each fn is integrable then a
u− lim fn = lim fn .
n→∞ n→∞ a

160
• If each fn is differentiable and (fn0 ) converges uniformly then ( lim fn )0 =
n→∞
u− lim fn0 .∗
n→∞

• Some good practice exercises: Exercise 8.2.9, 8.3.3, 8.3.7.

Chapter 9 – Power series

• Definitions: power series, interval of convergence, radius of convergence,


Taylor series/polynomial.

• A power series converges on (c−R, c+R) and diverges on R\[c−R, c+R]


(where c is the centre and R is the radius of convergence).

• A power series converges uniformly on [a, b] for any [a, b] ⊆ (c−R, c+R).

• Continuity, integration, and differentiation of power series.

• Lagrange Remainder Theorem∗ .

• Some good practice exercises: Exercise 9.1.2, 9.2.2, 9.3.1, 9.3.3.

161
Index

M -test, 132 Cauchy–Schwarz Inequality, 55


centre (of a power series), 141
Absolute Convergence Test, 43
Chain Rule, 98
absolute value, 9
Change of Variables (integration),
accumulation point, 64
123
addition (field), 1
closed set, 61
Algebra of Limits, 74
closure (of a set), 64
Alternating Series Test, 48
compact set, 67, 69
Archimedean Property, 7
Comparison Test, 42
bijective function, 83 complete (ordered field), 6
Bolzano–Weierstrass Theorem, 25 component functions, 74
Bolzano–Weierstrass Theorem for continuity (at a point), 77
Rd , 59 continuous function (globally), 80
boundary (of a set), 64 convergence in Rd , 57
boundary point, 64 convergent sequence, 12
bounded sequence, 11 convergent series, 36
bounded set, 4 converges uniformly, 130
bounded sets, sequences in Rd , 59
Darboux sum, 108
Boundedness Test, 41
decreasing function, 84
Cauchy, 58 decreasing sequence, 21
Cauchy Convergence Criterion, 26 derivative, 93
Cauchy Convergence Criterion differentiable function, 93
(for series), 52 Dirichlet function, 79
Cauchy Convergence Criterion for distance between real numbers, 9
Rd , 58 Divergence Test, 40
Cauchy sequence, 26 divergence to ±∞ (sequence), 15
Cauchy’s Mean Value Theorem, divergent sequence, 12
103 divergent series, 36

162
dot product, 55 liminf, 28
limit in Rd , 57
Euclidean norm, 55 limit inferior, 28
even function, 150 limit of a function, 71
eventual upper/lower bound, 28 limit of a sequence, 12
Extreme Value Theorem, 80 limit point, 64
field, 1 limit superior, 28
fixed point, 83 limsup, 28
Fundamental Theorem of Lipschitz function, 90
Calculus, 121 local maximum, 100
local minimum, 100
geometric sequence, 19 lower bound, 4
Geometric Series, 142 lower Darboux sum, 108
geometric series, 40
greatest lower bound, 5 maximum, 5
Mean Value Theorem, 104
harmonic series, 38 metric space, 69
Heine–Borel Theorem, 68 minimum, 5
Monotone Convergence Criterion,
improper integral, 124 22
increasing function, 84 monotone sequence, 22
increasing sequence, 21 multiplication (field), 1
infimum, 5
injective function, 83 norm, 54
integral, 111
odd function, 150
Integral Test, 50, 125
one-to-one, 83
Integration by Parts, 123
onto, 83
interior (of a set), 64
open ball, 61
interior point, 64
open set, 61
Intermediate Value Theorem, 81
ordered field, 3
interval of convergence, 142
inverse function, 83 partition, 107
Inverse Rule, 99 periodic function, 82
isolated point, 64 pointwise convergence, 127
power series, 141
L’Hôpital’s Rule, 105 preimage, 79
Lagrange Remainder Theorem, Product Rule, 97
152
least upper bound, 5 Quotient Rule, 99

163
radius of convergence, 143 supremum, 5
Ratio Test, 44 surjective function, 83
Riemann integrable function, 111
Riemann sum, 117 Taylor polynomial, 151
Rolle’s Theorem, 103 Taylor series, 151
Root Test, 47 telescoping series, 38
topological space, 69
sequence, 11 total order, 3
sequence in Rd , 57 triangle inequality (for R), 9
Sequential Characterization of triangle inequality (for a norm),
Limits, 73 54
sequentially compact set, 67
series, 35 uniformly continuous, 87
Squeeze Theorem (for a function), upper bound, 4
74 upper Darboux sum, 108
Squeeze Theorem (for sequences), vector space, 54
18
subsequence, 23 Weierstrass M -test, 132

164
Bibliography

[Leb16] Jiřı̀ Lebl. Basic analysis: introduction to real analysis (version 4.0).
2016. Available online at http://www.jirka.org/ra/.

[Sav17] Alistair Savage. Elementary real analysis.


MAT2125, Winter 2017 course notes. 2017.
http://alistairsavage.ca/mat2125/notes/MAT2125-
Elementary Real Analysis.pdf.

[TBB08] Brian S. Thomson, Judith B. Bruckner, and Andrew M. Bruckner.


Elementary real analysis, second edition. 2008. Available online at
http://classicalrealanalysis.info/Elementary-Real-Analysis.php.

165

Das könnte Ihnen auch gefallen